Вы находитесь на странице: 1из 135

William A. Adkins, Mark G.

Davidson

ORDINARY DIFFERENTIAL
EQUATIONS
Solution Manual
August 15, 2009

Springer
Berlin Heidelberg New York
Hong Kong London
Milan Paris Tokyo
1
Solutions

Section 1.1
1. The rate of change in the population 𝑃 (𝑡) is the derivative 𝑃 ′ (𝑡). The
Malthusian Growth Law states that the rate of change in the population is
proportional to 𝑃 (𝑡). Thus 𝑃 ′ (𝑡) = 𝑘𝑃 (𝑡), where 𝑘 is the proportionality
constant. Without reference to the 𝑡 variable, the differential equation
becomes 𝑃 ′ = 𝑘𝑃

2. a. This statement mathematically is 𝑏(𝑡) = 𝑏0 𝑃 (𝑡) where we have used


𝑏0 to represent the proportionality constant.
b. This statement translates as 𝑑(𝑡) = 𝑑0 𝑃 2 (𝑡) where we have used 𝑑0 to
represent the proportionality constant.
c. The overall growth rate is 𝑃 ′ (𝑡). Thus the Logistic Growth Law is

𝑃 ′ (𝑡) = 𝑏(𝑡) − 𝑑(𝑡)


= 𝑏0 𝑃 (𝑡) − 𝑑0 𝑃 2 (𝑡)
= (𝑏0 − 𝑑0 𝑃 (𝑡))𝑃 (𝑡).

3. Torricelli’s law states that the√change in height, ℎ′√


(𝑡) is proportional to
the square root of the height, ℎ(𝑡). Thus ℎ′ (𝑡) = 𝜆 ℎ(𝑡), where 𝜆 is the
proportionality constant.

4. The highest order derivative is 𝑦 ′ so the order is 1 and the standard form
is 𝑦 ′ = 𝑡3 /𝑦 2 .

5. The highest order derivative is 𝑦 ′′ so the order is 2. The standard form is


𝑦 ′′ = 𝑡3 /𝑦 ′ .

6. The highest order derivative is 𝑦 ′ so the order is 1 and the standard form
is 𝑦 ′ = (𝑒𝑡 − 𝑡𝑦)/𝑡2 .
4 1 Solutions

7. The highest order derivative is 𝑦 ′′ so the order is 2. The standard form is


𝑦 ′′ = (3𝑦 + 𝑡𝑦 ′ )/𝑡2 .

8. The highest order derivative is 𝑦 ′′ so the order is 2 and the standard form
is 𝑦 ′′ = 𝑡2 − 3𝑦 ′ − 2𝑦.

9. The highest order derivative


√ is 𝑦 (4) so the order is 4. Solving for 𝑦 (4) gives
(4)
the standard form: 𝑦 = (1 − (𝑦 ′′′ )4 )/𝑡.
3

10. The highest order derivative is 𝑦 ′ so the order is 1 and the standard form
is 𝑦 ′ = 𝑡𝑦 4 − 𝑡2 𝑦.

11. The highest order derivative is 𝑦 ′′′ so the order is 3. Solving for 𝑦 ′′′ gives
the standard form: 𝑦 ′′′ = 2𝑦 ′′ − 3𝑦 ′ + 𝑦.

12. The following table summarizes the needed calculations:


Function 𝑦 ′ (𝑡) 2𝑦(𝑡)
𝑦1 (𝑡) = 0 𝑦1′ (𝑡) = 0 2𝑦1 (𝑡) = 0
𝑦2 (𝑡) = 𝑡2 𝑦2′ (𝑡) = 2𝑡 2𝑦2 (𝑡) = 2𝑡2
𝑦3 (𝑡) = 3𝑒2𝑡 𝑦3′ (𝑡) = 6𝑒2𝑡 2𝑦3 (𝑡) = 6𝑒2𝑡
𝑦4 (𝑡) = 2𝑒3𝑡 𝑦4′ (𝑡) = 6𝑒3𝑡 2𝑦4 (𝑡) = 4𝑒3𝑡

Thus 𝑦1 and 𝑦3 are the only solutions.

13. The following table summarizes the needed calculations:


Function 𝑡𝑦 ′ (𝑡) 𝑦(𝑡)
𝑦1 (𝑡) = 0 𝑡𝑦1′ (𝑡) =0 𝑦1 (𝑡) = 0
𝑦2 (𝑡) = 3𝑡 𝑡𝑦2′ (𝑡) = 3𝑡 𝑦2 (𝑡) = 3𝑡
𝑦3 (𝑡) = −5𝑡 𝑡𝑦3′ (𝑡) = −5𝑡 𝑦3 (𝑡) = −5𝑡
3
𝑦4 (𝑡) = 𝑡 𝑡𝑦4′ (𝑡) = 3𝑡 3
𝑦4 (𝑡) = 𝑡3

Thus 𝑦1 , 𝑦2 , and 𝑦3 are solutions.

14. We first write the differential equation in standard form: 𝑦 ′′ = −4𝑦. The
following table summarizes the needed calculations:
Function 𝑦 ′′ (𝑡) −4𝑦(𝑡)
2𝑡
𝑦1 (𝑡) = 𝑒 𝑦1′′ (𝑡) = 4𝑒 2𝑡
−4𝑦1 (𝑡) = −4𝑒2𝑡
𝑦2 (𝑡) = sin 2𝑡 𝑦2′′ (𝑡) = −4 sin 2𝑡 −4𝑦2 (𝑡) = −4 sin 2𝑡
𝑦3 (𝑡) = cos(2𝑡 − 1) 𝑦3′′ (𝑡) − 4 cos(2𝑡 − 1) −4𝑦3 (𝑡) = −4 cos(2𝑡 − 1)
2
𝑦4 (𝑡) = 𝑡 𝑦4′′ (𝑡) =2 −4𝑦4 (𝑡) = −4𝑡2
1 Solutions 5

Thus 𝑦2 and 𝑦3 are solutions.

15. The following table summarizes the needed calculations:


Function 𝑦 ′ (𝑡) 2𝑦(𝑡)(𝑦(𝑡) − 1)
𝑦1 (𝑡) = 0 𝑦1′ (𝑡) =0 2𝑦1 (𝑡)(𝑦1 (𝑡) − 1) = 2 ⋅ 0 ⋅ (−1) = 0
𝑦2 (𝑡) = 1 𝑦2′ (𝑡) =0 2𝑦2 (𝑡)(𝑦2 (𝑡) − 1) = 2 ⋅ 1 ⋅ 0 = 0
𝑦3 (𝑡) = 2 𝑦3′ (𝑡) =0 2𝑦3 (𝑡)(𝑦3 (𝑡) − 1) = 2 ⋅ 2 ⋅ 1 = 4
2𝑡
( )
1 2𝑒 1 1
𝑦4 (𝑡) = 1−𝑒2𝑡 𝑦4′ (𝑡) = (1−𝑒2𝑡 )2 2𝑦4 (𝑡)(𝑦4 (𝑡) − 1) = 2 1−𝑒 2𝑡 1−𝑒 2𝑡 − 1
1 𝑒2𝑡 2𝑒2𝑡
= 2 1−𝑒 2𝑡 1−𝑒2𝑡 = (1−𝑒2𝑡 )2

Thus 𝑦1 , 𝑦2 , and 𝑦4 are solutions.

16. The following table summarizes the needed calculations:


Function 2𝑦(𝑡)𝑦 ′ (𝑡) 1
𝑦1 (𝑡) = 1 2𝑦1 (𝑡)𝑦1′ (𝑡) =0 1
𝑦2 (𝑡) = 𝑡 2𝑦2 (𝑡)𝑦2′ (𝑡) = 2𝑡 1
𝑦3 (𝑡) = ln 𝑡 2𝑦3 (𝑡)𝑦3′ (𝑡) = 2 1𝑡
ln 𝑡 = 2 ln𝑡
𝑡
1
√ √
𝑦4 (𝑡) = 𝑡 − 4 2𝑦4 (𝑡)𝑦4′ (𝑡) =2 𝑡− 4 2√1𝑡−4 =1 1

Thus 𝑦4 is the only solution.

17. The following table summarizes the needed calculations:


Function 2𝑦(𝑡)𝑦 ′ (𝑡) 𝑦2 + 𝑡 − 1
√ √ −1 √
𝑦1 (𝑡) = −𝑡 2 −𝑡 √ = −1 ( −𝑡)2 + 𝑡 − 1 = −1
2 −𝑡
√ √ −(𝑒𝑡 − 1) √
𝑦2 (𝑡) = − 𝑒𝑡 − 𝑡 −2 𝑒𝑡 − 𝑡 √ 𝑡 = 𝑒𝑡 − 1 (− 𝑒𝑡 − 𝑡)2 + 𝑡 − 1 = 𝑒𝑡 − 1
2 𝑒 −𝑡
√ √ 1 √
𝑦3 (𝑡) = 𝑡 2 𝑡 √ =1 ( 𝑡)2 + 𝑡 − 1 = 2𝑡 − 1
2 𝑡
√ √ 1 √
𝑦4 (𝑡) = − −𝑡 2(− −𝑡) √ = −1 (− −𝑡))2 + 𝑦 − 1 = −1
2 −𝑡
Thus 𝑦1 , 𝑦2 , and 𝑦4 are solutions.

18. The following table summarizes the needed calculations for the first three
functions:
6 1 Solutions

𝑦 2 (𝑡) − 4𝑦(𝑡)𝑡 + 6𝑡2


Function 𝑦 ′ (𝑡)
𝑡2
𝑡 − 4𝑡 + 6𝑡2
2 2
𝑦1 (𝑡) = 𝑡 1 =3
𝑡2
2 2 2
4𝑡 − 8𝑡 + 6𝑡
𝑦2 (𝑡) = 2𝑡 2 =2
𝑡2
9𝑡2 − 12𝑡2 + 6𝑡2
𝑦3 (𝑡) = 3𝑡 3 =3
𝑡2
3𝑡 + 2𝑡2 𝑡(3 + 2𝑡)
For 𝑦4 (𝑡) = = the quotient rule and simplifying gives
1+𝑡 1+𝑡
2𝑡2 + 4𝑡 + 3
𝑦4′ (𝑡) = . On the other hand,
(1 + 𝑡)2
𝑡2 (3 + 2𝑡)2 4𝑡2 (3 + 2𝑡)
− + 6𝑡2
𝑦42 (𝑡) − 4𝑦4 (𝑡)𝑡 + 6𝑡 2
(1 + 𝑡) 2 (1 + 𝑡)
=
𝑡2 𝑡2
(3 + 2𝑡)2 − 4(3 + 2𝑡)(1 + 𝑡) + 6(1 + 𝑡)2
=
(1 + 𝑡)2
2
2𝑡 + 4𝑡 + 3
= .
(1 + 𝑡)2
It follows that 𝑦2 , 𝑦3 , and 𝑦4 are solutions.

19.
𝑦 ′ (𝑡) = 3𝑐𝑒3𝑡
3𝑦 + 12 = 3(𝑐𝑒3𝑡 − 4) + 12 = 3𝑐𝑒3𝑡 − 12 + 12 = 3𝑐𝑒3𝑡 .
Note that 𝑦(𝑡) is defined for all 𝑡 ∈ ℝ.

20.
𝑦 ′ (𝑡) = −𝑐𝑒−𝑡 + 3
−𝑦(𝑡) + 3𝑡 = −𝑐𝑒−𝑡 − 3𝑡 + 3 + 3𝑡 = −𝑐𝑒−𝑡 + 3.
Note that 𝑦(𝑡) is defined for all 𝑡 ∈ ℝ.

21.
𝑐𝑒𝑡
𝑦 ′ (𝑡) =
(1 − 𝑐𝑒𝑡 )2
1 1 1 − (1 − 𝑐𝑒𝑡 ) 𝑐𝑒𝑡
𝑦 2 (𝑡) − 𝑦(𝑡) = − = = .
(1 − 𝑐𝑒𝑡 )2 1 − 𝑐𝑒𝑡 (1 − 𝑐𝑒𝑡 )2 (1 − 𝑐𝑒𝑡 )2
If 𝑐 ≤ 0 then the denominator 1 − 𝑐𝑒𝑡 > 0 and 𝑦(𝑡) has domain ℝ. If 𝑐 > 0
then 1 − 𝑐𝑒𝑡 = 0 if 𝑡 = ln 1𝑐 = − ln 𝑐. Thus 𝑦(𝑡) is defined either on the
interval (−∞, − ln 𝑐) or (− ln 𝑐, ∞).
1 Solutions 7

22.
2 2
𝑦 ′ (𝑡) = 𝑐𝑒𝑡 2𝑡 = 2𝑐𝑡𝑒𝑡
2
2𝑡𝑦(𝑡) = 2𝑡𝑐𝑒𝑡 .

23.
−𝑐𝑒𝑡
𝑦 ′ (𝑡) =
𝑐𝑒𝑡 − 1
𝑡 −1 −𝑐𝑒𝑡
−𝑒𝑦 − 1 = −𝑒− ln(𝑐𝑒 −1)
−1= − 1 = .
𝑐𝑒𝑡 − 1 𝑐𝑒𝑡 − 1

24. We first calculate 𝑦 ′ (𝑡) = −𝑐(𝑡 + 1)−2 so


−𝑐 𝑐 −𝑐 𝑐
(𝑡 + 1)𝑦 ′ (𝑡) + 𝑦(𝑡) = (𝑡 + 1) + = + = 0.
(𝑡 + 1)2 𝑡+1 𝑡+1 𝑡+1

Observe that 𝑦(𝑡) is not defined at 𝑡 = −1 so the two intervals where 𝑦 is


defined are (−∞, −1) and (−1, ∞).

25.
1
𝑦 ′ (𝑡) = −(𝑐 − 𝑡)−2 (−1) =
(𝑐 − 𝑡)2
1
𝑦 2 (𝑡) = .
(𝑐 − 𝑡)2

The denominator of 𝑦(𝑡) is 0 when 𝑡 = 𝑐. Thus the two intervals where


𝑦(𝑡) is defined are (−∞, 𝑐) and (𝑐, ∞).

26. This is a differential equation we can solve by simple integration: We get


2
𝑦(𝑡) = 𝑡2 + 3𝑡 + 𝑐.
𝑒2𝑡
27. Integration gives 𝑦(𝑡) = 2 − 𝑡 + 𝑐.

28. Integration (by parts) gives 𝑦(𝑡) = −𝑡𝑒−𝑡 − 𝑒−𝑡 + 𝑐.


𝑡+1
29. Observe that 𝑡 = 1 + 1𝑡 . Integration gives 𝑦(𝑡) = 𝑡 + ln ∣𝑡∣ + 𝑐.
𝑡3 2
30. We integrate two times. First, 𝑦 ′ (𝑡) = 𝑡2 + 𝑡 + 𝑐1. Second, 𝑦(𝑡) = 3 + 𝑡2 +
𝑐1 𝑡 + 𝑐 2 .

31. We integrate two times. First, 𝑦 ′ (𝑡) = −2 cos 3𝑡 + 𝑐1 . Second, 𝑦(𝑡) =


−2
3 sin 3𝑡 + 𝑐1 𝑡 + 𝑐2 .

32. From Problem 19 the general solution is 𝑦(𝑡) = 𝑐𝑒3𝑡 − 4. At 𝑡 = 0 we get


−2 = 𝑦(0) = 𝑐𝑒0 − 4 = 𝑐 − 4. It follows that 𝑐 = 2 and 𝑦(𝑡) = 2𝑒3𝑡 − 4.
8 1 Solutions

33. From Problem 20 the general solution is 𝑦(𝑡) = 𝑐𝑒−𝑡 + 3𝑡 − 3. At 𝑡 = 0


we get 0 = 𝑦(0) = 𝑐𝑒0 + 3(0) − 3 = 𝑐 − 3. It follows that 𝑐 = 3 and
𝑦(𝑡) = 3𝑒−𝑡 + 3𝑡 − 3.

34. From Problem 21 the general solution is 𝑦(𝑡) = 1/(1 − 𝑐𝑒𝑡 ). At 𝑡 = 0 we


1
get 1/2 = 𝑦(0) = 1−𝑐 . It follows that 𝑐 = −1 and 𝑦(𝑡) = 1/(1 + 𝑒𝑡 ).

35. From Problem 24 the general solution is 𝑦(𝑡) = 𝑐(𝑡 + 1)−1 . At 𝑡 = 1


we get −9 = 𝑦(1) = 𝑐(1 + 1)−1 = 𝑐/2. It follows that 𝑐 = −18 and
𝑦(𝑡) = −18(𝑡 + 1)−1 .

36. From Problem 27 the general solution is 𝑦(𝑡) = 𝑒2𝑡 /2 − 𝑡 + 𝑐. Evaluation


at 𝑡 = 0 gives 4 = 𝑒0 /2 − 0 + 𝑐 = 1/2 + 𝑐. Hence 𝑐 = 7/2 and

37. From Problem 28 the general solution is 𝑦(𝑡) = −𝑡𝑒−𝑡 −𝑒−𝑡 +𝑐. Evaluation
at 𝑡 = 0 gives −1 = 𝑦(0) = −1 + 𝑐 so 𝑐 = 0. Hence 𝑦(𝑡) = −𝑡𝑒−𝑡 − 𝑒−𝑡 .

38. From Problem 31 the general solution is 𝑦(𝑡) = −2 3 sin 3𝑡 + 𝑐1 𝑡 + 𝑐2 and


a 𝑦 ′ (𝑡) = −2 cos 3𝑡 + 𝑐1 . Evaluation at 𝑡 = 0 gives 1 = 𝑦(0) = 𝑐2 and
2 = 𝑦 ′ (0) = −2 + 𝑐1 . If follows that 𝑐1 = 4 and 𝑐2 = 1. Thus 𝑦(𝑡) =
−2
3 sin 3𝑡 + 4𝑡.

39. Implicit differentiation with respect to 𝑡 gives 6𝑡 + 8𝑦𝑦 ′ = 0.

40. Implicit differentiation with respect to 𝑡 gives 2𝑦𝑦 ′ − 2𝑡 − 3𝑡2 = 0.

41. Differentiation gives 𝑦 ′ = 2𝑐𝑒2𝑡 + 1. However, from the given function we


have 𝑐𝑒2𝑡 = 𝑦 − 𝑡. Substitution gives 𝑦 ′ = 2(𝑦 − 𝑡) + 1 = 2𝑦 − 2𝑡 + 1.

42. Differentiation gives 𝑦 ′ = 3𝑐𝑡2 + 2𝑡. However, from the given function
2
we have 𝑐𝑡3 = 𝑦 − 𝑡2 and hence 𝑐𝑡2 = 𝑦−𝑡 ′
𝑡 . Substitution gives 𝑦 =
2
3 𝑦−𝑡
𝑡 + 2𝑡 = 3𝑦
𝑡 − 𝑡.
1 Solutions 9

Section 1.2
1. 𝑦′ = 𝑡
5

0
y

−1

−2

−3

−4

−5
−5 −4 −3 −2 −1 0 1 2 3 4 5
t

2. 𝑦′ = 𝑦2
5

0
y

−1

−2

−3

−4

−5
−5 −4 −3 −2 −1 0 1 2 3 4 5
t

3. 𝑦 ′ = 𝑦(𝑡 + 𝑡)
5

0
y

−1

−2

−3

−4

−5
−5 −4 −3 −2 −1 0 1 2 3 4 5
t
10 1 Solutions

4. 4

−1

−2

−3

−4

−4 −3 −2 −1 0 1 2 3 4

5. 5

0
y

−1

−2

−3

−4

−5
−5 −4 −3 −2 −1 0 1 2 3 4 5
t

6. 5

0
y

−1

−2

−3

−4

−5
−5 −4 −3 −2 −1 0 1 2 3 4 5
t
1 Solutions 11

7. 5

0
y

−1

−2

−3

−4

−5
−5 −4 −3 −2 −1 0 1 2 3 4 5
t

8. 5

0
y

−1

−2

−3

−4

−5
−5 −4 −3 −2 −1 0 1 2 3 4 5
t

9. 5

0
y

−1

−2

−3

−4

−5
−5 −4 −3 −2 −1 0 1 2 3 4 5
t

10. We set 𝑦 2 = 0 and see that 𝑦 = 0 is the only constant (= equilibrium)


solution.

11. We set 𝑦(𝑦 + 𝑡) = 0. We look for constant solutions to 𝑦(𝑦 + 𝑡) = 0, and


we see that 𝑦 = 0 is the only constant (= equilibrium) solution.
12 1 Solutions

12. The equation 𝑦 − 𝑡 = 0 has no constant solution. Thus, there are no


equilibrium solutions.

13. The equation 1 − 𝑦 2 = 0 has two constant solutions: 𝑦 = 1 and 𝑦 = −1

14. We substitute 𝑦 = 𝑎𝑡 + 𝑏 into 𝑦 ′ = 𝑦 − 𝑡 to get 𝑎 = (𝑎 − 1)𝑡 + 𝑏. Equality


for all 𝑡 forces 𝑎 − 1 = 0 and 𝑎 = 𝑏. Thus 𝑎 = 1 and 𝑏 = 1 and the only
linear solution is 𝑦 = 𝑡 + 1.

15. We substitute 𝑦 = 𝑎𝑡 + 𝑏 into 𝑦 ′ = cos(𝑡 + 𝑦) to get 𝑎 = cos((𝑎 + 1)𝑡 + 𝑏).


Equality for all 𝑡 means that cos((𝑎 + 1)𝑡 + 𝑏) must be a constant function,
which can occur only if the coefficient of 𝑡 is 0. This forces 𝑎 = −1 leaving
us with the equation −1 = cos 𝑏. This implies 𝑏 = (2𝑛 + 1)𝜋, where 𝑛 is an
integer. Hence 𝑦 = −𝑡 + (2𝑛 + 1)𝜋, 𝑛 ∈ ℤ is a family of linear solutions.

Section 1.3
1. separable; ℎ(𝑡) = 1 and 𝑔(𝑦) = 2𝑦(5 − 𝑦)

2. In standard form we get 𝑦 ′ = (1 − 𝑦)/𝑦. This is separable; ℎ(𝑡) = 1 and


𝑔(𝑦) = (1 − 𝑦)/𝑦.

3. First write in standard form: 𝑦 ′ = 1−2𝑡𝑦


𝑡2 . We cannot write
1−2𝑡𝑦
𝑡2 as a
product of a function of 𝑡 and a function of 𝑦. It is not separable.

4. In standard form we get 𝑦 ′ = 𝑦(𝑦 − 𝑡). We cannot write 𝑦(𝑦 − 𝑡) as a


product of a function of 𝑡 and a function of 𝑦. It is not separable.

5. Write in standard form to get: 𝑦 ′ = (𝑦 − 2𝑦𝑡)/𝑦. Here we can write (𝑦 −


2𝑡𝑦)/𝑦 = 1 − 2𝑡. It is separable; ℎ(𝑡) = 1 − 2𝑡 and 𝑔(𝑦) = 1.

6. We can factor to get 𝑦 ′ = 𝑦 2 (𝑡 − 1) + 𝑡 − 1 = (𝑦 2 + 1)(𝑡 − 1). It is separable;


ℎ(𝑡) = 𝑡 − 1 and 𝑔(𝑦) = 𝑦 2 + 1.

7. In standard form we get 𝑦 ′ = 𝑡2−2𝑡𝑦 ′ −2𝑡𝑦


+3𝑦 2 . We cannot write 𝑦 = 𝑡2 +3𝑦 2 as a
product of a function of 𝑡 and a function of 𝑦. It is not separable

8. It is not separable as 𝑡2 + 𝑦 2 cannot be written as a product of a function


of 𝑡 and a function of 𝑦.

9. In standard form we get: 𝑦 ′ = 𝑒−𝑡 (𝑦 3 − 𝑦) It is separable; ℎ(𝑡) = 𝑒−𝑡 and


𝑔(𝑦) = 𝑦 3 − 𝑦

10. The variables are already separated, so integrate both sides of the equa-
tion to get 𝑦 2 /2 = 𝑡2 /2 + 𝑐, which we can rewrite as 𝑦 2 − 𝑡2 = 𝑘 where
𝑘 = 2𝑐 ∈ ℝ is a constant. Since 𝑦(2) = −1, substitute 𝑡 = 2 and 𝑦 = −1
to get that 𝑘 = (−1)2 − 22 = −3. Thus the solution is given implicitly by
1 Solutions 13

the equation 𝑦 2 − 𝑡2 = −3 or we can solve explicitly to get 𝑦 = − 𝑡2 − 3,
where the negative square root is used since 𝑦(2) = −1 < 0.
2
11. In standard form we get 𝑦 ′ = 1−𝑦 𝑡𝑦 . Clearly, 𝑦 = ±1 are equilibrium
solutions. Separating the variables gives
𝑦 1
2
𝑑𝑦 = 𝑑𝑡.
1−𝑦 𝑡

Integrating both sides of this equation (using the substitution 𝑢 = 1 − 𝑦 2 ,


𝑑𝑢 = −2𝑦 𝑑𝑦 for the integral on the left) gives
1
− ln ∣1 − 𝑦 2 ∣ = ln ∣𝑡∣ + 𝑐.
2
Multiplying by −2, taking the exponential of both sides, and removing
the absolute values gives 1 − 𝑦 2 = 𝑘𝑡−2 where 𝑘 is a nonzero constant.
However, when 𝑘 = 0 the equation becomes 1 − 𝑦 2 = 0 and hence 𝑦 = ±1.
By considering an arbitrary constant (which we will call 𝑐), the implicit
equation 𝑡2 (1 − 𝑦 2 ) = 𝑐 includes the two equilibrium solutions for 𝑐 = 0.

12. The variables are already separated, so integrate both sides to get 𝑦 4 /4 =
𝑡2 /2 + 𝑐, 𝑐 a real constant. This can be simplified to 𝑦 4 = 2𝑡2 + 𝑐. (where
we replace 4𝑐 by 𝑐) We leave the answer in implicit form.

13. The variables are already separated, so integrate both sides to get 𝑦 5 /5 =
𝑡2 /2 + 2𝑡 + 𝑐, 𝑐 a real constant. Simplifying gives 𝑦 5 = 25 𝑡2 + 10𝑡 + 𝑐. We
leave the answer in implicit form

14. There is an equilibrium solution 𝑦 = 0. Separating variables give 𝑦 −2 𝑦 ′ =


𝑡 and integrating gives −𝑦 −1 = 𝑡2 /2 + 𝑐. Thus 𝑦 = −2/(𝑡2 + 2𝑐), 𝑐 a real
constant. This is equivalent to writing 𝑦 = −2/(𝑡2 + 𝑐), 𝑐 a real constant,
since twice an arbitrary constant is still an arbitrary constant.

15. In standard form we get 𝑦 ′ = (1 − 𝑦) tan 𝑡 so 𝑦 = 1 is a solution. Sepa-


𝑑𝑦
rating variables gives 1−𝑦 = tan 𝑡 𝑑𝑡. The function tan 𝑡 is continuous on
the interval (−𝜋/2, 𝜋/2) and so has an antiderivative. Integration gives
− ln ∣1 − 𝑦∣ = − ln ∣cos 𝑡∣+𝑘1 . Multiplying by −1 and exponentiating gives
∣1 − 𝑦∣ = 𝑘2 ∣cos 𝑡∣ where 𝑘2 is a positive constant. Removing the absolute
value signs gives 1−𝑦 = 𝑘3 cos 𝑡, with 𝑘3 ∕= 0. If we allow 𝑘3 = 0 we get the
equilibrium solution 𝑦 = 1. Thus the solution can be written 𝑦 = 1−𝑐 cos 𝑡,
𝑐 any real constant.

16. An equilibrium solution is 𝑦 = 0. Separating variables gives 𝑦 −𝑛 𝑑𝑦 =


1−𝑛 𝑚+1
𝑡𝑚 𝑑𝑡 and integrating gives 𝑦1−𝑛 = 𝑡𝑚+1 + 𝑐, 𝑐 a real constant. Simplifying
1−𝑛 𝑚+1
gives 𝑦 1−𝑛 = 𝑚+1 𝑡 + 𝑐, and the equilibrium solution 𝑦 = 0.
14 1 Solutions

17. There are two equilibrium solutions; 𝑦 = ( 0 and 𝑦)= 4. Separating vari-
1 1 1
ables and using partial fractions gives 4 𝑦 + 4−𝑦 𝑑𝑦 = 𝑑𝑡. Integrating

𝑦 𝑦
and simplifying gives ln 4−𝑦 = 4𝑡 + 𝑘1 which is equivalent to 4−𝑦 = 𝑐𝑒4𝑡 ,
4𝑡
4𝑐𝑒
𝑐 a nonzero constant. Solving for 𝑦 gives 𝑦 = 1+𝑐𝑒 4𝑡 . When 𝑐 = 0 we get

the equilibrium solution 𝑦 = 0. However, there is no 𝑐 which gives the


other equilibrium solution 𝑦 = 4.

18. There are no equilibrium solutions. Separating variables gives 𝑦2𝑦+1 𝑑𝑦 =


𝑑𝑡 and integrating gives 21 ln(𝑦 2 + 1) = 𝑡 + 𝑘. Solving for 𝑦 2 gives 𝑦 2 =
𝑐𝑒2𝑡 − 1, where 𝑐 > 0.

19. Separating variables gives 𝑦2𝑑𝑦


+1 = 𝑑𝑡 and integrating gives tan
−1
𝑦 = 𝑡+𝑐.
Thus 𝑦 = tan(𝑡 + 𝑐), 𝑐 a real constant.
𝑦2
20. Separating variables gives 𝑦 𝑑𝑦 = −1
( )
𝑡 − 𝑡 𝑑𝑡 and integrating gives 2 =
2
− ln ∣𝑡∣ − 𝑡2 + 𝑐. Simplifying gives 𝑦 2 + 𝑡2 + ln 𝑡2 = 𝑐, 𝑐 a real constant.

21. In standard form we get 𝑦 ′ = −(𝑦+1) 1


𝑦−1 1+𝑡2 from which we see that 𝑦 =
( is an )
−1 equilibrium solution. Separating variables and simplifying gives
2 𝑑𝑡 2
𝑦+1 − 1 𝑑𝑦 = 𝑡2 +1 . Integrating and simplifying gives ln(𝑦 + 1) − 𝑦 =
tan−1 𝑡 + 𝑐.

22. Separating variables gives 2𝑦 𝑑𝑦 = 𝑒𝑡 𝑑𝑡 and integrating gives 𝑦 2 = 𝑒𝑡 + 𝑐,


𝑐 a constant.

23. The equilibrium solution is 𝑦 = 0. Separating variables gives 𝑦 −2 𝑑𝑦 =


𝑑𝑡 1
1−𝑡 . Integrating and simplifying gives 𝑦 = ln∣1−𝑡∣+𝑐 , 𝑐 real constant.

24. In standard form we get 𝑦 ′ = 𝑦(𝑦+1) from which we see 𝑦 = 0 and 𝑦 = −1


are equilibrium solutions. The equilibrium solution 𝑦(𝑡) = 0 satisfies the
initial condition 𝑦(0) = 0 so 𝑦(𝑡) = 0 is the required solution.

25. 𝑦 = 0 is the only equilibrium solution. The equilibrium solution 𝑦(𝑡) = 0


satisfies the initial condition 𝑦(1) = 0 so 𝑦(𝑡) = 0 is the required solution.
𝑑𝑦
26. Rewriting we get 𝑦 ′ = 𝑑𝑥 = 𝑥+2
𝑥 𝑦 from which we see (that 𝑦 = 0 is
𝑑𝑦 2
)
an equilibrium solution. Separating variables gives 𝑦 = 1 + 𝑥 𝑑𝑥 and
integrating gives ln ∣𝑦∣ = 𝑥 + ln 𝑥2 + 𝑘, 𝑘 a constant. Solving for 𝑦 by
taking the exponential of both sides gives 𝑦 = 𝑐𝑥2 𝑒𝑥 , and allowing 𝑐 = 0
gives the equilibrium solution. The initial condition gives 𝑒 = 𝑦(1) = 𝑐𝑒
so 𝑐 = 1. Thus 𝑦 = 𝑥2 𝑒𝑥 .

27. In standard form we get 𝑦 ′ = −2𝑡𝑦 so 𝑦 = 0 is a solution. Separating vari-


2
ables and integrating gives ln ∣𝑦∣ = −𝑡2 + 𝑘. Solving for 𝑦 gives 𝑦 = 𝑐𝑒−𝑡
1 Solutions 15

and allowing 𝑐 = 0 gives the equilibrium solution. The initial condition


2
implies 4 = 𝑦(0) = 𝑐𝑒0 = 𝑐. Thus 𝑦 = 4𝑒−𝑡 .

28. Since cot 𝑦 = 0 at 𝑦 = 𝜋2 + 𝑚𝜋 for all integers 𝑚 we have equilibrium


lines at 𝑦 = 𝜋2 + 𝑚𝜋, none of which satisfy the initial condition 𝑦(1) = 𝜋4 .
Separating variables gives tan 𝑦 𝑑𝑦 = 𝑑𝑡𝑡 and integrating gives − ln ∣cos 𝑦∣ =
ln 𝑡 + 𝑐. We can (solve
√ )
for 𝑐 here using the initial condition: we get 𝑐 =
− ln cos 4 = − ln 2 . Solving for 𝑦 gives 𝑦 = cos−1 √12𝑡
𝜋 2

29. Separating variables gives 𝑑𝑦 𝑢


𝑦 = 𝑢2 +1 𝑑𝑢√and integrating gives ln ∣𝑦∣ =

ln 𝑢2 + 1 + 𝑘. Solving for 𝑦 gives 𝑦 √= 𝑐 𝑢2 + 1, for 𝑐 ∕= 0. The initial
condition gives 2 = 𝑦(0) = 𝑐. So 𝑦 = 2 𝑢2 + 1.
𝑡
30. In standard form we get 𝑦 ′ = 𝑡+2 so 𝑦 = 0 is an equilibrium solution.
𝑦
( )
𝑑𝑦 2
Separating variables gives 𝑦 = 1 − 𝑡+2 𝑑𝑡. Integrating we get ln ∣𝑦∣ =
𝑡
𝑒
𝑡 − 2 ln ∣𝑡 + 2∣ + 𝑘. Solving for 𝑦 we get 𝑦 = 𝑐 (𝑡+2) 2 , for 𝑐 ∕= 0. However,

allowing 𝑐 = 0 gives the equilibrium solution.

31. We assume the decay model 𝑁 (𝑡) = 𝑁 (0)𝑒−𝜆𝑡 . If 𝑡 is the age of the
bone then 𝑁 (𝑡) = 31 𝑁 (0). Thus 13 = 𝑒−𝜆𝑡 . Solving for 𝑡 gives 𝑡 = ln𝜆3 =
5730 ln 3
ln 2 ≈ 9082 years

32. Let 𝑚 denote the number of Argon-40 atoms in the sample. Then 8𝑚 is
the number of Potassium-40 atoms. Let 𝑡 be the age of the rock. Then
𝑡 years ago there were 𝑚 + 8𝑚 = 9𝑚 atoms of Potassium-40. Hence
𝑁 (0) = 9𝑚. On the other hand, 8𝑚 = 𝑁 (𝑡) = 𝑁 (0)𝑒−𝜆𝑡 = 9𝑚𝑒−𝜆𝑡 . This
− ln 8
implies that 98 = 𝑒−𝜆𝑡 and hence 𝑡 = 𝜆 9 = ln
−𝜏 8
2 ln 9 ≈ 212 million years
old.

33. We need only solve .3𝑁 (0) = 𝑁 (0)𝑒−𝜆𝑡 for 𝑡. We get 𝑡 = − ln𝜆.3 =
− 5.27 ln .3
ln 2 = 9.15 years.

34. The ambient temperature is 32∘ F, the temperature of the ice water.
From Equation (12) we get 𝑇 (𝑡) = 32 + 𝑘𝑒𝑟𝑡 . At 𝑡 = 0 we get 70 =
32 + 𝑘, so 𝑘 = 38 and 𝑇 (𝑡) = 32 + 38𝑒𝑟𝑡 . After 30 minutes we have
1
55 = 𝑇 (30) = 32 + 38𝑒30𝑟 and solving for 𝑟 gives 𝑟 = 30 ln 23
38 . To find the
𝑟𝑡
time 𝑡 when 𝑇 (𝑡) = 45 we solve 45 = 32 + 38𝑒 , with 𝑟 as above. We get
ln 13−ln 38
𝑡 = 30 ln 23−ln 38 ≈ 64 minutes.

35. The ambient temperature is 𝑇𝑎 = 70∘ . Equation (12) gives 𝑇 (𝑡) =


70 + 𝑘𝑒𝑟𝑡 for the temperature of the coffee at time 𝑡. Since the initial
temperature of the coffee is 𝑇 (0) = 180 we get 180 = 𝑇 (0) = 70 + 𝑘.
Thus 𝑘 = 110. The constant 𝑟 is determined from the temperature at a
second time: 140 = 𝑇 (3) = 70 + 110𝑒3𝑟 so 𝑟 = 13 ln 11
7
≈ −.1507. Thus
16 1 Solutions

𝑇 (𝑡) = 70 + 110𝑒𝑟𝑡, with 𝑟 as calculated. The temperature requested is


( 7 ) 53
𝑇 (5) = 70 + 110 11 ≈ 121.8∘.

36. The ambient temperature is 𝑇𝑎 = 65∘ . Equation (12) gives 𝑇 (𝑡) = 65 +


𝑘𝑒𝑟𝑡 for the temperature at time 𝑡. Since the initial temperature of the
thermometer is 𝑇 (0) = 90 we get 90 = 𝑇 (0) = 65 + 𝑘. Thus 𝑘 = 25. The
constant 𝑟 is determined from the temperature at a second time: 85 =
𝑇 (2) = 65 + 25𝑒2𝑟 so 𝑟 = 12 ln 45 . Thus 𝑇 (𝑡) = 65 + 25𝑒𝑟𝑡 , with 𝑟 = 12 ln 54 .
To answer the first question we solve the equation 75 = 𝑇 (𝑡) = 65 + 25𝑒 𝑟𝑡
ln 2−ln 5
for 𝑡. We get 𝑡 = 2 ln ≈ 8.2 minutes. The temperature at 𝑡 = 20 is
( 4 )104−ln 5 ∘
𝑇 (20) = 65 + 25 5 ≈ 67.7 .

37. The ambient temperature is 𝑇𝑎 = 70∘ . Equation (12) gives 𝑇 (𝑡) = 70 +


𝑘𝑒𝑟𝑡 for the temperature of the soda at time 𝑡. Since the initial temperature
of the soda is 𝑇 (0) = 40 we get 40 = 𝑇 (0) = 70 + 𝑘. Thus 𝑘 = −30. The
constant 𝑟 is determined from the temperature at a second time: 60 =
𝑇 (2) = 70 − 30𝑒2𝑟 so 𝑟 = 12 ln 13 . Thus 𝑇 (𝑡) = 70 − 30𝑒𝑟𝑡 , with 𝑟 = 12 ln 31 .
1 1 30
The temperature at 𝑡 = 1 is 𝑇 (1) = 70 − 30𝑒 2 ln 3 = 70 − √ 3
≈ 52.7∘ .

38. The ambient temperature is 𝑇𝑎 = 70∘ . Equation (12) gives 𝑇 (𝑡) = 70 +


𝑘𝑒𝑟𝑡 for the temperature of the coffee at time 𝑡. We are asked to determine
the initial temperature of the coffee so 𝑇 (0) is unknown. However, we have
the equations
150 = 𝑇 (5) = 70 + 𝑘𝑒5𝑟
142 = 𝑇 (6) = 70 + 𝑘𝑒6𝑟
or
80 = 𝑘𝑒5𝑟
72 = 𝑘𝑒4𝑟 .
Dividing the second equation by the first gives 72 𝑟
80 = 𝑒 so 𝑟 = ln 0.9.
−5𝑟
From the first equation we get 𝑘 = 80𝑒 ≈ 135.5. We now calculate
𝑇 (0) = 70 + 𝑘 ≈ 205.5∘

39. The ambient temperature is 𝑇𝑎 = 40∘ . Equation (12) gives 𝑇 (𝑡) = 40 +


𝑘𝑒𝑟𝑡 for the temperature of the beer at time 𝑡. Since the initial temperature
of the beer is 𝑇 (0) = 80 we get 80 = 𝑇 (0) = 40 + 𝑘. Thus 𝑘 = 40.
The constant 𝑟 is determined from the temperature at a second time:
60 = 𝑇 (1) = 40 + 40𝑒𝑟 so 𝑟 = − ln 2. Thus 𝑇 (𝑡) = 40 + 40𝑒𝑟𝑡 , with
𝑟 = − ln 2. We now solve the equation 50 = 𝑇 (𝑡) = 40 + 40𝑒𝑟𝑡 for 𝑡 and
get 𝑡 = − ln 4
− ln 2 = 2. She should therefore put the beer in the refrigerator at
2 p.m.

40. Let us start time 𝑡 = 0 at 1980. Then 𝑃 (0) = 290. The Malthusian growth
model gives 𝑃 (𝑡) = 290𝑒𝑟𝑡. At 𝑡 = 10 (1990) we have 370 = 290𝑒10𝑟
1 Solutions 17

1
and hence 𝑟 = 10 ln 37
29 . At 𝑡 = 30 (2010) we have 𝑃 (30) = 290𝑒
30𝑟
=
( 37 )3
290 29 ≈ 602.

41. The initial population is 40 = 𝑃 (0). Since the population doubles in 3


hours we have 𝑃 (3) = 80 or 80 = 40𝑒3𝑟 . Hence 𝑟 = ln32 . Now we can
compute the population after 30 hours: 𝑃 (30) = 40𝑒30𝑟 = 40(210 ) =
40, 960.

42. We have 3𝑃 (0) = 𝑃 (5) = 𝑃 (0)𝑒3𝑟 . So 𝑟 = ln53 . Now we solve the equation
2𝑃 (0) = 𝑃 (𝑡) = 𝑃 (0)𝑒𝑟𝑡 for 𝑡. We get 𝑡 = ln𝑟2 = 5lnln32 ≈ 3.15 years.

43. In the logistic growth equation 𝑚 = 800 and 𝑃 (0) = 290. Thus 𝑃 (𝑡) =
800⋅290 800⋅290
290+510𝑒−𝑟𝑡 . To determine 𝑟 we use 𝑃 (10) = 370 to get 370 = 290+510𝑒−10𝑟 .
1
A simple calculation give 𝑟 = 10 ln 1887
1247 . Now the population in 2010 is
800⋅290
𝑃 (30) = 1247 3
≈ 530
290+510( 1887 )

44. In the logistics equation 𝑚 = 5000 and 𝑃0 = 2000. Thus 𝑃 (𝑡) =


10,000,000 10,000 10,000
2,000+3,000𝑒−𝑟𝑡 = 2+3𝑒−𝑟𝑡 . Since 𝑃 (2) = 3000 we get 3000 = 2+3𝑒−𝑟𝑡 . Solv-
10,000 10,000
ing this equation for 𝑟 gives 𝑟 = ln 32 . Now 𝑃 (4) = 2+3𝑒 −4𝑟 = 4 ≈
2+3( 23 )
3857

45. Let 𝑥 = 𝑒−𝑟𝑡0 . Then 𝑥2 = 𝑒−2𝑟𝑡 . The equation 𝑃 (𝑡0 ) = 𝑃1 implies that
𝑃0 (𝑚−𝑃1 )
𝑥= 𝑃 1 (𝑚−𝑃0 )
. The equation 𝑃 (2𝑡0 ) = 𝑃2 implies 𝑥2 = 𝑃 0 (𝑚−𝑃2 )
𝑃2 (𝑚−𝑃0 ) . These
𝑃02 (𝑚−𝑃1 )2
equation together imply 𝑃12 (𝑚−𝑃0 )2
= 𝑃 0 (𝑚−𝑃2 )
𝑃2 (𝑚−𝑃0 ) . Cross multiplying and
simplifying leads to (𝑃0 𝑃2 −𝑃12 )𝑚+(𝑃12 𝑃0 +𝑃12 𝑃2 −2𝑃0 𝑃1 𝑃2 ) = 0. Solving
−𝑟𝑡0
for 𝑚 gives the result. Now replace the formula for 𝑚 into 𝑒 =𝑥=
𝑃0 (𝑚−𝑃1 ) −𝑟𝑡0 𝑃0 𝑃2 −𝑃1
𝑃1 (𝑚−𝑃0 ) . Simplifying gives 𝑒 = 𝑃2 𝑃1 −𝑃0 . The formula for 𝑟 follows
after taking the natural log of both sides.

46. We have 𝑃 (0) = 𝑃0 = 400, 𝑃 (3) = 𝑃1 = 700, and 𝑃 (6) = 𝑃2 = 1000. Us-
ing the result of the previous problem we get 𝑚 = 700(700(400+1000)−2⋅400⋅1000)
(700)2 −400⋅1000 =
1, 400

Section 1.4
1. This equation is already in standard form with 𝑝(𝑡) = 3. An antiderivative
of 𝑝(𝑡) is 𝑃 (𝑡) = 3 𝑑𝑡 = 3𝑡 so the integrating factor is 𝜇(𝑡) = 𝑒3𝑡 . If we

multiply the differential equation 𝑦 ′ + 3𝑦 = 𝑒𝑡 by 𝜇(𝑡), we get the equation

𝑒3𝑡 𝑦 ′ + 3𝑒3𝑡 𝑦 = 𝑒4𝑡 ,

and the left hand side of this equation is a perfect derivative, namely,
(𝑒3𝑡 𝑦)′ . Thus, (𝑒3𝑡 𝑦)′ = 𝑒4𝑡 . Now take antiderivatives of both sides and
multiply by 𝑒−3𝑡 . This gives
18 1 Solutions

1 𝑡
𝑦= 𝑒 + 𝑐𝑒−3𝑡
4
for the general solution of the equation. To find the constant 𝑐 to satisfy
the initial condition 𝑦(0) = −2, substitute 𝑡 = 0 into the general solution
to get −2 = 𝑦(0) = 41 + 𝑐. Hence 𝑐 = − 49 , and the solution of the initial
value problem is
1 9
𝑦 = 𝑒𝑡 − 𝑒−3𝑡 .
4 4

2. Divide by cos 𝑡 to put the equation in the standard form

𝑦 ′ + (tan 𝑡)𝑦 = sec 𝑡.

In this case 𝑝(𝑡) = tan 𝑡, an antiderivative is 𝑃 (𝑡) = ln(sec 𝑡), and the
integrating factor is 𝜇(𝑡) = sec 𝑡. (We do not need ∣ sec 𝑡∣ since we are
working near 𝑡 = 0 where sec 𝑡 > 0.) Now multiply by the integrating
factor to get (sec 𝑡)𝑦 ′ + (sec 𝑡 tan 𝑡)𝑦 = sec2 𝑡, the left hand side of which
is a perfect derivative. Thus ((sec 𝑡)𝑦)′ = sec2 𝑡 and taking antiderivatives
of both sides gives (sec 𝑡)𝑦 = tan 𝑡 + 𝑐 where 𝑐 ∈ ℝ is a constant. Now
multiply by 1/ sec 𝑡 = cos 𝑡 to get 𝑦 = sin 𝑡+𝑐 cos 𝑡 for the general solution.
Letting 𝑡 = 0 gives 5 = 𝑦(0) = sin 0 + 𝑐 cos 0 = 𝑐 so 𝑐 = 5 and

𝑦 = sin 𝑡 + 5 cos 𝑡.

3. This equation is already in standard form. In this case 𝑝(𝑡) = −2, an


antiderivative is 𝑃 (𝑡) = −2𝑡, and the integrating factor is 𝜇(𝑡) = 𝑒−2𝑡 .
Now multiply by the integrating factor to get

𝑒−2𝑡 𝑦 ′ − 2𝑒−2𝑡 𝑦 = 1,

the left hand side of which is a perfect derivative ((𝑒−2𝑡 )𝑦)′ . Thus ((𝑒−2𝑡 )𝑦)′ =
1 and taking antiderivatives of both sides gives

(𝑒−2𝑡 )𝑦 = 𝑡 + 𝑐,
where 𝑐 ∈ ℝ is a constant. Now multiply by 𝑒2𝑡 to get 𝑦 = 𝑡𝑒2𝑡 + 𝑐𝑒2𝑡 for
the general solution. Letting 𝑡 = 0 gives 4 = 𝑦(0) = 𝑐 so

𝑦 = 𝑡𝑒2𝑡 + 4𝑒2𝑡 .

4. Divide by 𝑡 to put the equation in the standard form


1 𝑒𝑡
𝑦′ + 𝑦 =
𝑡 𝑡
In this case 𝑝(𝑡) = 1/𝑡, an antiderivative is 𝑃 (𝑡) = ln 𝑡, and the integrating
factor is 𝜇(𝑡) = 𝑡. Now multiply the standard form equation by the inte-
grating factor to get 𝑡𝑦 ′ + 𝑦 = 𝑒𝑡 , the left hand side of which is a perfect
1 Solutions 19

derivative (𝑡𝑦)′ . (Note that this is just the original left hand side of the
equation. Thus if we had recognized that the left hand side was already a
perfect derivative, the preliminary steps could have been skipped for this
problem, and we could have proceeded directly to the next step.) Thus
the equation can be written as (𝑡𝑦)′ = 𝑒𝑡 and taking antiderivatives of
both sides gives 𝑡𝑦 = 𝑒𝑡 + 𝑐 where 𝑐 ∈ ℝ is a constant. Now divide by 𝑡 to
get
𝑒𝑡 𝑐
𝑦= +
𝑡 𝑡
for the general solution.
𝑒𝑡
5. The general solution from Problem 4 is 𝑦 = 𝑡 + 𝑐𝑡 . Now let 𝑡 = 1 to get
𝑡
0 = 𝑒 + 𝑐. So 𝑐 = −𝑒 and 𝑦 = 𝑒𝑡 − 𝑒𝑡 .

6. Divide by 𝑡 to put the equation in the standard form


𝑚
𝑦′ + 𝑦 = ln 𝑡.
𝑡
In this case 𝑝(𝑡) = 𝑚 𝑚
𝑡 , an antiderivative is 𝑃 (𝑡) = 𝑚 ln 𝑡 = ln 𝑡 , and the
𝑚
integrating factor is 𝜇(𝑡) = 𝑡 . Now multiply the standard form equation
by the integrating factor to get 𝑡𝑚 𝑦 ′ + 𝑚𝑡𝑚−1 𝑦 = 𝑡𝑚 ln 𝑡, the left hand
side of which is a perfect derivative ((𝑡𝑚 )𝑦)′ . Thus ((𝑡𝑚 )𝑦)′ = 𝑡𝑚 ln 𝑡. To
integrate 𝑡𝑚 ln 𝑡 we consider the cases 𝑚 = −1 and 𝑚 ∕= −1 separately.
2
Case 𝑚 = −1: A simple substitution gives 𝑡−1 ln 𝑡 𝑑𝑡 = (ln2𝑡) + 𝑐.

(ln 𝑡)2 𝑡(ln 𝑡)2
Hence, 𝑡−1 𝑦 = 2 + 𝑐 and so 𝑦 = 2 + 𝑐𝑡
𝑡𝑚+1 ln 𝑡
𝑡𝑚 ln 𝑡 𝑑𝑡 =

Case 𝑚 ∕= −1: Use integration by parts to get 𝑚+1 −
𝑡𝑚+1 𝑡 ln 𝑡 𝑡 𝑐
(𝑚+1)2 + 𝑐. Then 𝑦 = 𝑚+1 − (𝑚+1)2 + 𝑡𝑚 .

7. We first put the equation in standard form and get


1
𝑦 ′ + 𝑦 = cos(𝑡2 ).
𝑡
In this case 𝑝(𝑡) = 1𝑡 , an antiderivative is 𝑃 (𝑡) = ln 𝑡, and the integrating
factor is 𝜇(𝑡) = 𝑡. Now multiply by the integrating factor to get

𝑡𝑦 ′ + 𝑦 = 𝑡 cos(𝑡2 ),

the left hand side of which is a perfect derivative (𝑡𝑦)′ . Thus (𝑡𝑦)′ =
𝑡 cos(𝑡2 ) and taking antiderivatives of both sides gives 𝑡𝑦 = 21 sin(𝑡2 ) + 𝑐
sin(𝑡2 )
where 𝑐 ∈ ℝ is a constant. Now divide by 𝑡 to get 𝑦 = 2𝑡 + 𝑐𝑡 . for the
general solution.
∫ new: please check
8. In this case 𝑝(𝑡) = 2 and the integrating factor is 𝑒 2 𝑑𝑡 = 𝑒2𝑡 . Now
multiply to get 𝑒2𝑡 𝑦 ′ + 2𝑒2𝑡 𝑦 = 𝑒2𝑡 sin 𝑡, which simplifies to (𝑒2𝑡 𝑦)′ =
20 1 Solutions

1
𝑒2𝑡 sin 𝑡. Now integrate both sides to get 𝑒2𝑡 𝑦 = (− cos 𝑡 + 2 sin 𝑡)𝑒2𝑡 + 𝑐,
5
where we computed 𝑒2𝑡 sin 𝑡 by parts two times. Dividing by 𝑒2𝑡 gives

1
𝑦 = (2 sin 𝑡 − cos 𝑡) + 𝑐𝑒−2𝑡 .
5

9. In this case 𝑝(𝑡) = −3 and the integrating factor is 𝑒 −3 𝑑𝑡 = 𝑒−3𝑡 .
Now multiply to get 𝑒−3𝑡 𝑦 ′ + 2𝑒−3𝑡 𝑦 = 25𝑒−3𝑡 cos 4𝑡, which simplifies
to (𝑒−3𝑡 𝑦)′ = 25𝑒−3𝑡 cos 4𝑡. Now integrate both∫ sides to get 𝑒−3𝑡 𝑦 =
(4 sin 4𝑡 − 3 cos 4𝑡)𝑒−3𝑡 + 𝑐, where we computed 25𝑒−3𝑡 cos 4𝑡 by parts
twice. Dividing by 𝑒−3𝑡 gives 𝑦 = 4 sin 4𝑡 − 3 cos 4𝑡 + 𝑐𝑒3𝑡 .

10. In standard form this equation becomes


1 2
𝑦′ − 𝑦= .
𝑡(𝑡 + 1) 𝑡(𝑡 + 1)
−1 1
Using partial fractions we get 𝑝(𝑡) = 𝑡(𝑡+1) = 𝑡+1 − 1𝑡 , an antiderivative is
𝑃 (𝑡) = ln(𝑡 + 1) − ln 𝑡 = ln 𝑡 , and the integrating factor is 𝜇(𝑡) = 𝑡+1
( 𝑡+1 )
𝑡 .
Now multiply by the integrating factor to get
𝑡+1 ′ 1 2
𝑦 − 2𝑦 = 2,
𝑡 𝑡 𝑡
the left hand side of which is a perfect derivative ( 𝑡+1 ′
𝑡 𝑦) . Thus

𝑡+1 ′ 2
( 𝑦) = 2
𝑡 𝑡
and taking antiderivatives of both sides gives 𝑡+1 −2
𝑡 𝑦 = 𝑡 + 𝑐 where 𝑐 ∈ ℝ
𝑡 −2 𝑐𝑡
is a constant. Now multiply by 𝑡+1 to get 𝑦 = 𝑡+1 + 𝑡+1 = 𝑐𝑡−2
𝑡+1 for the
general solution.

11. In
∫ standard form we get 𝑧 ′ − 2𝑡𝑧 = −2𝑡3 . An integrating factor is
−2𝑡 𝑑𝑡 −𝑡2 2 2
𝑒 = 𝑒 . Thus (𝑒−𝑡 𝑧)′ = −2𝑡3 𝑒−𝑡 . Integrating both sides gives
2 2
𝑒−𝑡 𝑧 = (𝑡2 + 1)𝑒−𝑡 + 𝑐, where the integral of the right hand side is done
2 2
by parts. Now divide by the integrating factor 𝑒−𝑡 to get 𝑧 = 𝑡2 +1+𝑐𝑒𝑡 .

12. The given differential equation is in standard form, 𝑝(𝑡) = 𝑎, an an-


tiderivative is 𝑃 (𝑡) = 𝑎𝑡, and the integrating factor is 𝜇(𝑡) = 𝑒𝑎𝑡 . Now
multiply by the integrating factor to get
𝑒𝑎𝑡 𝑦 ′ + 𝑎𝑒𝑎𝑡 𝑦 = 𝑏𝑒𝑎𝑡 ,
the left hand side of which is a perfect derivative ((𝑒𝑎𝑡 )𝑦)′ . Thus
((𝑒𝑎𝑡 )𝑦)′ = 𝑏𝑒𝑎𝑡 .
If 𝑎 ∕= 0 then taking antiderivatives of both sides gives 𝑒𝑎𝑡 𝑦 = 𝑎𝑏 𝑒𝑎𝑡 + 𝑐
where 𝑐 ∈ ℝ is a constant. Now multiply by 𝑒−𝑎𝑡 to get 𝑦 = 𝑎𝑏 + 𝑐𝑒−𝑎𝑡 for
the general solution. In the case 𝑎 = 0 then 𝑦 ′ = 𝑏 and 𝑦 = 𝑏𝑡 + 𝑐.
1 Solutions 21

13. The given equation is in standard form, 𝑝(𝑡) = cos 𝑡, an antiderivative is


𝑃 (𝑡) = − sin 𝑡, and the integrating factor is 𝜇(𝑡) = 𝑒− sin 𝑡 . Now multiply
by the integrating factor to get

𝑒− sin 𝑡 𝑦 ′ + (cos 𝑡)𝑒− sin 𝑡 𝑦 = (cos 𝑡)𝑒− sin 𝑡 ,

the left hand side of which is a perfect derivative ((𝑒− sin 𝑡 )𝑦)′ . Thus

((𝑒− sin 𝑡 )𝑦)′ = (cos 𝑡)𝑒− sin 𝑡

and taking antiderivatives of both sides gives (𝑒− sin 𝑡 )𝑦 = 𝑒− sin 𝑡 +𝑐 where
𝑐 ∈ ℝ is a constant. Now multiply by 𝑒sin 𝑡 to get 𝑦 = 1 + 𝑐𝑒sin 𝑡 for the
general solution. To satisfy the initial condition, 0 = 𝑦(0) = 1 + 𝑐𝑒sin 0 =
1 + 𝑐, so 𝑐 = −1. Thus, the solution of the initial value problem is 𝑦 =
1 − 𝑒sin 𝑡
−2
14. The given equation is in standard form, 𝑝(𝑡) = 𝑡+1 , an antiderivative is
−2
𝑃 (𝑡) = −2 ln(𝑡 + 1) = ln((𝑡 + 1) ), and the integrating factor is 𝜇(𝑡) =
(𝑡 + 1)−2 . Now multiply by the integrating factor to get
2
(𝑡 + 1)−2 𝑦 ′ − 𝑦 = 1,
(𝑡 + 1)3

the left hand side of which is a perfect derivative (((𝑡 + 1)−2 )𝑦)′ . Thus

(((𝑡 + 1)−2 )𝑦)′ = 1

and taking antiderivatives of both sides gives ((𝑡 + 1)−2 )𝑦 = 𝑡 + 𝑐 where


𝑐 ∈ ℝ is a constant. Now multiply by (𝑡 + 1)2 to get 𝑦 = (𝑡 + 𝑐)(𝑡 + 1)2 for
the general solution.

15. The given linear differential equation is in standard form, 𝑝(𝑡) = −2 𝑡 ,


an antiderivative is 𝑃 (𝑡) = −2 ln 𝑡 = ln 𝑡−2 , and the integrating factor is
𝜇(𝑡) = 𝑡−2 . Now multiply by the integrating factor to get
2 𝑡+1
𝑡−2 𝑦 ′ − 𝑦 = 3 = 𝑡−2 + 𝑡−3 ,
𝑡3 𝑡
the left hand side of which is a perfect derivative (𝑡−2 𝑦)′ . Thus

(𝑡−2 𝑦)′ = 𝑡−2 + 𝑡−3


−2
and taking antiderivatives of both sides gives (𝑡−2 )𝑦 = −𝑡−1 − 𝑡 2 +𝑐 where
𝑐 ∈ ℝ is a constant. Now multiply by 𝑡2 to and we get 𝑦 = −𝑡 − 12 + 𝑐𝑡−2
for the general solution. Letting 𝑡 = 1 gives −3 = 𝑦(1) = −3 −3
2 + 𝑐 so 𝑐 = 2
and
1 3
𝑦(𝑡) = −𝑡 − − 𝑡−2 .
2 2
22 1 Solutions

16. The given equation is in standard form, 𝑝(𝑡) = 𝑎, an antiderivative is


𝑃 (𝑡) = 𝑎𝑡, and the integrating factor is 𝜇(𝑡) = 𝑒𝑎𝑡 . Now multiply by the
integrating factor to get 𝑒𝑎𝑡 𝑦 ′ + 𝑎𝑒𝑎𝑡 𝑦 = 1, the left hand side of which is a
perfect derivative (𝑒𝑎𝑡 𝑦)′ . Thus (𝑒𝑎𝑡 𝑦)′ = 1 and taking antiderivatives of
both sides gives 𝑒𝑎𝑡 𝑦 = 𝑡 + 𝑐 where 𝑐 ∈ ℝ is a constant. Now multiply by
𝑒−𝑎𝑡 to get 𝑦 = 𝑡𝑒−𝑎𝑡 + 𝑐𝑒−𝑎𝑡 for the general solution.

17. The given equation is in standard form, 𝑝(𝑡) = 𝑎, 𝑝(𝑡) = 𝑎, an antideriva-


tive is 𝑃 (𝑡) = 𝑎𝑡, and the integrating factor is 𝜇(𝑡) = 𝑒𝑎𝑡 . Now multiply
by the integrating factor to get 𝑒𝑎𝑡 𝑦 ′ + 𝑎𝑒𝑎𝑡 𝑦 = 𝑒(𝑎+𝑏)𝑡 , the left hand side
of which is a perfect derivative (𝑒𝑎𝑡 𝑦)′ . Thus (𝑒𝑎𝑡 𝑦)′ = 𝑒(𝑎+𝑏)𝑡 and taking
antiderivatives of both sides gives
1 (𝑎+𝑏)𝑡
(𝑒𝑎𝑡 )𝑦 = 𝑒 +𝑐
𝑎+𝑏
where 𝑐 ∈ ℝ is a constant. Now multiply by 𝑒−𝑎𝑡 to get
1 𝑏𝑡
𝑦= 𝑒 + 𝑐𝑒−𝑎𝑡
𝑎+𝑏
for the general solution.

18. The given differential equation is in standard form, 𝑝(𝑡) = 𝑎, an an-


tiderivative is 𝑃 (𝑡) = 𝑎𝑡, and the integrating factor is 𝜇(𝑡) = 𝑒𝑎𝑡 . Now
multiply by the integrating factor to get 𝑒𝑎𝑡 𝑦 ′ + 𝑎𝑒𝑎𝑡 𝑦 = 𝑡𝑛 , the left
hand side of which is a perfect derivative (𝑒𝑎𝑡 𝑦)′ . Thus (𝑒𝑎𝑡 𝑦)′ = 𝑡𝑛 .
Now assume 𝑛 ∕= −1. Then taking antiderivatives of both sides gives
𝑛+1
(𝑒𝑎𝑡 )𝑦 = 𝑡𝑛+1 + 𝑐 where 𝑐 ∈ ℝ is a constant. Now multiply by 𝑒−𝑎𝑡 to
𝑛+1
get 𝑦 = 𝑡𝑛+1 𝑒−𝑎𝑡 + 𝑐𝑒−𝑎𝑡 for the general solution. If 𝑛 = 1 then taking
antiderivatives leads to 𝑒𝑎𝑡 𝑦 = ln 𝑡 + 𝑐 and hence 𝑦 = (ln 𝑡)𝑒−𝑎𝑡 + 𝑐𝑒−𝑎𝑡 is
the general solution in this case.

19. In standard form we get 𝑦 ′ − (tan 𝑡)𝑦 = sec 𝑡. In this case 𝑝(𝑡) =
− tan 𝑡, an antiderivative is 𝑃 (𝑡) = ln cos 𝑡, and the integrating factor
is 𝜇(𝑡) = 𝑒𝑃 (𝑡) = cos 𝑡. Now multiply by the integrating factor to get
(cos 𝑡)𝑦 ′ − (sin 𝑡)𝑦 = 1, the left hand side of which is a perfect deriva-
tive ((cos 𝑡)𝑦)′ . Thus ((cos 𝑡)𝑦)′ = 1 and taking antiderivatives of both
sides gives (cos 𝑡)𝑦 = 𝑡 + 𝑐 where 𝑐 ∈ ℝ is a constant. Now multiply by
1/ cos 𝑡 = sec 𝑡 and we get 𝑦 = (𝑡 + 𝑐) sec 𝑡 for the general solution.

20. Divide by 𝑡 to put the equation in the standard form


2 ln 𝑡 4 ln 𝑡
𝑦′ + 𝑦= .
𝑡 𝑡
In this case 𝑝(𝑡) = 2 ln 𝑡 2
𝑡 , an antiderivative is 𝑃 (𝑡) = (ln 𝑡) , and the inte-
(ln 𝑡)2
grating factor is 𝜇(𝑡) = 𝑒 . Now multiply by the integrating factor to
get
1 Solutions 23

2 2 ln 𝑡 (ln 𝑡)2 4 ln 𝑡 (ln 𝑡)2


𝑒(ln 𝑡) 𝑦 ′ + 𝑒 𝑦= 𝑒 ,
𝑡 𝑡
2
the left hand side of which is a perfect derivative (𝑒(ln 𝑡) 𝑦)′ . Thus
2
𝑡 (ln 𝑡)2
(𝑒(ln 𝑡) 𝑦)′ = 4 ln
𝑡 𝑒 and taking antiderivatives of both sides gives
2 2
𝑒(ln 𝑡) 𝑦 = 2𝑒(ln 𝑡) + 𝑐
2
where 𝑐 ∈ ℝ is a constant. Now multiply by 𝑒−(ln 𝑡) and we get 𝑦 =
2
2 + 𝑐𝑒−(ln 𝑡) for the general solution.

21. The given differential equation is in standard form, 𝑝(𝑡) = −𝑛/𝑡, an


antiderivative is 𝑃 (𝑡) = −𝑛 ln 𝑡 = ln(𝑡−𝑛 ), and the integrating factor
is 𝜇(𝑡) = 𝑡−𝑛 . Now multiply by the integrating factor to get 𝑡−𝑛 𝑦 ′ −
𝑛𝑡−𝑛−1 𝑦 = 𝑒𝑡 , the left hand side of which is a perfect derivative (𝑡−𝑛 𝑦)′ .
Thus (𝑡−𝑛 𝑦)′ = 𝑒𝑡 and taking antiderivatives of both sides gives (𝑡−𝑛 )𝑦 =
𝑒𝑡 + 𝑐 where 𝑐 ∈ ℝ is a constant. Now multiply by 𝑡𝑛 to and we get
𝑦 = 𝑡𝑛 𝑒𝑡 + 𝑐𝑡𝑛 for the general solution.

22. The given differential equation is in standard form, 𝑝(𝑡) = −1, an an-
tiderivative is 𝑃 (𝑡) = −𝑡, and the integrating factor is 𝜇(𝑡) = 𝑒−𝑡 . Now
multiply by the integrating factor to get 𝑒−𝑡 𝑦 ′ − 𝑒−𝑡 𝑦 = 𝑡𝑒𝑡 , the left
hand side of which is a perfect derivative (𝑒−𝑡 𝑦)′ . Thus (𝑒−𝑡 𝑦)′ = 𝑡𝑒𝑡 .
Taking antiderivatives of both sides and using integration by parts gives
𝑒−𝑡 𝑦 = 𝑡𝑒𝑡 − 𝑒𝑡 + 𝑐 = (𝑡 − 1)𝑒𝑡 + 𝑐 where 𝑐 ∈ ℝ is a constant. Now multiply
by 𝑒𝑡 to get 𝑦 = (𝑡 − 1)𝑒2𝑡 + 𝑐𝑒𝑡 for the general solution. Letting 𝑡 = 0
gives 𝑎 = 𝑦(0) = −1 + 𝑐 so 𝑐 = 𝑎 + 1 and

𝑦 = (𝑡 − 1)𝑒2𝑡 + (𝑎 + 1)𝑒𝑡 .

23. Divide by 𝑡 to put the equation in the standard form


3
𝑦 ′ + 𝑦 = 𝑡.
𝑡
In this case 𝑝(𝑡) = 3/𝑡, an antiderivative is 𝑃 (𝑡) = 3 ln 𝑡 = ln(𝑡3 ), and the
integrating factor is 𝜇(𝑡) = 𝑡3 . Now multiply the standard form equation
by the integrating factor to get 𝑡3 𝑦 ′ +3𝑡2 𝑦 = 𝑡4 , the left hand side of which
is a perfect derivative (𝑡3 𝑦)′ . Thus (𝑡3 𝑦)′ = 𝑡4 and taking antiderivatives
of both sides gives 𝑡3 𝑦 = 51 𝑡5 + 𝑐 where 𝑐 ∈ ℝ is a constant. Now multiply
by 𝑡−3 and we get 𝑦 = 51 𝑡2 + 𝑐𝑡−3 for the general solution. Letting 𝑡 = −1
gives 2 = 𝑦(−1) = 51 − 𝑐 so 𝑐 = −9 5 and

1 2 9 −3
𝑦= 𝑡 − 𝑡 .
5 5
24 1 Solutions

24. In this case the given differential equation is in standard form. We have
𝑝(𝑡) = 2𝑡, an antiderivative is 𝑃 (𝑡) = 𝑡2 , and the integrating factor is
2 2 2
𝜇(𝑡) = 𝑒𝑡 . Now multiply by the integrating factor to get 𝑒𝑡 𝑦 ′ + 2𝑡𝑒𝑡 𝑦 =
2 2 2
𝑒𝑡 , the left hand side of which is a perfect derivative (𝑒𝑡 𝑦)′ . Thus (𝑒𝑡 𝑦)′ =
2 2 ∫ 2
𝑒𝑡 and taking antiderivatives of both sides gives 𝑒𝑡 𝑦 = 𝑒𝑡 𝑑𝑡 + 𝑐 where
𝑐 ∈ ℝ is a constant. However, the right hand side has no closed form
antiderivative. Using Corollary 8 to can write
∫ 𝑡 ∫ 𝑡
2 2 2 2 2 2
𝑦 = 𝑒−𝑡 𝑒𝑢 𝑑𝑢 + 𝑦(0)𝑒−𝑡 = 𝑒−𝑡 𝑒𝑢 𝑑𝑢 + 𝑒−𝑡 .
0 0

25. Divide by 𝑡2 to put the equation in the standard form


2
𝑦 ′ + 𝑦 = 𝑡−2 .
𝑡
In this case 𝑝(𝑡) = 2/𝑡, an antiderivative is 𝑃 (𝑡) = 2 ln 𝑡 = ln 𝑡2 , and the
integrating factor is 𝜇(𝑡) = 𝑡2 . Now multiply by the integrating factor to
get 𝑡2 𝑦 ′ + 2𝑡𝑦 = 1, the left hand side of which is a perfect derivative (𝑡2 𝑦)′ .
Thus (𝑡2 𝑦)′ = 1 and taking antiderivatives of both sides gives 𝑡2 𝑦 = 𝑡 + 𝑐
where 𝑐 ∈ ℝ is a constant. Now multiply by 𝑡−2 to get 𝑦 = 1𝑡 + 𝑐𝑡−2 for
the general solution. Letting 𝑡 = 2 gives 𝑎 = 𝑦(2) = 21 + 4𝑐 so 𝑐 = 4𝑎 − 2
and
1
𝑦 = + (4𝑎 − 2)𝑡−2 .
𝑡

gal lbs lbs


26. input rate: input rate = 4 ×0 =0 .
min gal min
gal 𝑦(𝑡) lbs 4 lbs
output rate: output rate = 4 × = 𝑦(𝑡) .
min 100 gal 100 min
Since
𝑦 ′ = input rate − output rate
we get the initial value problem
4
𝑦′ = 0 − 𝑦, 𝑦(0) = 80.
100
1
Simplifying and putting in standard form gives 𝑦 ′ + 𝑦 = 0. The coeffi-
∫ 25
cient function is 𝑝(𝑡) = 1/25, 𝑃 (𝑡) = 𝑝(𝑡) 𝑑𝑡 = 𝑡/25, and the integrating
factor is 𝜇(𝑡) = 𝑒𝑡/25 . Thus (𝑒𝑡/25 𝑦)′ = 0. Integrating and simplifying gives
𝑦 = 𝑐𝑒−𝑡/25 . The initial condition implies 𝑐 = 80 so 𝑦 = 80𝑒−𝑡/25 . The
concentration of the brine solution is now obtained by dividing by the
80 −𝑡/25
volume which is 100 gallons: 100 𝑒 = 0.8𝑒−𝑡/25 .
1 Solutions 25

27. Let 𝑉 (𝑡) denote the volume of fluid in the tank at time 𝑡. Initially, there
are 10 gal of brine. For each minute that passes there is a net decrease of
4 − 3 = 1 gal of brine. Thus 𝑉 (𝑡) = 10 − 𝑡 gal.
gal lbs lbs
input rate: input rate = 3 ×1 =3 .
min gal min
gal 𝑦(𝑡) lbs 4𝑦(𝑡) lbs
output rate: output rate = 4 × = .
min 𝑉 (𝑡) gal 10 − 𝑡 min
Since 𝑦 ′ = input rate − output rate, it follows that 𝑦(𝑡) satisfies the initial
value problem
4
𝑦′ = 3 − 𝑦(𝑡) , 𝑦(0) = 2.
10 − 𝑡
Put in standard form, this equation becomes
4
𝑦′ + 𝑦 = 3.
10 − 𝑡
4

The coefficient function is 𝑝(𝑡) = 10−𝑡 , 𝑃 (𝑡) = 𝑝(𝑡) 𝑑𝑡 = −4 ln(10 − 𝑡) =
ln(10 − 𝑡)−4 , and the integrating factor is 𝜇(𝑡) = (10 − 𝑡)−4 . Multiplying
the standard form equation by the integrating factor gives

((10 − 𝑡)−4 𝑦)′ = 3(10 − 𝑡)−4 .

Integrating and simplifying gives 𝑦 = (10 − 𝑡) + 𝑐(10 − 𝑡)4 . The initial


condition 𝑦(0) = 2 implies 2 = 𝑦(0) = 10 + 𝑐104 and hence 𝑐 = −8/104 so
8
𝑦 = (10 − 𝑡) − (10 − 𝑡)4 .
104
Of course, this formula is valid for 0 ≤ 𝑡 ≤ 10. After 10 minutes there is
no fluid and hence no salt in the tank.
L g g
28. input rate: input rate = 1 × 10 = 10 .
min L min
L 𝑦(𝑡) g 𝑦(𝑡) g
output rate: output rate = 1 × = .

min 10 L 10 min
Since 𝑦 = input rate − output rate, it follows that 𝑦(𝑡) satisfies the initial
value problem
1
𝑦 ′ = 10 − 𝑦 , 𝑦(0) = 0.
10
1
Put in standard form, this equation becomes 𝑦 ′ + 10 𝑦 = 10. The coefficient
1

function is 𝑝(𝑡) = 10 , 𝑃 (𝑡) = 𝑝(𝑡) 𝑑𝑡 = 𝑡/10, and the integrating factor
is 𝜇(𝑡) = 𝑒𝑡/10 . Multiplying the standard form equation by the integrating
factor gives (𝑒𝑡/10 𝑦)′ = 10𝑒𝑡/10 . Integrating and simplifying gives 𝑦 =
100 + 𝑐𝑒−𝑡/10 . The initial condition 𝑦(0) = 0 implies 𝑐 = −100 so 𝑦 =
100 − 100𝑒−𝑡/10. After 10 minutes we have 𝑦(10) = 100 − 100𝑒−1 g of salt.
The concentration is thus (100 − 100𝑒−1)/10 = 10 − 10𝑒−1 g/L
26 1 Solutions

29. Let 𝑉 (𝑡) denote the volume of fluid in the container at time 𝑡. Initially,
there are 10 L. For each minute that passes there is a net gain of 4−2 = 2 L
of fluid. So 𝑉 (𝑡) = 10 + 2𝑡. The container overflows when 𝑉 (𝑡) = 10 + 2𝑡 =
30 or 𝑡 = 10 minutes.
L g g
input rate: input rate = 4 × 20 = 80 .
min L min
L 𝑦(𝑡) g 2𝑦(𝑡) g
output rate: output rate = 2 × = .
min 10 + 𝑡 L 10 + 𝑡 min
Since 𝑦 ′ = input rate − output rate, it follows that 𝑦(𝑡) satisfies the initial
value problem
2𝑦
𝑦 ′ = 80 − , 𝑦(0) = 0.
10 + 𝑡
Simplifying and putting in standard form gives the equation
1
𝑦′ + 𝑦 = 80.
5+𝑡
1

The coefficient function is 𝑝(𝑡) = 5+𝑡 , 𝑃 (𝑡) = 𝑝(𝑡) 𝑑𝑡 = ln(5 + 𝑡), and the
integrating factor is 𝜇(𝑡) = 5 + 𝑡. Multiplying the standard form equation
by the integrating factor gives ((5 + 𝑡)𝑦)′ = 80(5 + 𝑡). Integrating and
simplifying gives 𝑦 = 40(5+𝑡)+𝑐(5+𝑡)−1, where 𝑐 is a constant. The initial
condition 𝑦(0) = 0 implies 𝑐 = −1000 so 𝑦 = 40(5 + 𝑡) − 1000(5 + 𝑡) −1 .
At the time the container overflows 𝑡 = 10 we have 𝑦(10) = 600 − 1000 15 ≈
533.33 g of salt.

30. Let 𝑉 (𝑡) denote the volume of fluid in the tank at time 𝑡. Initially, there
are 10 gallons of fluid. For each minute that goes by there is a net increase
of 4 − 2 = 2 gallons. It follows that 𝑉 (𝑡) = 10 + 2𝑡. The tank will overflow
when 100 = 𝑉 (𝑡). Solving 100 = 10 + 2𝑡 gives 𝑡 = 45. Thus 𝑇 = 45
minutes. Next we find 𝑦(𝑡):
gal lbs lbs
input rate: input rate = 4 × 0.5 =2 .
min gal min
gal 𝑦(𝑡) lbs 2𝑦(𝑡) lbs
output rate: output rate = 2 × = .
min 10 + 2𝑡 gal 10 + 2𝑡 min
Since 𝑦 ′ = input rate − output rate, it follows that 𝑦(𝑡) satisfies the initial
value problem
𝑦
𝑦′ = 2 − , 𝑦(0) = 0.
5+𝑡
Putting this equation in standard form gives
1
𝑦′ + 𝑦 = 2.
5+𝑡
1

The coefficient function is 𝑝(𝑡) = 5+𝑡 , 𝑃 (𝑡) = 𝑝(𝑡) 𝑑𝑡 = ln(5 + 𝑡), and the
integrating factor is 𝜇(𝑡) = 5 + 𝑡. Thus ((5 + 𝑡)𝑦)′ = 2(5 + 𝑡). Integrating
1 Solutions 27

and simplifying gives 𝑦 = (5+𝑡)+𝑐(5+𝑡)−1 . The initial condition 𝑦(0) = 0


implies 𝑐 = −25 so 𝑦 = (5 + 𝑡) − 25(5 + 𝑡)−1, for 0 ≤ 𝑡 ≤ 45. At 𝑡 = 𝑇 = 45
25
we get 𝑦(45) = 50 − 50 = 49.5 lbs salt. Once the tank is full, the inflow
and outflow rates will be equal and the brine in the tank will (in the
limit as 𝑡 → ∞) stabilize to the concentration of the incoming brine, i.e.,
0.5 lb/gal. Since the tank holds 100 gal, the total amount present will
approach 0.5 × 100 = 50 lbs. Thus lim𝑡→∞ 𝑦(𝑡) = 50.

31. input rate: input rate = 𝑟𝑐


output rate: output rate = 𝑟 𝑃𝑉(𝑡)
Let 𝑃0 denote the amount of pollutant at time 𝑡 = 0. Since 𝑃 ′ =
input rate − output rate it follows that 𝑃 (𝑡) is a solution of the initial
value problem
𝑟𝑃 (𝑡)
𝑃 ′ = 𝑟𝑐 − , 𝑃 (0) = 𝑃0 .
𝑉
Rewriting this equation in standard form gives the differential equation
𝑃 ′ + 𝑉𝑟 𝑃 = 𝑟𝑐. The coefficient function is 𝑝(𝑡) = 𝑟/𝑉 and the integrating
𝑟𝑡 𝑟𝑡
factor is 𝜇(𝑡) = 𝑒𝑟𝑡/𝑉 . Thus (𝑒 𝑉 𝑃 )′ = 𝑟𝑐𝑒 𝑉 . Integrating and simplifying
−𝑟𝑡
gives 𝑃 (𝑡) = 𝑐𝑉 +𝑘𝑒 𝑉 , where 𝑘 is the constant of integration. The initial
−𝑟𝑡
condition 𝑃 (0) = 𝑃0 implies 𝑐 = 𝑃0 − 𝑐𝑉 so 𝑃 (𝑡) = 𝑐𝑉 + (𝑃0 − 𝑐𝑉 )𝑒 𝑉 .
(a) lim𝑡→∞ 𝑃 (𝑡) = 𝑐𝑉.
(b) When the river is cleaned up at 𝑡 = 0 we assume the input con-
centration is 𝑐 = 0. The amount of pollutant is therefore given by
−𝑟𝑡
𝑃 (𝑡) = 𝑃0 𝑒 𝑉 . This will reduce by 1/2 when 𝑃 (𝑡) = 12 𝑃0 . We solve the
−𝑟𝑡
equation 21 𝑃0 = 𝑃0 𝑒 𝑉 for 𝑡 and get 𝑡1/2 = 𝑉 ln𝑟2 . Similarly, the pollutant
will reduce by 1/10 when 𝑡1/10 = 𝑉 ln𝑟10 .
(c) Letting 𝑉 and 𝑟 be given as stated for each lake gives:
Lake Erie: 𝑡1/2 = 1.82 years, 𝑡1/10 = 6.05 years.
Lake Ontario: 𝑡1/2 = 5.43 years, 𝑡1/10 = 18.06 years

32. Let 𝑦1 (𝑡) and 𝑦2 (𝑡) denote the amount of salt in Tank 1 and Tank 2,
respectively, at time 𝑡.
L g g
input rate for Tank 1: input rate = 4 × 100 = 400 .
min L min
L 𝑦1 (𝑡) g 4𝑦1 (𝑡) g
output rate for Tank 1: output rate = 4 × = .
min 10 L 10 m
The initial value problem for Tank 1 is thus:
4
𝑦1′ = 400 − 𝑦1 , 𝑦1 (0) = 0.
10
4
Simplifying and putting this equation in standard form gives 𝑦1′ + 10 𝑦1 =
4𝑡/10 4𝑡/10
400. The integrating factor is 𝜇(𝑡) = 𝑒 . Thus (𝑒 𝑦1 ) = 400𝑒4𝑡/10 .

28 1 Solutions

Integrating and simplifying gives 𝑦1 = 1000 + 𝑐𝑒−4𝑡/10 . The initial condi-


tion 𝑦1 (0) = 0 implies 𝑐 = −1000 so 𝑦1 = 1000 − 1000𝑒−4𝑡/10. Now the
brine solution in Tank 1 has concentration 𝑦1 (𝑡)/10 = 100 − 100𝑒−4𝑡/10
and flows into Tank 2 at a rate of 4 liters per minute. Thus
L g
input rate for Tank 2: input rate = 4 × (100 − 100𝑒−4𝑡/10 ) =
min L
g
400 − 400𝑒−4𝑡/10 .
min
L 𝑦2 (𝑡) g 4𝑦2 (𝑡) g
output rate for Tank 2: output rate = 4 × = .
min 10 L 10 min
The initial value problem for Tank 2 is thus:
4
𝑦2′ = 400 − 400𝑒−4𝑡/10 − 𝑦2 , 𝑦2 (0) = 0.
10
Simplifying and putting this equation in standard form gives
4
𝑦2′ + 𝑦2 = 400 − 400𝑒−4𝑡/10.
10
The integrating factor is again (as for the Tank 1 equation) 𝜇(𝑡) = 𝑒4𝑡/10 .
Thus multiplying by the integrating factor gives

(𝑒4𝑡/10 𝑦2 )′ = 400𝑒4𝑡/10 − 400.

Integrating and simplifying gives

𝑦2 (𝑡) = 1000 − 400𝑡𝑒−4𝑡/10 + 𝑐𝑒−4𝑡/10 .

The initial condition 𝑦2 (0) = 0 implies 𝑐 = −1000 so

𝑦2 (𝑡) = 1000 − 400𝑡𝑒−4𝑡/10 − 1000𝑒−4𝑡/10.

33. Let 𝑦1 (𝑡) and 𝑦2 (𝑡) denote the amount of salt in Tank 1 and Tank 2,
respectively, at time 𝑡. The volume of fluid at time 𝑡 in Tank 1 is 𝑉1 (𝑡) =
10 + 2𝑡 and Tank 2 is 𝑉2 (𝑡) = 5 + 𝑡.
L g g
input rate for Tank 1: input rate = 4 × 10 = 40 .
min L min
L 𝑦1 (𝑡) g
output rate for Tank 1: output rate = 2 × =
min 10 + 2𝑡 L
2𝑦(𝑡) g
. The initial value problem for Tank 1 is thus
10 + 2𝑡 min
2
𝑦1′ = 40 − 𝑦1 , 𝑦1 (0) = 0.
10 + 2𝑡
Simplifying this equation and putting it in standard form gives
1 Solutions 29

1
𝑦1′ + 𝑦1 = 40.
5+𝑡
The integrating factor is 𝜇(𝑡) = 5 + 𝑡. Thus ((5 + 𝑡)𝑦1 )′ = 40(5 + 𝑡).
Integrating and simplifying gives 𝑦1 (𝑡) = 20(5 + 𝑡) + 𝑐/(5 + 𝑡). The initial
condition 𝑦(0) = 0 implies 𝑐 = −500 so 𝑦1 = 20(5 + 𝑡) − 500/(5 + 𝑡).
L 𝑦1 (𝑡) g
input rate for Tank 2: input rate = 2 × = 20 −
min 10 + 2𝑡 L
500 g
.
(5 + 𝑡)2 min
L 𝑦2 (𝑡) g 𝑦2 (𝑡) g
output rate for Tank 2: output rate = 1 × = .
min 5 + 𝑡 L 5 + 𝑡 min
The initial value problem for Tank 2 is thus
1
𝑦2′ = 20 − 500/(5 + 𝑡)2 − 𝑦2 , 𝑦2 (0) = 0.
(5 + 𝑡)

When this equation is put in standard form we get


1 500
𝑦2′ + 𝑦2 = 20 − .
(5 + 𝑡) (5 + 𝑡)2

The integrating factor is 𝜇(𝑡) = 5 + 𝑡. Thus


500
((5 + 𝑡)𝑦2 )′ = 20(5 + 𝑡) − .
5+𝑡
Integrating and simplifying gives

500 ln(5 + 𝑡) 𝑐
𝑦2 (𝑡) = 10(5 + 𝑡) − + .
5+𝑡 5+𝑡
The initial condition 𝑦2 (0) = 0 implies 𝑐 = 500 ln 5 − 250 so

500 ln(5 + 𝑡) 500 ln 5 − 250


𝑦2 (𝑡) = 10(5 + 𝑡) − + .
5+𝑡 5+𝑡

Section 1.5
𝑦 2 + 𝑦𝑡 + 𝑡2
1. In standard form we get 𝑦 ′ = which is homogeneous since
𝑡2
the degrees of the numerator and denominator are each two. Let 𝑦 = 𝑡𝑣.
Then 𝑣 + 𝑡𝑣 ′ = 𝑣 2 + 𝑣 + 1 and so 𝑡𝑣 ′ = 𝑣 2 + 1. Separating variables gives
𝑑𝑣 𝑑𝑡
= . Integrating gives tan−1 𝑣 = ln ∣𝑡∣ + 𝑐. So 𝑣 = tan(ln ∣𝑡∣ + 𝑐).
𝑣2 + 1 𝑡
Substituting 𝑣 = 𝑦/𝑡 gives 𝑦 = 𝑡 tan(ln ∣𝑡∣ + 𝑐).
30 1 Solutions

2. Since the numerator and denominator are homogeneous of degree 1 the


4 − 3𝑣
quotient is homogeneous. Let 𝑦 = 𝑡𝑣. Then 𝑣 + 𝑡𝑣 ′ = and so
1−𝑣
2
(𝑣 − 2)
𝑡𝑣 ′ = . Clearly, 𝑣 = 2 is an equilibrium solution. Separating the
1−𝑣
−𝑑𝑣 𝑑𝑣 𝑑𝑡
variables and using partial fractions gives − = . Integrat-
(𝑣 − 2)2 𝑣 − 2 𝑡
ing gives (𝑣 − 2)−1 − ln ∣𝑣 − 2∣ = ln ∣𝑡∣ + 𝑐. Simplifying and exponentiating
−1 𝑡
gives 𝑒(𝑣−2) = 𝑘𝑡(𝑣 − 2), 𝑘 ∕= 0. Now let 𝑣 = 𝑦/𝑡 then 𝑒 𝑦−2𝑡 = 𝑘(𝑦 − 2𝑡)
for 𝑘 ∕= 0. The equilibrium solution 𝑣 = 2 gives 𝑦 = 2𝑡 as another solution.

3. Since the numerator and denominator are homogeneous of degree 2 the


quotient is homogeneous. Let 𝑦 = 𝑡𝑣. Then 𝑣 + 𝑡𝑣 ′ = 𝑣 2 − 4𝑣 + 6. So
𝑡𝑣 ′ = 𝑣 2 − 5𝑣 + 6 = (𝑣 − 2)(𝑣 − 3). There are two equilibrium solu-
tions 𝑣 = 2, 3. Separating
( ) the variables and using partial fractions
gives

1 1 𝑑𝑡 𝑣 − 3
− 𝑑𝑣 = . Integrating and simplifying gives ln =
𝑣−3 𝑣−2 𝑡 𝑣 − 2
3 − 2𝑘𝑡 3𝑡 − 2𝑘𝑡2
ln ∣𝑡∣ + 𝑐. Solving for 𝑣 gives 𝑣 = , for 𝑘 ∕= 0, and so 𝑦 = ,
1 − 𝑘𝑡 1 − 𝑘𝑡
for 𝑘 ∕= 0. When 𝑘 = 0 we get 𝑣 = 3 or 𝑦 = 3𝑡, which is the same as the
equilibrium solution 𝑣 = 3. The equilibrium solution 𝑣 = 2 gives 𝑦 = 2𝑡.
3𝑡 − 2𝑘𝑡2
Thus we can write the solutions as 𝑦 = , 𝑘 ∈ ℝ and 𝑦 = 2𝑡.
1 − 𝑘𝑡
4. Since the numerator and denominator are homogeneous of degree 2 the
𝑡2 𝑣 2 + 2𝑡2 𝑣
quotient is homogeneous. Let 𝑦 = 𝑡𝑣. Then 𝑣 + 𝑡𝑣 ′ = =
𝑡2 + 𝑡 2 𝑣
2
𝑣 + 2𝑣 𝑣
. Subtract 𝑣 from both sides to get 𝑡𝑣 ′ = . Separate the
1+𝑣 𝑣+1
variables to get ( )
1 1
1+ 𝑑𝑣 = 𝑑𝑡.
𝑣 𝑡
Integrating gives 𝑣 + ln ∣𝑣∣ = ln ∣𝑡∣ + 𝑐. Now exponentiate, substitute 𝑣 =
𝑦/𝑡, and simplify to get 𝑦𝑒𝑦/𝑡 = 𝑘𝑡2 , 𝑘 ∈ ℝ.

5. Since the numerator and denominator are homogeneous of degree 2 the


3𝑣 2 − 1
quotient is homogeneous. Let 𝑦 = 𝑡𝑣. Then 𝑣 + 𝑡𝑣 ′ = . Subtract
2𝑣
2
𝑣 −1
𝑣 from both sides to get 𝑡𝑣 ′ = . The equilibrium solutions are
2𝑣
2𝑣 𝑑𝑣 𝑑𝑡
𝑣 = ±1. Separating variables gives 2 = and integrating gives
2 𝑣 − 1 𝑡
2
ln 𝑣 − 1 = ln ∣𝑡∣+𝑐. Exponentiating gives 𝑣 −1 = 𝑘𝑡 and by simplifying

√ √
we get 𝑣 = ± 1 + 𝑘𝑡. Now 𝑣 = 𝑦/𝑡 so 𝑦 = ±𝑡 1 + 𝑘𝑡. The equilibrium
solutions 𝑣 = ±1 become 𝑦 = ±𝑡. These occur when 𝑘 = 0, so are already
included in the general formula.
1 Solutions 31

6. Since 𝑡2 +𝑦 2 and 𝑡𝑦 are homogeneous of degree 2 their quotient (𝑡2 +𝑦 2 )/𝑡𝑦


is a homogeneous function. Let 𝑦 = 𝑡𝑣. Then 𝑦 ′ = 𝑣 + 𝑡𝑣 ′ and the given
differential equation becomes

1 + 𝑣2 1
𝑣 + 𝑡𝑣 ′ = = + 𝑣.
𝑣 𝑣
Simplifying and separating variables√gives 𝑣 𝑑𝑣 = 𝑑𝑡/𝑡. Integrating we
get 𝑣 2 /2
√ = ln 𝑡 + 𝑐 and so 𝑣 = ± 2 ln 𝑡 + 2𝑐. Since 𝑣 = 𝑦/𝑡 we get
𝑦 = ±𝑡 2 ln 𝑡 + 2𝑐.

′ 𝑦 + 𝑡2 − 𝑦 2 √
7. In standard form we get 𝑦 = . Since (𝛼𝑡)2 − (𝛼𝑦)2 =
√ √ 𝑡
𝛼2√(𝑡2 − 𝑦 2 ) = 𝛼 𝑡2 − 𝑦 2 for 𝛼 > 0 it is easy to see that 𝑦 ′ =
𝑦 + 𝑡2 − 𝑦 2 √
is homogeneous. Let 𝑦 = 𝑡𝑣. Then 𝑣 + 𝑡𝑣 ′ = 𝑣 + 1 − 𝑣 2 .
𝑡 √
Simplifying gives 𝑡𝑣 ′ = 1 − 𝑣 2 . Clearly 𝑣 = ±1 are equilibrium solution.
𝑑𝑣 𝑑𝑡
Separating variables gives √ = . Integrating gives sin−1 𝑣 =
1−𝑣 2 𝑡
ln ∣𝑡∣ + 𝑐 and so 𝑣 = sin(ln ∣𝑡∣ + 𝑐). Now substitute 𝑣 = 𝑦/𝑡 to get
𝑦 = 𝑡 sin(ln ∣𝑡∣ + 𝑐). The equilibrium solutions imply 𝑦 = ±𝑡 are also
solutions.

′ 𝑦 𝑦 𝑡2 + 𝑦 2
8. In standard form we get 𝑦 = + . It is straightforward to see
𝑡 𝑡2 √

2 2 ′ 𝑦 𝑦 𝑡2 + 𝑦 2
that 𝑡 + 𝑦 is homogeneous of degree one. So 𝑦 = +
𝑡 𝑡2 ′
is a homogeneous
√ differential
√ equation. Let 𝑦 = 𝑡𝑣 then 𝑣 + 𝑡𝑣 =
𝑣 + 𝑣 1 + 𝑣 2 or 𝑡𝑣 ′ = 𝑣 1 + 𝑣 2 . It follows that 𝑣 = 0 is an equilib-
𝑑𝑣 𝑑𝑡
rium solution. Separating variables gives √ = . Integrating
𝑣 1+𝑣 2 𝑡
𝑣
gives ln √ = ln ∣𝑡∣ + 𝑐. (To integrate the left hand side use
1+ 1+𝑣 2
𝑣
the trig substitution 𝑣 = tan 𝜃.) Exponentiating gives √ = 𝑘𝑡,
1 + 1 + 𝑣2
𝑦
𝑘 ∕= 0. Now let 𝑣 = 𝑦/𝑡. Then √ = 𝑘𝑡. The case where 𝑘 = 0
𝑡 + 𝑡2 + 𝑦 2
gives the equilibrium solution.

9. Note that although 𝑦 = 0 is part of the general solution it does not satisfy
the initial value. Divide both sides by 𝑦 2 to get 𝑦 −2 𝑦 ′ − 𝑦 −1 = 𝑡. Let
𝑧 = 𝑦 −1 . Then 𝑧 ′ = −𝑦 −2 𝑦 ′ . Substituting gives −𝑧 ′ − 𝑧 = 𝑡 or 𝑧 ′ + 𝑧 = −𝑡.
An integrating factor is 𝑒𝑡 . So (𝑒𝑡 𝑧) = −𝑡𝑒𝑡 . Integrating both sides gives
𝑡 𝑡 𝑡
∫𝑒 𝑧 = 𝑡−𝑡𝑒 + 𝑒 + 𝑐, where we have used integration by parts to compute
−𝑡𝑒 𝑑𝑡. Solving for 𝑧 gives 𝑧 = −𝑡 + 1 + 𝑐𝑒−𝑡 . Now substitute 𝑧 = 𝑦 −1
32 1 Solutions

1
and solve for 𝑦 to get 𝑦 = . The initial condition implies
−𝑡 + 1 + 𝑐𝑒−𝑡
1 1
1= and so 𝑐 = 0. The solution is thus 𝑦 = .
1+𝑐 1−𝑡
10. Note that 𝑦 = 0 is a solution. Divide by 𝑦 2 to get 𝑦 −2 𝑦 ′ + 𝑦 −1 = 1. Let
𝑧 = 𝑦 −1 . Then 𝑧 ′ = −𝑦 −2 𝑦 ′ and substituting gives −𝑧 ′ + 𝑧 = 1. In the
standard form for linear equations this becomes 𝑧 ′ − 𝑧 = −1. Multiplying
by the integrating factor 𝑒−𝑡 gives (𝑒−𝑡 𝑧)′ = −𝑒−𝑡 so that 𝑒−𝑡 𝑧 = 𝑒−𝑡 + 𝑐.
Hence 𝑧 = 1 + 𝑐𝑒𝑡 . Now go back to the original function 𝑦 by solving
1
𝑧 = 𝑦 −1 for 𝑦. Thus 𝑦 = 𝑧 −1 = (1 + 𝑐𝑒𝑡 )−1 = 1+𝑐𝑒 𝑡 . The general solution
1
is 𝑦 = 1+𝑐𝑒𝑡 and 𝑦 = 0.

11. Note that 𝑦 = 0 is a solution. First divide both sides by 𝑦 3 to get 𝑦 −3 𝑦 ′ +


𝑧′
𝑡𝑦 −2 = 𝑡. Let 𝑧 = 𝑦 −2 . Then 𝑧 ′ = −2𝑦 −3 𝑦 ′ , so = 𝑦 −3 𝑦 ′ . Substituting
−2
𝑧′
gives +𝑡𝑧 = 𝑡, which in standard form is 𝑧 ′ −2𝑡𝑧 = −2𝑡. An integrating
−2 ∫
2 2 2
factor is 𝑒 −2𝑡 𝑑𝑡 = 𝑒−𝑡 , so that (𝑒−𝑡 𝑧)′ = −2𝑡𝑒−𝑡 . Integrating both
2 2
sides gives 𝑒−𝑡 𝑧 = 𝑒−𝑡 + 𝑐, where the integral of the right hand side is
2
done by the substitution 𝑢 = −𝑡2 . Solving for 𝑧 gives 𝑧 = 1 + 𝑐𝑒𝑡 . Since
1
𝑧 = 𝑦 −2 we find 𝑦 = ± √ .
1 + 𝑐𝑒𝑡2
12. Note that 𝑦 = 0 is a solution. First divide both sides by 𝑦 3 to get
𝑧′
𝑦 −3 𝑦 ′ + 𝑡𝑦 −2 = 𝑡3 . Let 𝑧 = 𝑦 −2 . Then 𝑧 ′ = −2𝑦 −3 𝑦 ′ , so = 𝑦 −3 𝑦 ′ . Sub-
−2
𝑧′
stituting gives + 𝑡𝑧 = 𝑡3 , which in standard form is 𝑧 ′ − 2𝑡𝑧 = −2𝑡3 .
−2 ∫ 2 2 2
An integrating factor is 𝑒 −2𝑡 𝑑𝑡 = 𝑒−𝑡 . Thus (𝑒−𝑡 𝑧)′ = −2𝑡3 𝑒−𝑡 . Inte-
2 2
grating both sides gives 𝑒−𝑡 𝑧 = (𝑡2 + 1)𝑒−𝑡 + 𝑐, where the integral of the
right hand side is computed using integration by parts with 𝑢 = 𝑡2 and
2 2
𝑑𝑣 = −2𝑡𝑒−𝑡 𝑑𝑡. Solving for 𝑧 gives 𝑧 = 𝑡2 + 1 + 𝑐𝑒𝑡 . Since 𝑧 = 𝑦 −2 we
1
find 𝑦 = ± √ .
𝑡 + 1 + 𝑐𝑒𝑡2
2

13. Note that 𝑦 = 0 is a solution. Divide by 𝑦 2 and (1 − 𝑡2 ) to get 𝑦 −2 𝑦 ′ −


𝑡 5𝑡
2
𝑦 −1 = . Let 𝑧 = 𝑦 −1 . Then 𝑧 ′ = −𝑦 −2 𝑦 ′ and substituting
1−𝑡 1 − 𝑡2
𝑡 5𝑡 𝑡
gives −𝑧 ′ − 𝑦′ = . In standard form we get 𝑧 ′ + 𝑧=
1 − 𝑡2 (1 − 𝑡2 ) 1 − 𝑡2
−5𝑡
. Multiplying by the integrating factor
1 − 𝑡2
∫ 𝑡
𝑑𝑡 1 2
𝜇(𝑡) = 𝑒 1−𝑡2 = 𝑒− 2 ln(1−𝑡 )
= (1 − 𝑡2 )−1/2
1 Solutions 33

gives (𝑧(1 − 𝑡2 )−1/2 )′ = −5𝑡(1 − 𝑡2)−3/2 . Integrating


√ gives 𝑧(1 − 𝑡2)−1/2 =
2 −1/2
−5(1 − 𝑡 ) + 𝑐 and hence 𝑧 = −5 + 𝑐 1 − 𝑡 . Since 𝑧 = 𝑦 −1 we have
2
1
𝑦= √ .
−5 + 𝑐 1 − 𝑡2
14. Note that 𝑦 = 0 is a solution. Divide both sides by 𝑦 2/3 to get 𝑦 −2/3 𝑦 ′ +
𝑦 1/3 1 𝑧
= 1. Let 𝑧 = 𝑦 1/3 . Then 𝑧 ′ = 𝑦 −2/3 𝑦 ′ and hence 3𝑧 ′ + = 1. In
𝑡 3 𝑡∫
′ 𝑧 1 1
standard form we get 𝑧 + = . The integrating factor is 𝑒 3𝑡 𝑑𝑡 =
3𝑡 3
ln 𝑡 1
𝑒 3 = 𝑡1/3 . Multiplying by 𝑡1/3 gives (𝑡1/3 𝑧)′ = 𝑡1/3 and integrating
3
1 𝑡
gives 𝑡1/3 𝑧 = 𝑡4/3 + 𝑐. Solving for 𝑧 we get 𝑧 = + 𝑐𝑡−1/3 . Since 𝑧 = 𝑦 1/3
4 4
we can solve for 𝑦 to get 𝑦 = (𝑡/4 + 𝑐𝑡−1/3 )3 .

15. If we divide by 𝑦 we get 𝑦 ′ + 𝑡𝑦 = 𝑡𝑦 −1 which is a Bernoulli equation with


𝑛 = −1. Note that since 𝑛 < 0, 𝑦 = 0 is not a solution. Dividing by 𝑦 −1
𝑧′
gets us back to 𝑦𝑦 ′ + 𝑡𝑦 2 = 𝑡. Let 𝑧 = 𝑦 2 . Then 𝑧 ′ = 2𝑦𝑦 ′ so + 𝑡𝑧 = 𝑡
2 2
and in standard form we get 𝑧 ′ + 2𝑡𝑧 = 2𝑡. An integrating factor is 𝑒𝑡 so
2 2 2 2 2
(𝑒𝑡 𝑧)′ = 2𝑡𝑒𝑡 . Integration
√ gives 𝑒𝑡 𝑧 = 𝑒𝑡 + 𝑐 so 𝑧 = 1 + 𝑐𝑒−𝑡 . Since
𝑧 = 𝑦 2 we get 𝑦 = ± 1 + 𝑐𝑒−𝑡2 .
1 𝑡 − 1 −1
16. First divide by 2𝑦 to get 𝑦 ′ − 𝑦 = 𝑦 , which is a Bernoulli equation
2 2
with 𝑛 = −1. Since 𝑛 < 0, 𝑦 = 0 is not a solution. Now divide by 𝑦 −1 to
1 𝑡−1
get 𝑦𝑦 ′ − 𝑦 2 = . Let 𝑧 = 𝑦 2 . Then 𝑧 ′ = 2𝑦𝑦 ′ and substituting gives
2 2
1 ′ 1 𝑡−1
𝑧 − 𝑧= . In standard form we get 𝑧 ′ − 𝑧 = (𝑡 − 1). An integrating
2 2 ∫ 2
factor is 𝑒 −1 𝑑𝑡 = 𝑒−𝑡 . Multiplying by 𝑒−𝑡 gives (𝑒−𝑡 𝑧)′ = (𝑡 − 1)𝑒−𝑡 .
−𝑡 −𝑡 𝑡
Integration by parts gives
√ 𝑒 𝑧 = −𝑡𝑒 + 𝑐 and thus 𝑧 = −𝑡 + 𝑐𝑒 . Since
2 𝑡
𝑧 = 𝑦 we have 𝑦 = ± 𝑐𝑒 − 𝑡, 𝑐 ∈ ℝ.

17. Note that 𝑦 = 0 is a solution. First divide both sides by 𝑦 3 to get 𝑦 −3 𝑦 ′ +


𝑧′
𝑦 −2 = 𝑡. Let 𝑧 = 𝑦 −2 . Then 𝑧 ′ = −2𝑦 −3 𝑦 ′ . So +𝑧 = 𝑡. In standard form
−2 ∫
we get 𝑧 ′ − 2𝑧 = −2𝑡. An integrating factor is 𝑒 −2 𝑑𝑡 = 𝑒−2𝑡 and hence
1
(𝑒−2𝑡 𝑧)′ = −2𝑡𝑒−2𝑡 . Integration by parts gives 𝑒−2𝑡 𝑧 = (𝑡 + )𝑒−2𝑡 + 𝑐 and
2
1 1
hence 𝑧 = 𝑡 + + 𝑐𝑒2𝑡 . Since 𝑧 = 𝑦 −2 we get 𝑦 = ± √ .
2 𝑡 + 12 + 𝑐𝑒2𝑡

𝑃
18. The logistic differential equation is 𝑃 ′ = 𝑟(1 − )𝑃 which can be written
𝑚
𝑟
𝑃 ′ − 𝑟𝑃 = − 𝑃 2 . Note that 𝑃 = 0 is a solution. Divide by 𝑃 2 to get
𝑚
34 1 Solutions
−𝑟 −𝑟
𝑃 −2 𝑃 ′ −𝑟𝑃 −1 = . Let 𝑧 = 𝑃 −1 . Then 𝑧 ′ = −𝑃 −2 𝑃 ′ and −𝑧 ′ −𝑟𝑧 =
𝑚 𝑚
𝑟 𝑟
or 𝑧 ′ + 𝑟𝑧 = . An integrating factor is 𝑒𝑟𝑡 so (𝑒𝑟𝑡 𝑧)′ = 𝑒𝑟𝑡 . Integrating
𝑚 𝑚
𝑟𝑡 𝑒𝑟𝑡 1 −𝑟𝑡 1 + 𝑚𝑐𝑒−𝑟𝑡
gives 𝑒 𝑧 = + 𝑐. Solving for 𝑧 we get 𝑧 = + 𝑐𝑒 = .
𝑚 𝑚 𝑚
𝑚 𝑚
Since 𝑧 = 𝑃 −1 we get 𝑃 = . Now 𝑃0 = 𝑃 (0) =
1 + 𝑚𝑐𝑒−𝑟𝑡 1 + 𝑚𝑐
𝑚 − 𝑃0
and solving for 𝑐 we get 𝑐 = . Substituting and simplifying gives
𝑚𝑃0
𝑚𝑃0
𝑃 (𝑡) = .
𝑃0 + (𝑚 − 𝑃0 )𝑒−𝑟𝑡
2 − 𝑧′
19. Let 𝑧 = 2𝑡 − 2𝑦 + 1. Then 𝑧 ′ = 2 − 2𝑦 ′ and so 𝑦 ′ = . Substituting
2
2 − 𝑧′
we get = 𝑧 −1 and in standard form we get 𝑧 ′ = 2 − 2𝑧 −1 , a
2
separable differential equation. Clearly, 𝑧 = 1 is an equilibrium solution.
Assume for now that 𝑧 ∕= 1. Then separating
( variables
) and simplifying
−1 𝑧 1 1
using 1/(1−𝑧 ) = 𝑧−1 = 1+ 𝑧−1 gives 1 + 𝑧−1 𝑑𝑧 = 2 𝑑𝑡. Integrating
we get 𝑧 + ln ∣𝑧 − 1∣ = 2𝑡 + 𝑐. Now substitute 𝑧 = 2𝑡 − 2𝑦 + 1 and simplify
to get −2𝑦 + ln ∣2𝑡 − 2𝑦∣ = 𝑐, 𝑐 ∈ ℝ. (We absorb the constant 1 in 𝑐.) The
equilibrium solution 𝑧 = 1 becomes 𝑦 = 𝑡.

20. Let 𝑧 = 𝑡 − 𝑦. Then 𝑧 ′ = 1 − 𝑦 ′ and so 𝑦 ′ = 1 − 𝑧 ′ . Substituting we get


1 − 𝑧 ′ = 𝑧 2 and in standard form we get 𝑧 ′ = 1 − 𝑧 2 . We see that 𝑧 = ±1
𝑑𝑧
are equilibrium solutions. Separating variables we get = 𝑑𝑡. Partial
( ) 1 − 𝑧2
1 1
fractions gives + 𝑑𝑧 = 2 𝑑𝑡. Integrating and simplifying
1 − 𝑧 1 + 𝑧
2𝑡

1 + 𝑧
gives ln = 2𝑡 + 𝑐, 𝑐 ∈ ℝ. Solving for 𝑧 we get 𝑧 = 𝑘𝑒 − 1 , 𝑘 ∕= 0.
1 − 𝑧 𝑘𝑒2𝑡 + 1
However, the case 𝑘 = 0 gives the equilibrium solution 𝑧 = −1. Now
𝑘𝑒2𝑡 − 1
substitute 𝑧 = 𝑡 − 𝑦 and simplify to get 𝑦 = 𝑡 − 2𝑡 , 𝑘 ∈ ℝ. The
𝑘𝑒 + 1
equilibrium solution 𝑧 = 1 becomes 𝑦 = 𝑡 − 1.

21. Let 𝑧 = 𝑡 + 𝑦. Then 𝑧 ′ = 1 + 𝑦 ′ and substituting we get 𝑧 ′ − 1 = 𝑧 −2 . In


1 + 𝑧2
standard form we get 𝑧 ′ = . Separating variables and simplifying
( ) 𝑧2
1
we get 1 − 𝑑𝑧 = 𝑑𝑡. Integrating we get 𝑧 − tan−1 𝑧 = 𝑡 + 𝑐. Now
1 + 𝑧2
let 𝑧 = 𝑡 + 𝑦 and simplify to get 𝑦 − tan−1 (𝑡 + 𝑦) = 𝑐, 𝑐 ∈ ℝ.

22. Let 𝑧 = 𝑡 − 𝑦. Then 𝑧 ′ = 1 − 𝑦 ′ and substituting we get 1 − 𝑧 ′ = sin 𝑧. In


𝜋
standard form we get 𝑧 ′ = 1 − sin 𝑧. Notice that 𝑧 = + 2𝜋𝑛, 𝑛 ∈ ℤ, are
2
1 Solutions 35

𝑑𝑧
equilibrium solutions. Separating variable gives = 𝑑𝑡. Now
1 − sin 𝑧
1 1 + sin 𝑧 1 + sin 𝑧
= 2 = = sec2 𝑧 + sec 𝑧 tan 𝑧.
1 − sin 𝑧 1 − sin 𝑧 cos2 𝑧
So integrating gives tan 𝑧 + sec 𝑧 = 𝑡 + 𝑐. Substituting, we get the implicit
solution tan(𝑡 − 𝑦) + sec(𝑡 − 𝑦) = 𝑡 + 𝑐. For the equilibrium solution
𝜋 𝜋
𝑧 = + 2𝜋𝑛 we get 𝑦 = 𝑡 − − 2𝜋𝑛.
2 2
23. This is the same as Exercise 16 where the Bernoulli equation technique
there used the substitution 𝑧 = 𝑦 2 . Here use the given substitution to
get 𝑧 ′ = 2𝑦𝑦 ′ + 1. Substituting we get 𝑧 ′ − 1 = 𝑧 and in standard form
𝑧 ′ = 1+𝑧. Clearly, 𝑧 = −1 is an equilibrium solution. Separating variables
𝑑𝑧
gives = 𝑑𝑡 and integrating gives ln ∣1 + 𝑧∣ = 𝑡 + 𝑐, 𝑐 ∈ ℝ. Solving
1+𝑧
for 𝑧 we get 𝑧 = 𝑘𝑒𝑡 − 1, where 𝑘 ∕= 0. Since√𝑧 = 𝑦 2 + 𝑡 − 1 we get
𝑦 2 + 𝑡 − 1 = 𝑘𝑒𝑡 − 1 and solving for 𝑦 gives 𝑡
√ 𝑦 = ± 𝑘𝑒 − 𝑡. The case 𝑘 = 0
gives the equilibrium solutions 𝑦 = ± −𝑡.

24. If 𝑧 = sin 𝑦 then 𝑧 ′ = (cos 𝑦)𝑦 ′ . Multiply the given differential equation by
cos 𝑦 to get (cos 𝑦)𝑦 ′ = sin 𝑦 + 2 cos 𝑡. Substituting we get 𝑧 ′ = 𝑧 + 2 cos 𝑡
and in standard form 𝑧 ′ − 𝑧 = 2 cos 𝑡. An integrating factor is 𝑒−𝑡 so
(𝑧𝑒−𝑡 )′ = 2(cos 𝑡)𝑒−𝑡 . Integrating by parts twice leads to 𝑧𝑒−𝑡 = (sin 𝑡 −
cos 𝑡)𝑒−𝑡 + 𝑐 and hence 𝑧 = sin 𝑡 − cos 𝑡 + 𝑐𝑒𝑡 . Solving for 𝑦 gives 𝑦 =
sin−1 (sin 𝑡 − cos 𝑡 + 𝑐𝑒𝑡 ), 𝑐 ∈ ℝ.
𝑦′
25. If 𝑧 = ln 𝑦 then 𝑧 ′ = . Divide the given differential equation by 𝑦. Then
𝑦
𝑦′
+ ln 𝑦 = 𝑡 and substitution gives 𝑧 ′ + 𝑧 = 𝑡. An integrating factor is
𝑦
𝑒𝑡 so (𝑒𝑡 𝑧)′ = 𝑡𝑒𝑡 . Integration (by parts) gives 𝑒𝑡 𝑧 = (𝑡 − 1)𝑒𝑡 + 𝑐 and so
−𝑡
𝑧 = 𝑡 − 1 + 𝑐𝑒−𝑡 . Finally, solving for 𝑦 we get 𝑦 = 𝑒𝑡−1+𝑐𝑒 , 𝑐 ∈ ℝ.
−𝑧 ′
26. Let 𝑧 = 𝑒−𝑦 . Then 𝑧 ′ = −𝑒−𝑦 𝑦 ′ so −𝑧 ′ /𝑧 = 𝑦 ′ . Substituting gives =
𝑧
−1
−1. Multiply both sides by 𝑧 and put in standard form to get 𝑧 ′ −𝑧 = 1.
𝑧
An integrating factor is 𝑒−𝑡 so (𝑒−𝑡 𝑧)′ = 𝑒−𝑡 . Integrating we get 𝑒−𝑡 𝑧 =
−𝑒−𝑡 + 𝑐 and so 𝑧 = 𝑐𝑒𝑡 − 1. Since 𝑧 = 𝑒−𝑦 , 𝑧 > 0 and this requires 𝑐 > 0.
We thus get 𝑦 = − ln(𝑐𝑒𝑡 − 1), 𝑐 > 0.

Section 1.6
1. This can be written in the form 𝑀 (𝑡, 𝑦) + 𝑁 (𝑡, 𝑦)𝑦 ′ = 0 where 𝑀 (𝑡, 𝑦) =
𝑦 2 + 2𝑡 and 𝑁 (𝑡, 𝑦) = 2𝑡𝑦. Since 𝑀𝑦 (𝑡, 𝑦) = 2𝑦 = 𝑁𝑡 (𝑡, 𝑦), the equation is
exact (see Equation (3.2.2)), and the general solution is given implicitly
36 1 Solutions

by 𝐹 (𝑡, 𝑦) = 𝑐 where the function 𝐹 (𝑡, 𝑦) is determined by 𝐹𝑡 (𝑡, 𝑦) =


𝑀 (𝑡, 𝑦) = 𝑦 2 + 2𝑡 and 𝐹𝑦 (𝑡, 𝑦) = 𝑁 (𝑡, 𝑦) = 2𝑡𝑦. These equations imply
that 𝐹 (𝑡, 𝑦) = 𝑡2 + 𝑡𝑦 2 will work so the solutions are given implicitly by
𝑡2 + 𝑡𝑦 2 = 𝑐.
1
2. 𝑡𝑦 + 𝑦 2 − 𝑡2 = 𝑐
2
3. Not Exact

4. 𝑡𝑦 2 + 𝑡3 = 𝑐

5. Not Exact

6. 𝑡2 𝑦 + 𝑦 3 = 2

7. (𝑦 − 𝑡2 )2 − 2𝑡4 = 𝑐
1 2 𝑐
8. 𝑦 = 𝑡 −
3 𝑡
9. 𝑦 4 = 4𝑡𝑦 + 𝑐

10. 𝑏 + 𝑐 = 0

Section 1.7
1. We first change the variable 𝑡 to 𝑢 and write 𝑦 ′ (𝑢) = 𝑢𝑦(𝑢). Now integrate
∫𝑡 ∫𝑡
both sides from 1 to 𝑡 to get 1 𝑦 ′ (𝑢) 𝑑𝑢 = 1 𝑢𝑦(𝑢) 𝑑𝑢. Now the left side
∫𝑡 ′ ∫𝑡
is 1 𝑦 (𝑢) 𝑑𝑢 = 𝑦(𝑡) − 𝑦(1) = 𝑦(𝑡) − 1. Thus 𝑦(𝑡) = 1 + 1 𝑢𝑦(𝑢) 𝑑𝑢.

2. Change the variable 𝑡 to 𝑢 and write 𝑦 ′ (𝑢) = 𝑦 2 (𝑢). Now integrate both
∫𝑡 ∫𝑡
sides from 0 to 𝑡 to get 0 𝑦 ′ (𝑢) 𝑑𝑢 = 0 𝑦 2 (𝑢) 𝑑𝑢. The left side is 𝑦(𝑡) + 1
∫𝑡 2
so 𝑦(𝑡) = −1 + 0 𝑦 (𝑢) 𝑑𝑢.

𝑢 − 𝑦(𝑢)
3. Change the variable 𝑡 to 𝑢 and write 𝑦 ′ (𝑢) = . Now integrate
𝑢 + 𝑦(𝑢)
∫𝑡 ∫𝑡 𝑢 − 𝑦(𝑢)
both sides from 0 to 𝑡 to get 0 𝑦 ′ (𝑢) 𝑑𝑢 = 0 𝑑𝑢. The left side is
𝑢 + 𝑦(𝑢)
∫ 𝑡 𝑢 − 𝑦(𝑢)
𝑦(𝑡) − 1 so 𝑦(𝑡) = 1 + 0 𝑢 + 𝑦(𝑢)
𝑑𝑢.

4. Change the variable 𝑡 to 𝑢 and write 𝑦 ′ (𝑢) = 1 + 𝑢2 . Now integrate both


∫𝑡 ∫𝑡
sides from 0 to 𝑡 to get 0 𝑦 ′ (𝑢) 𝑑𝑢 = 0 (1 + 𝑢2 ) 𝑑𝑢. The left side is 𝑦(𝑡)
∫𝑡
so 𝑦(𝑡) = 0 (1 + 𝑢2 ) 𝑑𝑢.
∫𝑡
5. The corresponding integral equation is 𝑦(𝑡) = 1 + 1 𝑢𝑦(𝑢) 𝑑𝑢. We then
have
1 Solutions 37

𝑦0 (𝑡) = 1
𝑡 ) 𝑡
𝑢2 𝑡2 1 + 𝑡2
(
1

𝑦1 (𝑡) = 1 + 𝑢 ⋅ 1 𝑑𝑢 = 1 + = 1 + − =
1 2 1 2 2 2
∫ 𝑡 ( 𝑡
1 + 𝑢2 𝑢2 𝑢4 5 𝑡2 𝑡4
) ( )
𝑦2 (𝑡) = 1 + 𝑢 𝑑𝑢 = 1 + + = + +
1 2 4 8 1 8 4 8
∫ 𝑡( 3 5 6 𝑡
) ( 2 4
)
5𝑢 𝑢 𝑢 5𝑢 𝑢 𝑢
𝑦3 (𝑡) = 1 + + + 𝑑𝑢 = 1 + + +
1 8 4 8 16 16 48 1
29 5𝑡2 𝑡4 𝑡6
= + + + .
48 16 16 48

6. The corresponding integral equation is


∫ 𝑡
𝑦(𝑡) = 1 + (𝑢 − 𝑦(𝑢)) 𝑑𝑢.
0
We then have
𝑦0 (𝑡) = 1
𝑡 ) 𝑡
𝑢2 𝑡2
∫ (

𝑦1 (𝑡) = 1 + (𝑢 − 1) 𝑑𝑢 = 1 +
− 𝑢 = 1 − 𝑡 +
0 2 0 2
∫ 𝑡( ) 𝑡
𝑢2 𝑢3
( )) (
𝑦2 (𝑡) = 1 + 𝑢− 1−𝑢+ 𝑑𝑢 = 1 + −𝑢 + 𝑢2 −
0 2 6 0
𝑡3
= 1 − 𝑡 + 𝑡2 −
6
∫ 𝑡(
𝑢3 𝑡3 𝑡4
( ))
2
𝑦3 (𝑡) = 1 + 𝑢− 1−𝑢+𝑢 − 𝑑𝑢 = 1 − 𝑡 + 𝑡2 − +
0 6 3 4!
∫ 𝑡( 3 4
𝑡3 𝑡4 𝑡5
( ))
𝑢 𝑢
𝑦4 (𝑡) = 1 + 𝑢 − 1 − 𝑢 + 𝑢2 − + 𝑑𝑢 = 1 − 𝑡 + 𝑡2 − + − .
0 3 4! 3 12 5!
∫𝑡
7. The corresponding integral equation is 𝑦(𝑡) = 0
(𝑢 + 𝑦 2 (𝑢)) 𝑑𝑢. We then
have
𝑦0 (𝑡) = 0
∫ 𝑡
𝑡2
𝑦1 (𝑡) = (𝑢 + 0) 𝑑𝑢 =
0 2
∫ 𝑡( ( 2 )2 ) ∫ 𝑡(
𝑢4 𝑡2 𝑡5
)
𝑢
𝑦2 (𝑡) = 𝑢+ 𝑑𝑢 = 𝑢+ 𝑑𝑢 = +
0 2 0 4 2 20
∫ 𝑡( 2
)
( 2 𝑡
𝑢5 𝑢4 𝑢7 𝑢10
) ( )
𝑢

𝑦3 (𝑡) = 𝑢+ + 𝑑𝑢 = 𝑢+ + + 𝑑𝑢
0 2 20 0 4 20 400
𝑡2 𝑡5 𝑡8 𝑡11
= + + + .
2 20 160 4400
38 1 Solutions

8. The corresponding integral equation is


∫ 𝑡
𝑦(𝑡) = 1 + ((𝑦(𝑢))3 − 𝑦(𝑢)) 𝑑𝑢.
0

We now have

𝑦0 (𝑡) = 1
∫ 𝑡 ∫ 𝑡
𝑦1 (𝑡) = 1 + (13 − 1) 𝑑𝑢 = 1 + 0 𝑑𝑢 = 1
0 0
𝑦2 (𝑡) = 1
𝑦3 (𝑡) = 1

9. The corresponding integral equation is


∫ 𝑡
𝑦(𝑡) = (1 + (𝑢 − 𝑦(𝑢))2 ) 𝑑𝑢.
0

We then have

𝑦0 (𝑡) = 0
∫ 𝑡 ) 𝑡
𝑢3 𝑡3
(
2
( )
𝑦1 (𝑡) = 1 + (𝑢 − 0) 𝑑𝑢 = 𝑢 + =𝑡+
0 3 0 3
∫ 𝑡( 2
)
𝑡
𝑢3 𝑢6
( ( )) ∫ ( )
𝑦2 (𝑡) = 1+ 𝑢− 𝑢+ 𝑑𝑢 = 1+ 𝑑𝑢
0 3 0 9
) 𝑡
𝑢7 𝑡7
(
= 𝑢+ = 𝑡 +
63 0 7 ⋅ 32
∫ 𝑡( 14
𝑡15
)
𝑢
𝑦3 (𝑡) = 1 + 2 4 𝑑𝑢 = 𝑡 +
0 7 ⋅3 15 ⋅ 72 ⋅ 34
∫ 𝑡(
𝑢30 𝑡31
)
𝑦4 (𝑡) = 1 + 2 4 8 𝑑𝑢 = 𝑡 +
0 15 ⋅ 7 ⋅ 3 31 ⋅ 152 ⋅ 74 ⋅ 38
∫ 𝑡(
𝑢62 𝑡63
)
𝑦5 (𝑡) = 1+ 2 4 8 1
𝑑𝑢 = 𝑡 +
0 31 ⋅ 15 ⋅ 7 ⋅ 3 6 63 ⋅ 31 ⋅ 154 ⋅ 78 ⋅ 316
2

10. The right hand side is 𝐹 (𝑡, 𝑦) = 1 + 𝑦 2 . Then 𝐹𝑦 (𝑡, 𝑦) = 2𝑦. Both 𝐹 and
𝐹𝑦 are continuous in the whole (𝑡, 𝑦)-plane and thus are continuous on
any rectangle containing the origin (0, 0). Picard’s theorem applies and
we can conclude there is a unique solution on an interval about 0.

11. The right hand side is 𝐹 (𝑡, 𝑦) = 𝑦. If ℛ is any rectangle about (1, 0)
then there are 𝑦-coordinates that are negative. Hence 𝐹 is not defined on
ℛ and Picards’ theorem does not apply.
1 Solutions 39

√ 1
12. The right hand side is 𝐹 (𝑡, 𝑦) = √ . Choose a
𝑦. Then 𝐹𝑦 (𝑡, 𝑦) =
2 𝑦
rectangle ℛ about (0, 1) that lies above the 𝑡-axis. Then both 𝐹 and 𝐹𝑦
are continuous on ℛ. Picard’s theorem applies and we can conclude there
is a unique solution on an interval about 0.
𝑡−𝑦 −2𝑡
13. The right hand side is 𝐹 (𝑡, 𝑦) = . Then 𝐹𝑦 (𝑡, 𝑦) = . Choose
𝑡+𝑦 (𝑡 + 𝑦)2
a rectangle ℛ about (0, −1) that contains no points on the line 𝑡 + 𝑦 = 0.
Then both 𝐹 and 𝐹𝑦 are continuous on ℛ. Picard’s theorem applies and
we can conclude there is a unique solution on an interval about 0.
𝑡−𝑦
14. The right hand side is 𝐹 (𝑡, 𝑦) = , which is not defined at the initial
𝑡+𝑦
condition (𝑡0 , 𝑦0 ) = (1, −1). Thus Picard’s theorem does not apply.
∫𝑡
15. The corresponding integral equation is 𝑦(𝑡) = 1 + 0 𝑎𝑦(𝑢) 𝑑𝑢. We thus
have

𝑦0 (𝑡) = 1
∫ 𝑡
𝑦1 (𝑡) = 1 + 𝑎 𝑑𝑢 = 1 + 𝑎𝑡
0
𝑡 ∫ 𝑡
𝑎 2 𝑡2

𝑦2 (𝑡) = 1 + 𝑎(1 + 𝑎𝑢) 𝑑𝑢 = 1 + (𝑎 + 𝑎2 𝑢) 𝑑𝑢 = 1 + 𝑎𝑡 +
0 0 2
∫ 𝑡 (
𝑎 2 𝑢2 𝑎 2 𝑡2 𝑎 3 𝑡3
)
𝑦3 (𝑡) = 1 + 𝑎 1 + 𝑎𝑢 + 𝑑𝑢 = 1 + 𝑎𝑡 + +
0 2 2 3!
..
.
𝑎 2 𝑡2 𝑎 𝑛 𝑡𝑛
𝑦𝑛 (𝑡) = 1 + 𝑎𝑡 + +⋅⋅⋅ + .
2 𝑛!
𝑎 𝑘 𝑡𝑘
∑𝑛
We can write 𝑦𝑛 (𝑡) = 𝑘=0. We recognize this sum as the first 𝑛
𝑘!
terms of the Taylor series expansion for 𝑒𝑎𝑡 . Thus the limiting function
is 𝑦(𝑡) = lim𝑛→∞ 𝑦𝑛 (𝑡) = 𝑒𝑎𝑡 . It is straightforward to verify that it is a
solution. If 𝐹 (𝑡, 𝑦) = 𝑎𝑦 then 𝐹𝑦 (𝑡, 𝑦) = 𝑎. Both 𝐹 and 𝐹𝑦 are continuous
on the whole (𝑡, 𝑦)-plane. By Picard’s theorem, Theorem 5, 𝑦(𝑡) = 𝑒𝑎𝑡 is
the only solution to the given initial value problem.

16. 1. The equation is separable so separate the variables to get 𝑦 −2 𝑑𝑦 = 𝑑𝑡.


Integrating gives −𝑦 −1 = 𝑡 + 𝑐 and the initial condition 𝑦(0) = 1
implies that the integration constant 𝑐 = −1, so that the exact solution
is
1
𝑦(𝑡) = = 1 + 𝑡 + 𝑡2 + 𝑡3 + 𝑡4 + ⋅ ⋅ ⋅ ; ∣𝑡∣ < 1.
1−𝑡
2. To apply Picard’s method, let 𝑦0 = 1 and define
40 1 Solutions
∫ 𝑡 ∫ 𝑡
𝑦1 (𝑡) = 1 + (𝑦0 (𝑢))2 𝑑𝑢 = 1 + 1 𝑑𝑢 = 1 + 𝑡;
0 0
𝑡 𝑡
𝑡3
∫ ∫
𝑦2 (𝑡) = 1 + (𝑦1 (𝑢))2 𝑑𝑢 = 1 + (1 + 𝑢)2 𝑑𝑢 = 1 + 𝑡 + 𝑡2 + ;
0 0 3
𝑡 ∫ 𝑡( )2
𝑢3

2 2
𝑦3 (𝑡) = 1 + (𝑦2 (𝑢)) 𝑑𝑢 = 1 + 𝑑𝑢 1+𝑢+𝑢 +
0 0 3
∫ 𝑡( )
8 5 2 1
= 1+ 1 + 2𝑢 + 3𝑢2 + 𝑢3 + 𝑢4 + 𝑢5 + 𝑢6 𝑑𝑢
0 3 3 3 9
2 1 1 1
= 1 + 𝑡 + 𝑡 2 + 𝑡 3 + 𝑡4 + 𝑡5 + 𝑡6 + 𝑡7 .
3 3 9 63
Comparing 𝑦3 (𝑡) to the exact solution, we see that the series agree up
to order 3.

17. Let 𝐹 (𝑡, 𝑦) = cos(𝑡 + 𝑦). Then 𝐹𝑦 (𝑡, 𝑦) = − sin(𝑡 + 𝑦). Let 𝑦1 and
𝑦2 be arbitrary real numbers. Then by the mean value theorem there
is a number 𝑦0 in between 𝑦1 and 𝑦2 such that ∣𝐹 (𝑡, 𝑦1 ) − 𝐹 (𝑡, 𝑦2 )∣ =
∣sin(𝑡 + 𝑦0 )∣ ∣𝑦1 − 𝑦2 ∣ ≤ ∣𝑦1 − 𝑦2 ∣ . It follows that 𝐹 (𝑡, 𝑦) is Lipschitz on
any strip. Theorem 10 implies there is a unique solution on all of ℝ.

18. Let 𝐹 (𝑡, 𝑦) = −𝑝(𝑡)𝑦 + 𝑓 (𝑡). Since 𝑝(𝑡) and 𝑓 (𝑡) are continuous on [𝑎, 𝑏]
it follows that 𝐹 (𝑡, 𝑦) is continuous on the strip {(𝑡, 𝑦) : 𝑡 ∈ [𝑎, 𝑏], 𝑦 ∈ ℝ}.
Further more 𝑝(𝑡) is bounded: i.e. there is a number 𝐴 such that ∣𝑝(𝑡)∣ ≤ 𝐴,
for all 𝑡 ∈ [𝑎, 𝑏]. Let 𝑡 ∈ [𝑎, 𝑏] and 𝑦1 and 𝑦2 be arbitrary real numbers.
Then

∣𝐹 (𝑡, 𝑦1 ) − 𝐹 (𝑡, 𝑦2 )∣ ≤ ∣−𝑝(𝑡)𝑦1 + 𝑓 (𝑡) − (−𝑝(𝑡)𝑦2 + 𝑓 (𝑡))∣


= ∣𝑝(𝑡)∣ ∣𝑦1 − 𝑦2 ∣ ≤ 𝐴 ∣𝑦1 − 𝑦2 ∣ .

It follows that 𝐹 (𝑡, 𝑦) is Lipschitz with Lipschitz constant 𝐴. By Theorem


10, 𝑦 ′ + 𝑝(𝑡)𝑦 = 𝑓 (𝑡), 𝑦(𝑡0 ) = 𝑦0 has a unique solution on the entire
interval [𝑎, 𝑏].

19. 1. First assume that 𝑡 ∕= 0. Then 𝑡𝑦 ′ = 2𝑦 − 𝑡 is linear and in


standard ∫form becomes 𝑦 ′ − 2𝑦/𝑡 = −1. An integrating factor is
𝜇(𝑡) = 𝑒 (−2/𝑡) 𝑑𝑡 = 𝑡−2 and multiplying both sides by 𝜇 gives
𝑡−2 𝑦 ′ − 2𝑡−3 𝑦 = −𝑡−2 . This simplifies to (𝑡−2 𝑦)′ = −𝑡−2 . Now in-
tegrate to get 𝑡−2 𝑦 = 𝑡−1 + 𝑐 or 𝑦(𝑡) = 𝑡 + 𝑐𝑡2 . We observe that this
solution is also valid for 𝑡 = 0. Graphs are given below for various
values of 𝑐.
1 Solutions 41

−1

−2

−3

−4

−6 −4 −2 0 2 4 6
t

Graph of 𝑦(𝑡) = 𝑡 + 𝑐𝑡3 for various 𝑐


2. Every solution satisfies 𝑦(0) = 0. There is no contradiction to Theorem
2
5 since, in standard form, the equation is 𝑦 ′ = 𝑦 − 1 = 𝐹 (𝑡, 𝑦) and
𝑡
𝐹 (𝑡, 𝑦) is not continuous for 𝑡 = 0.

20. 1. If 𝐹 (𝑡, 𝑦) = 𝑦 2 then 𝐹𝑦 (𝑡, 𝑦) = 2𝑦. Both are continuous on any rectan-
gle that contains (𝑡0 , 𝑦0 ). Hence Theorem 5 applies and implies there
is a unique solution on an interval that contains 𝑡0 .
1
2. 𝑦(𝑡) = 0 is a solution defined for all 𝑡; 𝑦(𝑡) = is a solution defined
1−𝑡
on (−∞, 1) .

21. No. Both 𝑦1 (𝑡) and 𝑦2 (𝑡) would be solutions to the initial value problem
𝑦 ′ = 𝐹 (𝑡, 𝑦), 𝑦(0) = 0. If 𝐹 (𝑡, 𝑦) and 𝐹𝑦 (𝑡, 𝑦) are both continuous near
(0, 0), then the initial value problem would have a unique solution by
Theorem 5.

22. No, Both 𝑦1 (𝑡) and 𝑦2 (𝑡) would be solutions to the initial value problem
𝑦 ′ = 𝐹 (𝑡, 𝑦), 𝑦(0) = 1. If 𝐹 (𝑡, 𝑦) and 𝐹𝑦 (𝑡, 𝑦) are both continuous near
(0, 1), then the initial value problem would have a unique solution by
Theorem 5.

23. For 𝑡 < 0 we have 𝑦1′ (𝑡) = 0 and for 𝑡 > 0 we have 𝑦1′ (𝑡) = 3𝑡2 . For 𝑡 = 0
𝑦1 (ℎ) − 𝑦1 (0) 𝑦1 (ℎ)
we calculate 𝑦1′ (0) = limℎ→0 = limℎ→0 . To compute
ℎ−0 ℎ
this limit we show the left hand and right hand limits agree. We get
𝑦1 (ℎ) ℎ3
lim+ = lim+ = lim+ ℎ2 = 0
ℎ→0 ℎ ℎ→0 ℎ ℎ→0
𝑦1 (ℎ) 0
lim = lim =0
ℎ→0− ℎ ℎ→0+ ℎ
{
0, for 𝑡 < 0
It follows that 𝑦1′ (𝑡) = and so
3𝑡2 for 𝑡 ≥ 0
42 1 Solutions
{
0, for 𝑡 < 0
𝑡𝑦1′ (𝑡) =
3𝑡3 for 𝑡 ≥ 0
On the other hand,
{
0, for 𝑡 < 0
3𝑦1 (𝑡) =
3𝑡3 for 𝑡 ≥ 0

It follows that 𝑦1 is a solution. It is trivial to see that 𝑦2 (𝑡) is a solution.


3
There is no contraction to Theorem 5 since, in standard form 𝑦 ′ = 𝑦 =
𝑡
𝐹 (𝑡, 𝑦) has a discontinuous 𝐹 (𝑡, 𝑦) near (0, 0). So Picard’s theorem does
not even apply.

Section 2.1
1.
ℒ {3𝑡 + 1} (𝑠)
∫ ∞
= (3𝑡 + 1)𝑒−𝑠𝑡 𝑑𝑡
0
∫ ∞ ∫ ∞
=3 𝑡𝑒−𝑠𝑡 𝑑𝑡 + 𝑒−𝑠𝑡 𝑑𝑡
0 0
∞ ∞
1 ∞ −𝑠𝑡
( )
𝑡 −𝑠𝑡 −1 −𝑠𝑡

=3 𝑒 + 𝑒 𝑑𝑡 + 𝑒
−𝑠 𝑠 0 𝑠
(( ) ( 0 ) ∞ ) 0
1 −1 −𝑠𝑡 1
=3 𝑒 +
𝑠 𝑠 0 𝑠
3 1
= 2+ .
𝑠 𝑠

2.
ℒ 5𝑡 − 9𝑒𝑡 (𝑠)
{ }
∫ ∞
= 𝑒−𝑠𝑡 (5𝑡 − 9𝑒𝑡 ) 𝑑𝑡
0
∫ ∞ ∫ ∞
−𝑠𝑡
=5 𝑡𝑒 𝑑𝑡 − 9 𝑒−𝑠𝑡 𝑒𝑡 𝑑𝑡
0 0

1 ∞ −𝑠𝑡
( ) ∫ ∞
−𝑡 −𝑠𝑡

=5 𝑒 + 𝑒 𝑑𝑡 − 9 𝑒−(𝑠−1)𝑡 𝑑𝑡
𝑠 0 𝑠 0 0
( )
1 1
= 5 0+ 2 −9
𝑠 𝑠−1
5 9
= 2− .
𝑠 𝑠−1
1 Solutions 43

3.

ℒ 𝑒2𝑡 − 3𝑒−𝑡 (𝑠)


{ }
∫ ∞
= 𝑒−𝑠𝑡 (𝑒2𝑡 − 3𝑒−𝑡 ) 𝑑𝑡
0
∫ ∞ ∫ ∞
−𝑠𝑡 2𝑡
= 𝑒 𝑒 𝑑𝑡 − 3 𝑒−𝑠𝑡 𝑒−𝑡 𝑑𝑡
0 0
∫ ∞ ∫ ∞
= 𝑒−(𝑠−2)𝑡 𝑑𝑡 − 3 𝑒−(𝑠+1)𝑡 𝑑𝑡
0 0
1 3
= − .
𝑠−2 𝑠+1

4.

ℒ 𝑡𝑒−3𝑡 (𝑠)
{ }
∫ ∞
= 𝑒−𝑠𝑡 𝑡𝑒−3𝑡 𝑑𝑡
∫0 ∞
= 𝑡𝑒−(𝑠+3)𝑡 𝑑𝑡
0
∞ ∫ ∞
𝑡𝑒−(𝑠+3)𝑡 1
= + 𝑒−(𝑠+3)𝑡 𝑑𝑡
−(𝑠 + 3) 0 𝑠+3 0
1
= .
(𝑠 + 3)2

5
5. ℒ 5𝑒2𝑡 = 5ℒ 𝑒2𝑡 =
{ } { }
𝑠−2
3 3! 3 42
6. ℒ 3𝑒−7𝑡 − 7𝑡3 = 3ℒ 𝑒−7𝑡 − 7ℒ 𝑡3 =
{ } { } { }
−7 4 = −
𝑠+7 𝑠 𝑠 + 7 𝑠4
2 5 4
7. ℒ 𝑡2 − 5𝑡 + 4 = ℒ 𝑡2 − 5ℒ {𝑡} + 4ℒ {1} = 3 − 2 +
{ } { }
𝑠 𝑠 𝑠
6 2 1 1
8. ℒ 𝑡 + 𝑡2 + 𝑡 + 1 = ℒ 𝑡3 + ℒ 𝑡2 + ℒ {𝑡} + ℒ {1} = 4 + 3 + 2 +
{3 } { } { }
𝑠 𝑠 𝑠 𝑠
1 7
9. ℒ 𝑒−3𝑡 + 7𝑡𝑒−4𝑡 = ℒ 𝑒−3𝑡 + 7ℒ 𝑡𝑒−4𝑡 =
{ } { } { }
+
𝑠 + 3 (𝑠 + 4)2
2
10. ℒ 𝑡2 𝑒4𝑡 (𝑠) =
{ }
(𝑠 − 4)3
𝑠 2 𝑠+2
11. ℒ {cos 2𝑡 + sin 2𝑡} = ℒ {cos 2𝑡} + ℒ {sin 2𝑡} = + = 2
𝑠2 + 2 2 𝑠2 + 2 2 𝑠 +4
44 1 Solutions

1
12. ℒ {𝑒𝑡 (𝑡 − cos 4𝑡)} (𝑠) = ℒ {𝑡𝑒−𝑡 } (𝑠) − ℒ {𝑒−𝑡 cos 4𝑡} (𝑠) = −
(𝑠 − 1)2
𝑠−1
.
(𝑠 − 1)2 + 16
2
13. ℒ (𝑡𝑒−2𝑡 )2 (𝑠) = ℒ 𝑡2 𝑒−4𝑡 (𝑠) =
{ } { }
(𝑠 + 4)3
√ } 𝑠 + 31
14. ℒ 𝑒−𝑡/3 cos 6𝑡 (𝑠) =
{
(𝑠 + 13 )2 + 6

15. ℒ (𝑡 + 𝑒2𝑡 )2 (𝑠) = ℒ 𝑡2 + 2𝑡𝑒2𝑡 + 𝑒4𝑡 (𝑠) = ℒ 𝑡2 (𝑠)+2ℒ 𝑡𝑒2𝑡 (𝑠)+


{ } { } { } { }
2 2 1
ℒ 𝑒4𝑡 (𝑠) = 3 +
{ }
2
+
𝑠 (𝑠 − 2) 𝑠−4
16. ℒ {5 cos 3𝑡 − 3 sin 3𝑡 + 4} (𝑠) = 5ℒ {cos 3𝑡} (𝑠)−3ℒ {sin 3𝑡} (𝑠)+ℒ {4} (𝑠) =
𝑠 3 4 5𝑠 − 9 4
5 2 −3 2 + = 2 +
𝑠 + 32 𝑠 + 32 𝑠 𝑠 +9 𝑠
{ 4}
𝑡 4! 24
(𝑠) = ℒ 𝑡4 𝑒−4𝑡 (𝑠) =
{ }
17. ℒ =
𝑒4𝑡 (𝑠 + 4)5 (𝑠 + 4)5

18. ℒ 𝑒5𝑡 (8 cos 2𝑡 + 11 sin 2𝑡) (𝑠) = 8ℒ 𝑒5𝑡 cos 2𝑡 (𝑠)+11ℒ 𝑒5𝑡 sin 2𝑡 (𝑠) =
{ } { } { }

8(𝑠 − 5) 22 8𝑠 − 18
+ = 2
(𝑠 − 5)2 + 4 (𝑠 − 5)2 + 4 𝑠 − 10𝑠 + 29
( )′
{ 3𝑡 } ( { 3𝑡 })′ 3 1
19. ℒ 𝑡𝑒 (𝑠) = − ℒ 𝑒 (𝑠) = − =
𝑠−3 (𝑠 − 3)2
)′
𝑠2 − 9
(
′ 𝑠
20. ℒ {𝑡 cos 3𝑡} (𝑠) = − (ℒ {cos 3𝑡}) = − 2 = 2
𝑠 +9 (𝑠 + 9)2

21. Here we use the transform derivative principle twice to get ℒ 𝑡2 sin 2𝑡 (𝑠) =
{ }
)′′ ( )′
12𝑠2 − 16
(
2 −4𝑠
(ℒ {sin 2𝑡})′′ = = =
𝑠2 + 4 (𝑠2 + 4)2 (𝑠2 + 4)3
( )′ ( )′
−𝑡 −𝑡 ′ 𝑠 𝑠+1
22. ℒ {𝑡𝑒 cos 𝑡} (𝑠) = −ℒ {𝑒 cos 𝑡} (𝑠) = − = − =
𝑠2 + 1 𝑠→𝑠+1 𝑠2 + 2𝑠 + 2
𝑠2 + 2𝑠
(𝑠 + 2𝑠 + 2)2
2

( ( 2 ))′
′ 𝑠 2𝑠 2
23. ℒ {𝑡𝑓 (𝑡)} (𝑠) = −ℒ {𝑓 (𝑡)} (𝑠) = − ln 2
= 2 −
𝑠 +1 𝑠 +1 𝑠

(𝑠/5)2 𝑠2
{ } { }
1 − cos 5𝑡 1 − cos 5𝑡 1 1
24. ℒ = 5ℒ = ln = ln .
𝑡 5𝑡 2 (𝑠/5)2 + 1 2 𝑠2 + 25
1 Solutions 45

1 1 ln((𝑠/6) + 1) ln(𝑠 + 6) − ln 6
25. ℒ {Ei(6𝑡)} (𝑠) = ℒ {Ei(𝑡)} (𝑠)∣𝑠→𝑠/6 = =
6 6 𝑠/6 𝑠

𝑠2 + 2𝑏2
( )
1 1 1 𝑠
26. ℒ cos2 𝑏𝑡 (𝑠) = ℒ {1 + cos 2𝑏𝑡} (𝑠) =
{ }
+ 2 2
=
2 2 𝑠 𝑠 + 4𝑏 𝑠(𝑠2 + 4𝑏2 )
1 1
27. We use the identity sin2 𝜃 = (1−cos 2𝜃). ℒ sin2 𝑏𝑡 (𝑠) = ℒ {1 − cos 2𝑏𝑡} (𝑠) =
{ }
2 2
2𝑏2
( )
1 1 𝑠
− 2 2
= ;
2 𝑠 𝑠 + 4𝑏 𝑠(𝑠 + 4𝑏2 )
2

1
28. We use the identity sin 2𝜃 = 2 sin 𝜃 cos 𝜃. ℒ {sin 𝑏𝑡 cos 𝑏𝑡} (𝑠) = ℒ {sin 2𝑏𝑡} (𝑠) =
2
1 2𝑏 𝑏
= 2 ;
2 𝑠2 + 4𝑏2 𝑠 + 4𝑏2
1
29. We use the identity sin 𝑎𝑡 cos 𝑏𝑡 = (sin(𝑎 + 𝑏)𝑡 + sin(𝑎 − 𝑏)𝑡).
2
1
ℒ {sin 𝑎𝑡 cos 𝑏𝑡} = (ℒ {sin(𝑎 + 𝑏)𝑡} + ℒ {sin(𝑎 − 𝑏)𝑡})
2( )
1 𝑎−𝑏 𝑎+𝑏
= + .
2 𝑠2 + (𝑎 − 𝑏)2 𝑠2 + (𝑎 + 𝑏)2

( )
1 ( { 𝑏𝑡 }) 1 1 1 𝑠
30. ℒ {cosh 𝑏𝑡} = ℒ 𝑒 + 𝑒−𝑏𝑡 = + =
2 2 𝑠+𝑏 𝑠−𝑏 𝑠2 − 𝑏2
( )
1 ( { 𝑏𝑡 }) 1 1 1 𝑏
31. ℒ {sinh 𝑏𝑡} = ℒ 𝑒 − 𝑒−𝑏𝑡 = − =
2 2 𝑠+𝑏 𝑠−𝑏 𝑠2 − 𝑏 2

32. Let 𝑓 (𝑡) = 𝑒𝑎𝑡 . Then 𝑓 ′ (𝑡) = 𝑎𝑒𝑎𝑡 and 𝑓 (𝑡)∣𝑡=0 = 1. Thus 𝑎ℒ {𝑒𝑎𝑡 } =
1
ℒ {𝑓 ′ (𝑡)} = 𝑠ℒ {𝑒𝑎𝑡 } − 1. Solving for ℒ {𝑒𝑎𝑡 } gives ℒ {𝑒𝑎𝑡 } = .
𝑠−𝑎
33. Let 𝑓 (𝑡) = sinh 𝑏𝑡. Then 𝑓 ′ (𝑡) = 𝑏 cosh 𝑡 and 𝑓 ′′ (𝑡) = 𝑏2 sinh 𝑡. Fur-
ther, 𝑓 (𝑡)∣𝑡=0 = 0 and 𝑓 ′ (𝑡)∣𝑡=0 = 𝑏. Thus 𝑏2 ℒ {sinh 𝑏𝑡} = ℒ {𝑓 ′′ (𝑡)} =
𝑠2 ℒ {𝑓 (𝑡)} − 𝑠𝑓 (0) − 𝑓 ′ (0) = 𝑠2 ℒ {𝑓 (𝑡)} − 𝑏. Solving for ℒ {𝑓 (𝑡)} gives
𝑏
ℒ {sinh 𝑏𝑡} = 2 .
𝑠 − 𝑏2
34. Let 𝑓 (𝑡) = cosh 𝑏𝑡. Then 𝑓 ′ (𝑡) = 𝑏 sinh 𝑡 and 𝑓 ′′ (𝑡) = 𝑏2 cosh 𝑡. Fur-
ther, 𝑓 (𝑡)∣𝑡=0 = 1 and 𝑓 ′ (𝑡)∣𝑡=0 = 0. Thus 𝑏2 ℒ {cosh 𝑏𝑡} = ℒ {𝑓 ′′ (𝑡)} =
𝑠2 ℒ {𝑓 (𝑡)} − 𝑠𝑓 (0) − 𝑓 ′ (0) = 𝑠2 ℒ {𝑓 (𝑡)} − 𝑠. Solving for ℒ {𝑓 (𝑡)} gives
𝑠
ℒ {cosh 𝑏𝑡} = 2 .
𝑠 − 𝑏2
∫𝑡 ∫0
35. Let 𝑔(𝑡) = 0 𝑓 (𝑢) 𝑑𝑢 and note that 𝑔 ′ (𝑡) = 𝑓 (𝑡) and 𝑔(0) = 0 𝑓 (𝑢) 𝑑𝑢 =
0. Now apply the input derivative formula to 𝑔(𝑡), to get
46 1 Solutions

𝐹 (𝑠) = ℒ {𝑓 (𝑡)} (𝑠) = ℒ {𝑔 ′ (𝑡)} (𝑠) = 𝑠ℒ {𝑔(𝑡)} (𝑠) − 𝑔(0) = 𝑠𝐺(𝑠).

Solving for 𝐺(𝑠) gives 𝐺(𝑠) = 𝐹 (𝑠)/𝑠.

36. For 𝑡 ≥ 0, 𝑒𝑎𝑡 ≤ 𝑒𝑏𝑡 . Thus ∣𝑓 (𝑡)∣ ≤ 𝐾𝑒𝑎𝑡 ≤ 𝐾𝑒𝑏𝑡 . So 𝑓 is of exponential


type of order 𝑏.

37. Suppose 𝑓 is on exponential type of order 𝑎 and 𝑔 is of exponential type


of order 𝑏. Suppose 𝑎 ≤ 𝑏. Then there are numbers 𝐾 and 𝐿 so that
∣𝑓 (𝑡)∣ ≤ 𝐾𝑒𝑎𝑡 and ∣𝑔(𝑡)∣ ≤ 𝐿𝑒𝑏𝑡 . Now ∣𝑓 (𝑡) + 𝑔(𝑡)∣ ≤ ∣𝑓 (𝑡)∣ + ∣𝑔(𝑡)∣ ≤
𝐾𝑒𝑎𝑡 + 𝐿𝑒𝑏𝑡 ≤ (𝐾 + 𝐿)𝑒𝑏𝑡 . If follows that 𝑓 + 𝑔 is of exponential type of
order 𝑏.

38. Suppose 𝑓 is of exponential type of order 𝑎 and 𝑔 is of exponential type


of order 𝑏. Then there are numbers 𝐾 and 𝐿 so that ∣𝑓 (𝑡)∣ ≤ 𝐾𝑒𝑎𝑡 and
∣𝑔(𝑡)∣ ≤ 𝐿𝑒𝑏𝑡 . Now ∣𝑓 (𝑡)𝑔(𝑡)∣ ≤ 𝐾𝑒𝑎𝑡 𝐿𝑒𝑏𝑡 = 𝐾𝐿𝑒(𝑎+𝑏)𝑡 . If follows that
𝑓 + 𝑔 is of exponential type of order 𝑎 + 𝑏.

39. If 𝑓 is bounded by 𝐾, say, then 𝑓 (𝑡) ≤ 𝐾𝑒0𝑡 . So 𝑓 is of exponential type


of order 0.

40. Suppose 𝑓 is of exponential type of order 𝑎 in the sense given in the text.
Then 𝑁 can be chosen to be 0 and 𝑓 satisfies the definition given in the
statement of the problem. Now suppose 𝑓 satisfies the definition given in
the statement of the problem. I.e. there is a 𝐾 ≥ 0 and 𝑁 ≥ 0 so that
∣𝑓 ∣ ≤ 𝐾𝑒𝑎𝑡 for 𝑡 ≥ 𝑁 . Since ∣𝑓 ∣ is continuous on the interval [0, 𝑁 ] it has a
maximum, 𝐾1 , say. It follows that ∣𝑓 ∣ ≤ 𝐾1 ≤ 𝐾1 𝑒𝑎𝑡 on [0, 𝑁 ] and hence
∣𝑓 ∣ ≤ (𝐾 + 𝐾1 )𝑒𝑎𝑡 , for all 𝑡 ≥ 0. It follows that 𝑓 is of exponential type in
the sense given in the text.

41. Suppose 𝑎 and 𝐾 are real and ∣𝑦(𝑡)∣ ≤ 𝐾𝑒𝑎𝑡 . Then 𝑦(𝑡)𝑒−𝑎𝑡 is bounded
by 𝐾. But
2 2 2 −𝑎2
−𝑎𝑡+ 𝑎4
𝑒𝑡 𝑒−𝑎𝑡 = 𝑒𝑡 𝑒 4

𝑎 2 −𝑎2
= 𝑒(𝑡− 2 ) 𝑒 4

2 −𝑎2
= 𝑒𝑢 𝑒 4 ,
2
where 𝑢 = 𝑡− 𝑎2 . As 𝑡 approaches infinity so does 𝑢. Since lim𝑢→∞ 𝑒𝑢 = ∞
2
it is clear that lim𝑡→∞ 𝑒𝑡 𝑒−𝑎𝑡 = ∞, for all 𝑎 ∈ ℝ, and hence 𝑦(𝑡)𝑒−𝑎𝑡 is
not bounded. It follows that 𝑦(𝑡) is not of exponential type.
2 ∫∞ 2
42. First of all, fix 𝑎 ∈ ℝ. Since 𝑒𝑡 > 1 for all 𝑡 it follows that 𝑎 𝑒𝑡 𝑑𝑡 >
∫∞ ∫ ∞ 𝑡2
𝑎 1 𝑑𝑡 = ∞ by the comparison test. Thus 𝑎 𝑒 𝑑𝑡 does not exist for
any real number 𝑎. Now let 𝑠 be any real number. Then
1 Solutions 47
{ } ∫ ∞
2 2
ℒ 𝑒𝑡 (2𝑠) = 𝑒−2𝑠𝑡 𝑒𝑡 𝑑𝑡
∫0 ∞
2 2 2
= 𝑒𝑡 −2𝑠𝑡+𝑠 −𝑠 𝑑𝑡
0
∫ ∞
−𝑠2 2
=𝑒 𝑒(𝑡−𝑠) 𝑑𝑡
∫0 ∞
−𝑠2 2
=𝑒 𝑒𝑡 𝑑𝑡.
−𝑠

But this last integral does not exist. Since the Laplace transform does not
exist at 2𝑠, for any 𝑠, the Laplace transform does not exist.

43. 𝑦(𝑡) is of exponential type because it is continuous and bounded. On the


2 2
other hand, 𝑦 ′ (𝑡) = cos(𝑒𝑡 )𝑒𝑡 (2𝑡). Suppose there is a 𝐾 and 𝑎 so that
∣𝑦 ′ (𝑡)∣ ≤ 𝐾𝑒𝑎𝑡 for all 𝑡 ≥ 0. We need only show that there are some 𝑡 for
2
which this inequality does not hold. Since cos 𝑒𝑡 oscillates between −1
2 2
and 1 let’s focus on those 𝑡 for which
√ cos 𝑒𝑡 = 1. This happens when 𝑒𝑡
is a multiple of 2𝜋𝑛 or 𝑡 = 𝑡𝑛 = ln(2𝜋𝑛). If the inequality ∣𝑦 ′ (𝑡)∣ ≤ 𝐾𝑒𝑎𝑡
is valid for all 𝑡 ≥ 0 it is valid for 𝑡𝑛 for all 𝑛 > 0. We then get the
2
inequality 2𝑡𝑛 𝑒𝑡𝑛 ≤ 𝐾𝑒𝑎𝑡𝑛 . Now divide by 𝑒𝑎𝑡𝑛 , combine, complete the
2 2
square, and simplify to get the inequality 2𝑡𝑛 𝑒(𝑡𝑛 −𝑎/2) ≤ 𝐾𝑒𝑎 /4 . Choose
𝑛 so that 𝑡𝑛 > 𝐾 and 𝑡𝑛 > 𝑎. Then this last inequality is not satisfied.
It follows that 𝑦 ′ (𝑡) is not of exponential type. Now consider the definite
∫𝑀
integral 0 𝑒−𝑠𝑡 𝑦 ′ (𝑡) 𝑑𝑡 and compute by parts: We get
∫ 𝑀 𝑀
∫ 𝑀
𝑒−𝑠𝑡 𝑦 ′ (𝑡) 𝑑𝑡 = 𝑦(𝑡)𝑒−𝑠𝑡 0 + 𝑠 𝑒−𝑠𝑡 𝑦(𝑡) 𝑑𝑡.
0 0

2
Since 𝑦(𝑡) = sin(𝑒𝑡 ) is bounded and 𝑦(0) = 0 it follows that
𝑀
lim 𝑦(𝑡)𝑒−𝑠𝑡 0 = 0.
𝑀 →∞

Taking limits as 𝑀 → ∞ in the equation above gives ℒ {𝑦 ′ (𝑡)} =


𝑠ℒ {𝑦(𝑡)}. The righthand side exists because 𝑦(𝑡) is bounded.

(a) Show that 𝛤 (1) = 1.


(b) Show that 𝛤 satisfies the recursion formula 𝛤 (𝛽 + 1) = 𝛽𝛤 (𝛽).
(Hint: Integrate by parts.)
(c) Show that 𝛤 (𝑛 + 1) = 𝑛! when 𝑛 is a nonnegative integer.
∫∞ ∞
44. (a) 𝛤 (1) = 0 𝑒−𝑥 𝑑𝑥 = −𝑒−𝑥 ∣0 = 1.
∫ ∞ 𝛽 −𝑥 ∞ ∫∞
(b) 𝛤 (𝛽 + 1) = 0 𝑥 𝑒 𝑑𝑥 = −𝑥𝛽 𝑒−𝑥 0 + 𝛽 0 𝑥𝛽−1 𝑒−𝑥 𝑑𝑥 = 𝛽𝛤 (𝛽).
The second equality is obtained by integration by parts using 𝑢 = 𝑥𝛽 ,
𝑑𝑣 = 𝑒−𝑥 𝑑𝑥.
(c) Repeated use of (b) gives
48 1 Solutions

𝛤 (𝑛+1) = 𝑛𝛤 (𝑛) = 𝑛(𝑛−1)𝛤 (𝑛−1) = ⋅ ⋅ ⋅ = 𝑛(𝑛−1) ⋅ ⋅ ⋅ 2⋅1𝛤 (1) = 𝑛!.

45. Using polar coordinates 𝑥 = 𝑟 cos 𝜃, 𝑦 = 𝑟 sin 𝜃. Then 𝑑𝑥 𝑑𝑦 = 𝑟 𝑑𝑟 𝑑𝜃 and


the domain of integration is the first quadrant of the plane, which in polar
coordinates is given by 0 ≤ 𝜃 ≤ 𝜋/2, 0 ≤ 𝑟 < ∞. Thus
∫ ∞∫ ∞ ∫ 𝜋/2 ∫ ∞
2 2 2
𝑒−(𝑥 +𝑦 ) 𝑑𝑥 𝑑𝑦 = 𝑒−𝑟 𝑟 𝑑𝑟 𝑑𝜃
0 0 0 0
𝜋 ∞ −𝑟2

= 𝑒 𝑟 𝑑𝑟
2 0
2 ∞

𝜋 −𝑒−𝑟 𝜋
= = .
2 2 4
0

Hence, 𝐼 = 𝜋/2.
∫∞
46. 𝛤 ( 21 ) = 0 𝑥−1/2 𝑒−𝑥 𝑑𝑥. Using the change of variables 𝑥 = 𝑢2 , so 𝑑𝑥 =
2𝑢 𝑑𝑢 it follows that
∫ ∞ ∫ ∞
1 2 2 √
𝛤( ) = 𝑢−1 𝑒−𝑢 2𝑢 𝑑𝑢 = 2 𝑒−𝑢 𝑑𝑢 = 𝜋.
2 0 0

Then using the recursion formula√gives: √


(a) 𝛤 ( 32 ) = 𝛤 ( 21 + 1) = 12 𝛤 ( 21 ) = 𝜋/2, (b) 𝛤 ( 52 ) = 23 𝛤 ( 32 ) = 3 𝜋/4.
Now apply the general power formula (Formula 11) to get
√ √
{√ } 𝛤 ( 23 ) 𝜋 } 𝛤 ( 52 ) 3 𝜋
𝑡 = 3/2 = 3/2 , (d) ℒ 𝑡3/2 = 5/2
{
(c) ℒ = 5/2 .
𝑠 2𝑠 𝑠 4𝑠

Section 2.2

The 𝑠 − 1 -chain
5𝑠 + 10 3
1. (𝑠 − 1)(𝑠 + 4) 𝑠−1
2
𝑠+4

The 𝑠 − 2 -chain
10𝑠 − 2 6
2. (𝑠 + 1)(𝑠 − 2) (𝑠 − 2)
4
(𝑠 + 1)
1 Solutions 49

The 𝑠 − 5 -chain
1 1/7
3. (𝑠 + 2)(𝑠 − 5) (𝑠 − 5)
−1/7
(𝑠 + 2)

The 𝑠 + 3 -chain
5𝑠 + 9 3/2
4. (𝑠 − 1)(𝑠 + 3) 𝑠+3
7/2
𝑠−1

The 𝑠 − 1 -chain
3𝑠 + 1 2
5. (𝑠 − 1)(𝑠2 + 1) 𝑠−1
−2𝑠 + 1
𝑠2 + 1

The 𝑠 + 1 -chain
3𝑠2 − 𝑠 + 6 2
6. (𝑠 + 1)(𝑠2 + 4) (𝑠 + 1)
𝑠−2
𝑠2 + 4

The 𝑠 + 3 -chain
2
𝑠 +𝑠−3 3
(𝑠 + 3)3 (𝑠 + 3)3
𝑠−2 −5
7. (𝑠 + 3)2 (𝑠 + 3)2
1 1
𝑠+3 𝑠+3
0
50 1 Solutions

The 𝑠 + 2 -chain
2
5𝑠 − 3𝑠 + 10 −36
(𝑠 + 1)(𝑠 + 2)2 (𝑠 + 2)2
8. 5𝑠 + 23 −13
(𝑠 + 1)(𝑠 + 2) 𝑠+2
18
𝑠+1

The 𝑠 + 1 -chain
𝑠 −1
(𝑠 + 2)2 (𝑠
+ 1)2 (𝑠 + 1)2
9. 𝑠+4 3
(𝑠 + 2)2 (𝑠 + 1) 𝑠+1
−3𝑠 − 8
(𝑠 + 2)2

The 𝑠 − 1 -chain
16𝑠 4
(𝑠 − 1)3 (𝑠 − 3)2 (𝑠 − 1)3
−4𝑠 + 36 8
10. (𝑠 − 1)2 (𝑠 − 3)2 (𝑠 − 1)2
−8𝑠 + 36 7
(𝑠 − 1)(𝑠 − 3)2 (𝑠 − 1)
−7𝑠 + 27
(𝑠 − 3)2
1 Solutions 51

The 𝑠 − 5 -chain
1 −1
(𝑠 − 5)5 (𝑠 − 6) (𝑠 − 5)5
1 −1
(𝑠 − 5)4 (𝑠 − 6) (𝑠 − 5)4
1 −1
11. (𝑠 − 5)3 (𝑠 − 6) (𝑠 − 5)3
1 −1
(𝑠 − 5)2 (𝑠 − 6) (𝑠 − 5)2
1 −1
(𝑠 − 5)(𝑠 − 6) 𝑠−5
1
𝑠−6

12. Use the technique of distinct linear factors (Example 5).


3/2 7/2
+
𝑠+3 𝑠−1
13. Use the technique of distinct linear factors (Example 5).
13/8 5/8

𝑠−5 𝑠+3
−1 1
14. +
𝑠−1 𝑠−2
23 37
15. +
12(𝑠 − 5) 12(𝑠 + 7)
1 2
16. +
𝑠 𝑠+1
25 9
17. −
8(𝑠 − 7) 8(𝑠 + 1)
25 9 15
18. + −
2(𝑠 − 3) 2(𝑠 − 1) 𝑠 − 2
1 1 1
19. − +
2(𝑠 + 5) 2(𝑠 − 1) 𝑠 − 2
20. Use Theorem 1 to write
52 1 Solutions

𝑠2 𝐴1 𝑝1 (𝑠)
= +
(𝑠 − 1)3 (𝑠 − 1)3 (𝑠 − 1)2

𝑠2

where 𝐴1 = =1
1 𝑠=1
1 1
and 𝑝1 (𝑠) = (𝑠2 − (1)(1)) = (𝑠2 − 1) = 𝑠 + 1
𝑠−1 𝑠−1
Continuing gives
𝑠+1 𝐴2 𝑝2 (𝑠)
= +
(𝑠 − 1)2 (𝑠 − 1)2 𝑠−1

𝑠 + 1
where 𝐴2 = =2
1 𝑠=1
1 1
and 𝑝2 (𝑠) = (𝑠 + 1 − (2)(1)) = (𝑠 − 1) = 1
𝑠−1 𝑠−1
𝑠2 1 2 1
Thus = + + .
(𝑠 − 1)3 (𝑠 − 1)3 (𝑠 − 1)2 𝑠−1
Alternate Solution: Write 𝑠 = (𝑠 − 1) + 1 so that

𝑠2 ((𝑠 − 1) + 1)2 1 + 2(𝑠 − 1) + (𝑠 − 1)2 1 2 1


3
= 3
= 3
= 3
+ 2
+
(𝑠 − 1) (𝑠 − 1) (𝑠 − 1) (𝑠 − 1) (𝑠 − 1) 𝑠 − 1

7
21.
(𝑠 + 4)4
22. Use Theorem 1 to write
𝑠 𝐴1 𝑝1 (𝑠)
= +
(𝑠 − 3)3 (𝑠 − 3)3 (𝑠 − 3)2

𝑠
where 𝐴1 = =3
1 𝑠=3

1
and 𝑝1 (𝑠) = (𝑠 − (3)(1)) = 1.
𝑠−3
𝑠 3 1
Thus, = +
(𝑠 − 3)3 (𝑠 − 3)3 (𝑠 − 3)2
Alternate Solution: Write 𝑠 = (𝑠 − 3) + 3 so that
𝑠 (𝑠 − 3) + 3 3 1
= = + .
(𝑠 − 3)3 (𝑠 − 3)3 (𝑠 − 3)3 (𝑠 − 3)2
1 Solutions 53

23. Use Theorem 1 to write


𝑠2 + 𝑠 − 3 𝐴1 𝑝1 (𝑠)
= +
(𝑠 + 3)3 (𝑠 + 3)3 (𝑠 + 3)2

𝑠2 + 𝑠 − 3

where 𝐴1 = =3
1
𝑠=−3
1 1
and 𝑝1 (𝑠) = (𝑠2 + 𝑠 − 3 − (3)(1)) = (𝑠2 + 𝑠 − 6) = 𝑠 − 2
𝑠+3 𝑠+3

Continuing gives
𝑠−2 𝐴2 𝑝2 (𝑠)
= +
(𝑠 + 3)2 (𝑠 + 3)2 𝑠+3

𝑠 − 2
where 𝐴2 = = −5
1 𝑠=−3
1 1
and 𝑝2 (𝑠) = (𝑠 − 2 − (−5)(1)) = (𝑠 + 3) = 1
𝑠+3 𝑠+3
𝑠2 + 𝑠 − 3 3 5 1
Thus = − +
(𝑠 + 3)3 (𝑠 + 3)3 (𝑠 + 3)2 𝑠+3
Alternate Solution: Write 𝑠 = (𝑠 + 3) − 3 so that

𝑠2 + 𝑠 − 3 ((𝑠 + 3) − 3)2 + ((𝑠 + 3) − 3) − 3


3
=
(𝑠 + 3) (𝑠 − 3)3
(𝑠 + 3)2 − 5(𝑠 + 3) + 3
=
(𝑠 + 3)3
3 5 1
= 3
− 2
+ .
(𝑠 + 3) (𝑠 + 3) 𝑠+3

24. Use Theorem 1 to compute the (𝑠 + 2)-chain:

5𝑠2 − 3𝑠 + 10 𝐴1 𝑝1 (𝑠)
= +
(𝑠 + 1)(𝑠 + 2)2 (𝑠 + 2)2 (𝑠 + 2)(𝑠 + 1)

5𝑠2 − 3𝑠 + 10

where 𝐴1 = = −36
𝑠+1
𝑠=−2
1 1
and 𝑝1 (𝑠) = (5𝑠2 − 3𝑠 + 10 − (−36)(𝑠 + 1)) = (5𝑠2 + 33𝑠 + 46) = 5𝑠 + 23
𝑠+2 𝑠+2
Continuing gives
54 1 Solutions

5𝑠 + 23 𝐴2 𝑝2 (𝑠)
= +
(𝑠 + 2)(𝑠 + 1) 𝑠+2 𝑠+1

5𝑠 + 23
where 𝐴2 = = −13
𝑠 + 1 𝑠=−2
1 1
and 𝑝2 (𝑠) = (5𝑠 + 23 − (−13)(𝑠 + 1)) = (18𝑠 + 36) = 18
𝑠+2 𝑠+2
5𝑠2 − 3𝑠 + 10 −36 13 18
Thus = − +
(𝑠 + 1)(𝑠 + 2)2 (𝑠 + 2)2 𝑠+2 𝑠+1

𝑠2 − 6𝑠 + 7 𝑠2 − 6𝑠 + 7
25. = , so use Theorem 1 to compute the
(𝑠2 − 4𝑠 − 5)2 (𝑠 + 1)2 (𝑠 − 5)2
(𝑠 + 1)-chain:

𝑠2 − 6𝑠 + 7 𝐴1 𝑝1 (𝑠)
= +
(𝑠 + 1)2 (𝑠 − 5)2 (𝑠 + 1)2 (𝑠 + 1)(𝑠 − 5)2

𝑠2 − 6𝑠 + 7

7
where 𝐴1 = =
(𝑠 − 5)2 𝑠=−1 18
1
and 𝑝1 (𝑠) = (𝑠2 − 6𝑠 + 7 − (7/18)(𝑠 − 5)2 )
𝑠+1
1 1
= (11𝑠2 − 38𝑠 − 49)/18 = (11𝑠 − 49)
𝑠+1 18
Continuing gives

1 11𝑠 − 49 𝐴2 𝑝2 (𝑠)
2
= +
18 (𝑠 + 1)(𝑠 − 5) 𝑠+1 (𝑠 − 5)2

1 11𝑠 − 49
where 𝐴2 = = −5/54
18 (𝑠 − 5)2 𝑠=−1
1
and 𝑝2 (𝑠) = ((11𝑠 − 49)/18 − (−5/54)(𝑠 − 5)2 ) = (5𝑠 − 22)/54
𝑠+1
𝑠2 − 6𝑠 + 7 1/18 5/54 (5𝑠 − 22)/54
Thus = − + Now either con-
(𝑠 + 1)2 (𝑠 − 5)2 (𝑠 + 1)2 𝑠 + 1 (𝑠 − 5)2
tinue with Theorem 1 or replace 𝑠 with 𝑠 = (𝑠 − 5) + 5 in the numerator
𝑠2 − 6𝑠 + 7
of the last fraction to finish the calculation and get =
( ) (𝑠 + 1)2 (𝑠 − 5)2
1 5 21 3 5
+ 2
+ 2

54 𝑠 − 5 (𝑠 + 1) (𝑠 − 5) 𝑠+1
26. Use Theorem 1 to compute the (𝑠 + 9)-chain:
1 Solutions 55

81 𝐴1 𝑝1 (𝑠)
= +
𝑠3 (𝑠 + 9) 𝑠+9 𝑠3

81
where 𝐴1 = = −1/9
𝑠3 𝑠=−9
1 1
and 𝑝1 (𝑠) = (81 − (−1/9)(𝑠3 )) = (𝑠3 + 93 )/9 = (𝑠2 − 9𝑠 + 81)/9
𝑠+9 𝑠+9
81 9 1 1 1 1
Thus = 3− 2+ −
𝑠3 (𝑠+ 9) 𝑠 𝑠 9𝑠 9 𝑠 + 1
27. Use Theorem 1 to compute the (𝑠 + 2)-chain:

𝑠 𝐴1 𝑝1 (𝑠)
= +
(𝑠 + 2)2 (𝑠 + 1)2 (𝑠 + 2)2 (𝑠 + 2)(𝑠 + 1)2

𝑠
where 𝐴1 = 2
= −2
(𝑠 + 1) 𝑠=−2
1
and 𝑝1 (𝑠) = (𝑠 − (−2)(𝑠 + 1)2 )
𝑠+2
2𝑠2 + 5𝑠 + 2 (2𝑠 + 1)(𝑠 + 1)
= = = 2𝑠 + 1
𝑠+2 𝑠+2
Continuing gives

2𝑠 + 1 𝐴2 𝑝2 (𝑠)
= +
(𝑠 + 2)(𝑠 + 1)2 𝑠+2 (𝑠 + 1)2

2𝑠 + 1
where 𝐴2 = = −3
(𝑠 + 1)2 𝑠=−2
1
and 𝑝2 (𝑠) = (2𝑠 + 1 − (−3)(𝑠 + 1)2 ) = 3𝑠 + 2
𝑠+2
𝑠 −2 3 3𝑠 + 2
Thus = − + . Now continue using
(𝑠 + 2)2 (𝑠 + 1)2 (𝑠 + 2)2 𝑠 + 2 (𝑠 + 1)2
Theorem 1 or replace 𝑠 by (𝑠 + 1) − 1 in the numerator of the last fraction
𝑠 −2 3 1 3
to get = − − +
(𝑠 + 2)2 (𝑠 + 1)2 (𝑠 + 2)2 𝑠 + 2 (𝑠 + 1)2 𝑠+1
28. Use Theorem 1 to compute the (𝑠 + 2)-chain:
56 1 Solutions

𝑠2 𝐴1 𝑝1 (𝑠)
= +
(𝑠 + 2)2 (𝑠 + 1)2 (𝑠 + 2)2 (𝑠 + 2)(𝑠 + 1)2

𝑠2

where 𝐴1 = =4
(𝑠 + 1)2 𝑠=−2
1
and 𝑝1 (𝑠) = (𝑠2 − (4)(𝑠 + 1)2 )
𝑠+2
−3𝑠2 − 8𝑠 − 4 −(3𝑠 + 2)(𝑠 + 2)
= = = −(3𝑠 + 2)
𝑠+2 𝑠+2
Continuing gives

−3𝑠 − 2 𝐴2 𝑝2 (𝑠)
= +
(𝑠 + 2)(𝑠 + 1)2 𝑠+2 (𝑠 + 1)2

−3𝑠 − 2
where 𝐴2 = =4
(𝑠 + 1)2 𝑠=−2
1
and 𝑝2 (𝑠) = (−3𝑠 − 2 − (4)(𝑠 + 1)2 ) = −(4𝑠 + 3)
𝑠+2
𝑠2 4 −4𝑠 − 3
Thus = + + . Now continue using
(𝑠 + 2)2 (𝑠 + 1)2 (𝑠 + 2)2 𝑠 + 2 (𝑠 + 1)2
Theorem 1 or replace 𝑠 by (𝑠 + 1) − 1 in the numerator of the last fraction
𝑠2 4 4 1 4
to get = + + −
(𝑠 + 2)2 (𝑠 + 1)2 (𝑠 + 2)2 𝑠 + 2 (𝑠 + 1)2 𝑠+1
29. Use Theorem 1 to compute the (𝑠 − 3)-chain:

8𝑠 𝐴1 𝑝1 (𝑠)
= +
(𝑠 − 1)(𝑠 − 2)(𝑠 − 3)3 (𝑠 − 3)3 (𝑠 − 1)(𝑠 − 2)(𝑠 − 3)2

8𝑠
where 𝐴1 = = 12
(𝑠 − 1)(𝑠 − 2) 𝑠=3
1
and 𝑝1 (𝑠) = (8𝑠 − (12)(𝑠 − 1)(𝑠 − 2))
𝑠−3
−12𝑠2 + 44𝑠 − 24 (−12𝑠 + 8)(𝑠 − 3)
= = = −12𝑠 + 8
𝑠−3 𝑠−3
For the second step in the (𝑠 − 3)-chain:
1 Solutions 57

−12𝑠 + 8 𝐴2 𝑝2 (𝑠)
2
= +
(𝑠 − 1)(𝑠 − 2)(𝑠 − 3) (𝑠 − 3)2 (𝑠 − 1)(𝑠 − 2)(𝑠 − 3)2

−12𝑠 + 8
where 𝐴2 = = −14
(𝑠 − 1)(𝑠 − 2) 𝑠=3
1
and 𝑝2 (𝑠) = (−12𝑠 + 8 − (−14)(𝑠 − 1)(𝑠 − 2))
𝑠−3
14𝑠2 − 54𝑠 + 36 (14𝑠 − 12)(𝑠 − 3)
= = = 14𝑠 − 12
𝑠−3 𝑠−3
Continuing gives
14𝑠 − 12 𝐴3 𝑝3 (𝑠)
= +
(𝑠 − 1)(𝑠 − 2)(𝑠 − 3)2 𝑠−3 (𝑠 − 1)(𝑠 − 2)

14𝑠 − 12
where 𝐴3 = = 15
(𝑠 − 1)(𝑠 − 2) 𝑠=3
1
and 𝑝3 (𝑠) = (14𝑠 − 12 − (15)(𝑠 − 1)(𝑠 − 2)) = −15𝑠 + 14
𝑠−3
8𝑠 12 14 15 −15𝑠 + 14
Thus = − + + .
(𝑠 − 1)(𝑠 − 3)(𝑠 − 3)3 (𝑠 − 3)3 (𝑠 − 3)2 𝑠 − 3 (𝑠 − 1)(𝑠 − 2)
The last fraction has a denominator with distinct linear factors so we get
8𝑠 12 −14 15 −16 1
3
= 3
+ 2
+ + +
(𝑠 − 1)(𝑠 − 3)(𝑠 − 3) (𝑠 − 3) (𝑠 − 3) 𝑠−3 𝑠−2 𝑠−1
30. Use Theorem 1 to compute the 𝑠-chain:
25 𝐴1 𝑝1 (𝑠)
= 2 +
𝑠2 (𝑠 − 5)(𝑠 + 1) 𝑠 𝑠(𝑠 − 5)(𝑠 + 1)

25
where 𝐴1 = = −5
(𝑠 − 5)(𝑠 + 1) 𝑠=0
1
and 𝑝1 (𝑠) = (25 − (−5)(𝑠 − 5)(𝑠 + 1))
𝑠
5𝑠2 − 20𝑠
= = 5𝑠 − 20
𝑠
Continuing gives
5𝑠 − 20 𝐴2 𝑝2 (𝑠)
= +
𝑠(𝑠 − 5)(𝑠 + 1) 𝑠 (𝑠 − 5)(𝑠 + 1)

5𝑠 − 20
where 𝐴2 = =4
(𝑠 − 5)(𝑠 + 1) 𝑠=0
1
and 𝑝2 (𝑠) = (5𝑠 − 20 − (4)(𝑠 − 5)(𝑠 + 1)) = −4𝑠 + 16
𝑠
58 1 Solutions

25 5 4 −4𝑠 + 16
Thus = 2 − + . The last fraction has
𝑠2 (𝑠 − 5)(𝑠 + 1) 𝑠 𝑠 (𝑠 − 5)(𝑠 + 1)
25
a denominator with distinct linear factors so we get 2 =
𝑠 (𝑠 − 5)(𝑠 + 1)
2 1 20 1 5 4
− − − +
3𝑠−5 3 𝑠 + 1 𝑠2 𝑠
31. Use Theorem 1 to compute the (𝑠 − 2)-chain:
𝑠 𝐴1 𝑝1 (𝑠)
= +
(𝑠 − 2)2 (𝑠 − 3)2 (𝑠 − 2)2 (𝑠 − 2)(𝑠 − 3)2

𝑠
where 𝐴1 = 2
=2
(𝑠 − 3) 𝑠=2
1
and 𝑝1 (𝑠) = (𝑠 − (2)(𝑠 − 3)2 )
𝑠−2
−2𝑠2 + 13𝑠 − 18 (−2𝑠 + 9)(𝑠 − 2)
= = = −2𝑠 + 9
𝑠−2 𝑠−2
Continuing gives
−2𝑠 + 9 𝐴2 𝑝2 (𝑠)
= +
(𝑠 − 2)(𝑠 − 3)2 𝑠−3 (𝑠 − 3)2

−2𝑠 + 9
where 𝐴2 = =5
(𝑠 − 3)2 𝑠=2
1
and 𝑝2 (𝑠) = (−2𝑠 + 9 − (5)(𝑠 − 3)2 ) = −5𝑠 + 18
𝑠−2
𝑠 2 5 −5𝑠 + 18
Thus = + + . Now continue using
(𝑠 −2)2 (𝑠 −3)2 (𝑠 − 2) 2 𝑠 − 2 (𝑠 − 3)2
Theorem 1 or replace 𝑠 by (𝑠 − 3) + 3 in the numerator of the last fraction
𝑠 2 5 3 5
to get = + + −
(𝑠 − 2)2 (𝑠 − 3)2 (𝑠 − 2)2 𝑠 − 2 (𝑠 − 3)2 𝑠−3
32. Use Theorem 1 to compute the (𝑠 − 1)-chain:
16𝑠 𝐴1 𝑝1 (𝑠)
= +
(𝑠 − 1)3 (𝑠 − 3)2 (𝑠 − 1)3 (𝑠 − 1)2 (𝑠 − 3)2

16𝑠
where 𝐴1 = =4
(𝑠 − 3)2 𝑠=1
1
and 𝑝1 (𝑠) = (16𝑠 − (4)(𝑠 − 3)2 )
𝑠−1
−4𝑠2 + 40𝑠 − 36 (−4𝑠 + 36)(𝑠 − 1)
= = = −4𝑠 + 36
𝑠−1 𝑠−1
1 Solutions 59

For the second step in the (𝑠 − 1)-chain:

−4𝑠 + 36 𝐴2 𝑝2 (𝑠)
2 2
= +
(𝑠 − 1) (𝑠 − 3) (𝑠 − 1)2 (𝑠 − 1)(𝑠 − 3)2

−4𝑠 + 36
where 𝐴2 = =8
(𝑠 − 3)2 𝑠=1
1
and 𝑝2 (𝑠) = (−4𝑠 + 36 − (8)(𝑠 − 3)2 )
𝑠−1
−8𝑠2 + 44𝑠 − 36 (−8𝑠 + 36)(𝑠 − 1)
= = = −8𝑠 + 36
𝑠−1 𝑠−1
Continuing gives

−8𝑠 + 36 𝐴3 𝑝3 (𝑠)
= +
(𝑠 − 1)(𝑠 − 3)2 𝑠−1 (𝑠 − 3)2

−8𝑠 + 36
where 𝐴3 = =7
(𝑠 − 3)2 𝑠=1
1
and 𝑝3 (𝑠) = (−8𝑠 + 36 − (7)(𝑠 − 3)2 ) = −7𝑠 + 27
𝑠−1
16𝑠 4 8 7 −7𝑠 + 27
Thus = + + + . Now
(𝑠 − 1)3 (𝑠 − 3)2 (𝑠 − 1)3 (𝑠 − 1)2 𝑠−1 (𝑠 − 3)2
continue using Theorem 1 or replace 𝑠 by (𝑠 − 3) + 3 in the numerator of
16𝑠 4 8 7
the last fraction to get 3 2
= 3
+ 2
+ +
(𝑠 − 1) (𝑠 − 3) (𝑠 − 1) (𝑠 − 1) 𝑠−1
6 7
2

(𝑠 − 3) 𝑠−3

Section 2.3
1. Note that 𝑠 = 𝑖 is a root of 𝑠2 + 1. Applying Theorem 1 gives

1 𝐵1 𝑠 + 𝐶 1 𝑝1 (𝑠)
= 2 + 2
(𝑠2 2 2
+ 1) (𝑠 + 2) (𝑠 + 1) 2 (𝑠 + 1)(𝑠2 + 2)

1 1
where 𝐵1 𝑖 + 𝐶1 = 2
= 2 =1
(𝑠 + 2) 𝑠=𝑖
𝑖 +2
⇒ 𝐵1 = 0 and 𝐶1 = 1
1
and 𝑝1 (𝑠) = 2 (1 − (1)(𝑠2 + 2))
𝑠 +1
−𝑠2 − 1
= 2 = −1.
𝑠 +1
60 1 Solutions

−1
We now apply Theorem 1 on the remainder term .
(𝑠2 + 1)(𝑠2 + 2)

−1 𝐵2 𝑠 + 𝐶 2 𝑝2 (𝑠)
= + 2
(𝑠2 + 1)(𝑠2 + 2) (𝑠2 + 1) (𝑠 + 2)

−1
where 𝐵2 𝑖 + 𝐶2 = = −1
(𝑠2 + 2) 𝑠=𝑖
⇒ 𝐵2 = 0 and 𝐶2 = −1
1
and 𝑝2 (𝑠) = 2 (−1 − (−1)(𝑠2 + 2))
𝑠 +1
𝑠2 + 1
= 2 = 1.
𝑠 +1
Thus the (𝑠2 + 1)-chain is

The 𝑠2 + 1 -chain
1 1
(𝑠2 + 1)2 (𝑠2 + 2) (𝑠2 + 1)2
−1 −1
(𝑠2 + 1)(𝑠2 + 2) (𝑠2 + 1)
1
2
𝑠 +2

2. Note that 𝑠 = 2𝑖 is a root of 𝑠2 + 2. Applying Theorem 1 gives

𝑠3 𝐵1 𝑠 + 𝐶 1 𝑝1 (𝑠)
= + 2
(𝑠2 + 2)2 (𝑠2 + 3) (𝑠2 + 2)2 (𝑠 + 2)(𝑠2 + 3)

√ 𝑠3 ( 2𝑖)3 √

where 𝐵1 2𝑖 + 𝐶1 = 2 √
= √ = −2 2𝑖
(𝑠 + 3) 𝑠= 2𝑖
2
( 2𝑖) + 3
⇒ 𝐵1 = −2 and 𝐶1 = 0
1
and 𝑝1 (𝑠) = 2
(𝑠3 − (−2𝑠)(𝑠2 + 3))
𝑠 +1
3𝑠3 + 6𝑠
= = 3𝑠.
𝑠2 + 2
3𝑠
We now apply Theorem 1 on the remainder term .
(𝑠2 + 2)(𝑠2 + 3)
1 Solutions 61

3𝑠 𝐵2 𝑠 + 𝐶 2 𝑝2 (𝑠)
= + 2
(𝑠2 + 2)(𝑠2 + 3) 2
(𝑠 + 2) (𝑠 + 3)

√ √

3𝑠
where 𝐵2 2𝑖 + 𝐶2 = 2 √
= 3 2𝑖
(𝑠 + 3) 𝑠= 2𝑖

⇒ 𝐵2 = 3 and 𝐶2 = 0
1
and 𝑝2 (𝑠) = (3𝑠 − (3𝑠)(𝑠2 + 3))
𝑠2 + 2
−3𝑠(𝑠2 + 2)
= = −3𝑠.
𝑠2 + 2
Thus the (𝑠2 + 2)-chain is

The 𝑠2 + 2 -chain
𝑠3 −2𝑠
(𝑠2 + 2)2 (𝑠2 + 3) (𝑠2+ 2)2
3𝑠 3𝑠
(𝑠2 + 2)(𝑠2 + 3) 𝑠2 + 2
−3𝑠
𝑠2 + 3

3. Note that 𝑠 = 3𝑖 is a root of 𝑠2 + 3. Applying Theorem 1 gives

8𝑠 + 8𝑠2 𝐵1 𝑠 + 𝐶 1 𝑝1 (𝑠)
= + 2
(𝑠2 + 3)3 (𝑠2 + 1) (𝑠2 + 3)3 (𝑠 + 3)2 (𝑠2 + 1)
√ √
√ 8𝑠 + 8𝑠2 8 3𝑖 + 8( 3𝑖)2

where 𝐵1 3𝑖 + 𝐶1 = = √
(𝑠2 + 1) 𝑠=√3𝑖 ( 3𝑖)2 + 1

= −4 3𝑖 + 12
⇒ 𝐵1 = −4 and 𝐶1 = 12
1
and 𝑝1 (𝑠) = (8𝑠 + 8𝑠2 − (−4𝑠 + 12)(𝑠2 + 1))
𝑠2 + 3
4𝑠3 − 4𝑠2 + 12𝑠 − 12
= = 4(𝑠 − 1).
𝑠2 + 3
4𝑠 − 4
Apply Theorem 1 a second time on the remainder term .
(𝑠2 + 3)2 (𝑠2 + 1)
62 1 Solutions

4𝑠 − 4 𝐵2 𝑠 + 𝐶 2 𝑝2 (𝑠)
= + 2
(𝑠2 + 3)2 (𝑠2 + 1) 2
(𝑠 + 3) 2 (𝑠 + 3)(𝑠2 + 1)

√ √

4𝑠 − 4
where 𝐵2 3𝑖 + 𝐶2 = 2 √
= −2 3𝑖 + 2
(𝑠 + 1) 𝑠= 3𝑖

⇒ 𝐵2 = −2 and 𝐶2 = 2
1
and 𝑝2 (𝑠) = (4𝑠 − 4 − (−2𝑠 + 2)(𝑠2 + 1))
𝑠2 + 3
2𝑠3 − 2𝑠2 + 6𝑠 − 6
= = 2𝑠 − 2.
𝑠2 + 3
2𝑠 − 2
A third application of Theorem 1 on the remainder term
(𝑠2 + 3)(𝑠2 + 1)
gives

2𝑠 − 2 𝐵3 𝑠 + 𝐶 3 𝑝3 (𝑠)
= + 2
(𝑠2 + 3)(𝑠2 + 1) (𝑠2 + 3) (𝑠 + 1)

√ √

2𝑠 − 2
where 𝐵3 3𝑖 + 𝐶3 = 2 √
= − 3𝑖 + 1
(𝑠 + 1) 𝑠= 3𝑖

⇒ 𝐵3 = −1 and 𝐶3 = 1
1
and 𝑝3 (𝑠) = 2
(2𝑠 − 2 − (−𝑠 + 1)(𝑠2 + 1))
𝑠 +3
𝑠3 − 𝑠2 + 3𝑠 − 3
= = 𝑠 − 1.
𝑠2 + 3
Thus the (𝑠2 + 3)-chain is

The 𝑠2 + 3 -chain
8𝑠 + 8𝑠2 12 − 4𝑠
(𝑠+ 3)3 (𝑠2 + 1) (𝑠2 + 3)3
4(𝑠 − 1) 2 − 2𝑠
(𝑠 + 3)2 (𝑠2 + 1)
2 (𝑠2 + 3)2
2(𝑠 − 1) 1−𝑠
(𝑠2 + 3)(𝑠2 + 1) 𝑠2 + 3
𝑠−1
𝑠2 + 1

4. Note that 𝑠 = 2𝑖 is a root of 𝑠2 + 4. Applying Theorem 1 gives


1 Solutions 63

4𝑠4 𝐵1 𝑠 + 𝐶 1 𝑝1 (𝑠)
= + 2
(𝑠2 + 4)3 (𝑠2 + 6) (𝑠2 + 4)4 (𝑠 + 4)3 (𝑠2 + 6)

4𝑠4 4(2𝑖)4

where 𝐵1 2𝑖 + 𝐶1 = 2
=
(𝑠 + 6) 𝑠=2𝑖
(2𝑖)2 + 6
= 32
⇒ 𝐵1 = 0 and 𝐶1 = 32
1
and 𝑝1 (𝑠) = 2
(4𝑠4 − (32)(𝑠2 + 6))
𝑠 +4
4𝑠4 − 32𝑠2 − 192
= = 4𝑠2 − 48.
𝑠2 + 4
4𝑠2 − 48
Apply Theorem 1 a second time on the remainder term .
(𝑠2 + 4)3 (𝑠2 + 6)

4𝑠2 − 48 𝐵2 𝑠 + 𝐶 2 𝑝2 (𝑠)
= + 2
(𝑠2 + 4)3 (𝑠2 + 6) (𝑠2 + 4)3 (𝑠 + 4)2 (𝑠2 + 6)

4𝑠2 − 48

where 𝐵2 2𝑖 + 𝐶2 = = −32
(𝑠2 + 6) 𝑠=2𝑖
⇒ 𝐵2 = 0 and 𝐶2 = −32
1
and 𝑝2 (𝑠) = (4𝑠2 − 48 − (−32)(𝑠2 + 6))
𝑠2 + 4
36𝑠2 + 144
= = 36.
𝑠2 + 4
36
A third application of Theorem 1 on the remainder term
(𝑠2 + 4)2 (𝑠2 + 6)
gives

36 𝐵3 𝑠 + 𝐶 3 𝑝3 (𝑠)
= + 2
(𝑠2 + 4)2 (𝑠2 + 6) (𝑠2 + 4)2 (𝑠 + 4)(𝑠2 + 6)

36
where 𝐵3 2𝑖 + 𝐶3 = = 18
(𝑠2 + 6) 𝑠=2𝑖
⇒ 𝐵3 = 0 and 𝐶3 = 18
1
and 𝑝3 (𝑠) = (36 − (18)(𝑠2 + 6))
𝑠2 + 4
−18𝑠2 − 72
= = −18.
𝑠2 + 4
A fourth (and final) application of Theorem 1 on the remainder term
−18
gives
(𝑠2 + 4)(𝑠2 + 6)
64 1 Solutions

−18 𝐵4 𝑠 + 𝐶 4 𝑝4 (𝑠)
= + 2
(𝑠2 + 4)(𝑠2 + 6) 2
(𝑠 + 4) (𝑠 + 6)

−18
where 𝐵4 2𝑖 + 𝐶4 = = −9
(𝑠2 + 6) 𝑠=2𝑖
⇒ 𝐵4 = 0 and 𝐶4 = −9
1
and 𝑝4 (𝑠) = 2
(−18 − (−9)(𝑠2 + 6))
𝑠 +4
9𝑠2 + 36
= = 9.
𝑠2 + 4
Thus the (𝑠2 + 4)-chain is

The 𝑠2 + 4 -chain
4𝑠4 32
(𝑠2 + 4)4 (𝑠2 + 6) (𝑠2 + 4)4
4𝑠2 − 48 −32
(𝑠 + 4)3 (𝑠2 + 6)
2 (𝑠2 + 4)3
36 18
(𝑠2 + 4)2 (𝑠2 + 6) (𝑠2 + 4)2
−18 −9
(𝑠2 + 4)(𝑠2 + 6) 𝑠2 + 4
9
𝑠2 + 6

5. Note that 𝑠2 +2𝑠+2 = (𝑠+1)2 +1 so 𝑠 = −1±𝑖 are the roots of 𝑠2 +2𝑠+2.


We will use the root 𝑠 = −1+𝑖 for the partial fraction algorithm. Applying
Theorem 1 gives
1 Solutions 65

1 𝐵1 𝑠 + 𝐶 1
=
(𝑠2 + 2𝑠 + 2)2 (𝑠2 + 2𝑠 + 3)2 (𝑠2+ 2𝑠 + 2)2
𝑝1 (𝑠)
+
(𝑠2 + 2𝑠 + 2)(𝑠2 + 2𝑠 + 3)2

1
where 𝐵1 (−1 + 𝑖) + 𝐶1 = 2 2

(𝑠 + 2𝑠 + 3) 𝑠=−1+𝑖
1
= =1
((−1 + 𝑖)2 + 2)2
⇒ 𝐵1 = 0 and 𝐶1 = 1
1 − (1)(𝑠2 + 2𝑠 + 3)2
and 𝑝1 (𝑠) =
𝑠2 + 2𝑠 + 2
−(𝑠 + 2𝑠 + 4)(𝑠2 + 2𝑠 + 2)
2
=
𝑠2 + 2𝑠 + 2
2
= −(𝑠 + 2𝑠 + 4).

−(𝑠2 + 2𝑠 + 4)
Now apply Theorem 1 to the remainder term .
(𝑠2 + 2𝑠 + 2)(𝑠2 + 2𝑠 + 3)2

−(𝑠2 + 2𝑠 + 4) 𝐵2 𝑠 + 𝐶 2 𝑝2 (𝑠)
= +
(𝑠2 + 2𝑠 + 2)(𝑠2 + 2𝑠 + 3)2 (𝑠2 + 2𝑠 + 2) (𝑠2 + 2𝑠 + 3)2

−(𝑠2 + 2𝑠 + 4)

where 𝐵2 (−1 + 𝑖) + 𝐶2 = = −2
(𝑠2 + 2𝑠 + 3) 𝑠=−1+𝑖
⇒ 𝐵2 = 0 and 𝐶2 = −2
−(𝑠2 + 2𝑠 + 4) − (−2)(𝑠2 + 2𝑠 + 3)2
and 𝑝2 (𝑠) =
𝑠2 + 2𝑠 + 2
(2(𝑠 + 1) + 5)((𝑠 + 1)2 + 1)
2
=
𝑠2 + 2𝑠 + 2
2
= 2𝑠 + 4𝑠 + 7.

Thus the (𝑠2 + 2𝑠 + 2)-chain is

The 𝑠2 + 2𝑠 + 2 -chain
1 1
(𝑠2 + 2𝑠 + 2)2 (𝑠2 + 2𝑠 + 3)2 (𝑠2 + 2𝑠 + 2)2
−(𝑠2 + 2𝑠 + 4) −2
(𝑠 + 2𝑠 + 2)(𝑠2 + 2𝑠 + 3)2
2 𝑠2 + 2𝑠 + 2
2𝑠2 + 4𝑠 + 7
(𝑠2 + 2𝑠 + 3)2
66 1 Solutions

6. Note that 𝑠2 +2𝑠+2 = (𝑠+1)2 +1 so 𝑠 = −1±𝑖 are the roots of 𝑠2 +2𝑠+2.


We will use the root 𝑠 = −1+𝑖 for the partial fraction algorithm. Applying
Theorem 1 gives
1 𝐵1 𝑠 + 𝐶 1
=
(𝑠2 + 2𝑠 + 2)2 (𝑠2 + 4𝑠 + 5) (𝑠2 + 2𝑠 + 2)2
𝑝1 (𝑠)
+
(𝑠2 + 2𝑠 + 2)(𝑠2 + 4𝑠 + 5)

5𝑠 − 5
where 𝐵1 (−1 + 𝑖) + 𝐶1 =
(𝑠2 + 4𝑠 + 5) 𝑠=−1+𝑖
5𝑖 − 10
= = 5𝑖
2𝑖 + 1
⇒ 𝐵1 = 5 and 𝐶1 = 5
5𝑠 − 5 − (5𝑠 + 5)(𝑠2 + 4𝑠 + 5)
and 𝑝1 (𝑠) =
𝑠2 + 2𝑠 + 2
−(5𝑠 + 15)(𝑠2 + 2𝑠 + 2)
=
𝑠2 + 2𝑠 + 2
= −(5𝑠 + 15).

−(5𝑠 + 15)
Now apply Theorem 1 to the remainder term .
(𝑠2 + 2𝑠 + 2)(𝑠2 + 4𝑠 + 5)

−(5𝑠 + 15) 𝐵2 𝑠 + 𝐶 2 𝑝2 (𝑠)


= +
(𝑠2 + 2𝑠 + 2)(𝑠2 + 4𝑠 + 5) (𝑠2 + 2𝑠 + 2) (𝑠2 + 4𝑠 + 5)

−(5𝑠 + 15)
where 𝐵2 (−1 + 𝑖) + 𝐶2 = = −4 + 3𝑖
(𝑠2 + 4𝑠 + 5) 𝑠=−1+𝑖
⇒ 𝐵2 = 3 and 𝐶2 = −1
−(5𝑠 + 15) − (3𝑠 − 1)(𝑠2 + 4𝑠 + 5)
and 𝑝2 (𝑠) =
𝑠2 + 2𝑠 + 2
(−3𝑠 − 5)(𝑠2 + 2𝑠 + 2)
=
𝑠2 + 2𝑠 + 2
= −3𝑠 − 5.

Thus the (𝑠2 + 2𝑠 + 2)-chain is


1 Solutions 67

The 𝑠2 + 2𝑠 + 2 -chain
5𝑠 − 5 5𝑠 + 5
(𝑠2 + 2𝑠 + 2)2 (𝑠2 + 4𝑠 + 5) (𝑠2+ 2𝑠 + 2)2
−5𝑠 − 15 3𝑠 − 1
(𝑠2 + 2𝑠 + 2)(𝑠2 + 4𝑠 + 5) 𝑠2 + 2𝑠 + 2
−3𝑠 − 5
2
𝑠 + 4𝑠 + 5

7. Use Theorem 1 of Section 2.2 to compute the (𝑠 − 3)-chain:

𝑠 𝐴1 𝑝1 (𝑠)
= +
(𝑠2 + 1)(𝑠 − 3) 𝑠−3 𝑠2 + 1

𝑠 3
where 𝐴1 = 2
=
𝑠 + 1 𝑠=3
10
1 1
and 𝑝1 (𝑠) = (𝑠 − (3/10)(𝑠2 + 1)) = (−3𝑠2 + 10𝑠 − 3)
𝑠−3 10(𝑠 − 3)
−3𝑠 + 1
=
10
−3𝑠 + 1
Since the remainder term is already a simple partial fraction,
10(𝑠2 + 1)
we conclude
( )
𝑠 1 3 1 − 3𝑠
= +
(𝑠2 + 1)(𝑠 − 3) 10 𝑠 − 3 𝑠2 + 1

8. Use Theorem of Section 2.2 1 to compute the (𝑠 + 1)-chain:

4𝑠 𝐴1 𝑝1 (𝑠)
= +
(𝑠2 2
+ 1) (𝑠 + 1) 𝑠+1 (𝑠2 + 1)2

4𝑠
where 𝐴1 = 2 2
= −1
(𝑠 + 1) 𝑠=−1
1 𝑠4 + 2𝑠2 + 4𝑠 + 1
and 𝑝1 (𝑠) = (4𝑠 − (−1)(𝑠2 + 1)2 ) =
𝑠+1 𝑠+1
3 2
= 𝑠 − 𝑠 + 3𝑠 + 1
𝑠3 − 𝑠2 + 3𝑠 + 1
Now compute the 𝑠2 + 1-chain for the remainder term
(𝑠2 + 1)2
2
using Theorem 1. Since 𝑠 = 𝑖 is a root of 𝑠 + 1, an application of this
theorem gives
68 1 Solutions

𝑠3 − 𝑠2 + 3𝑠 + 1 𝐵1 𝑠 + 𝐶 1 𝑝1 (𝑠)
= 2 + 2
(𝑠2 + 1)2 (𝑠 + 1)2 (𝑠 + 1)

𝑠3 − 𝑠2 + 3𝑠 + 1

where 𝐵1 𝑖 + 𝐶1 = = 2𝑖 + 2
1
𝑠=𝑖
⇒ 𝐵1 = 2 and 𝐶1 = 2
1
and 𝑝1 (𝑠) = 2 (𝑠3 − 𝑠2 + 3𝑠 + 1 − (2𝑠 + 2)(1))
𝑠 +1
𝑠3 − 𝑠 2 + 𝑠 − 1
= = 𝑠 − 1.
𝑠2 + 1
𝑠−1
Since the remainder term is a simple partial fraction, we conclude
𝑠2 + 1
that the complete partial fraction decomposition is
4𝑠 2𝑠 + 2 𝑠−1 1
= 2 + 2 −
(𝑠2 2
+ 1) (𝑠 + 1) (𝑠 + 1) 2 𝑠 +1 𝑠+1

9. Use Theorem of Section 2.2 1 to compute the (𝑠 − 3)-chain:


2 𝐴1 𝑝1 (𝑠)
= +
(𝑠2 − 6𝑠 + 10)(𝑠 − 3) 𝑠−3 (𝑠2 − 6𝑠 + 10)

2
where 𝐴1 = 2 =2
(𝑠 − 6𝑠 + 10) 𝑠=3
1 −2𝑠2 + 12𝑠 − 18
and 𝑝1 (𝑠) = (2 − (2)(𝑠2 − 6𝑠 + 10)) =
𝑠−3 𝑠−3
= −2𝑠 + 6
−2𝑠 + 6
Since the remainder term is a simple partial fraction, we con-
𝑠2
− 6𝑠 + 10
2
clude that the complete partial fraction decomposition is 2 =
(𝑠 − 6𝑠 + 10)(𝑠 − 3)
2 6 − 2𝑠
+
𝑠 − 3 (𝑠 − 3)2 + 1
10. Use Theorem of Section 2.2 1 to compute the (𝑠 − 1)-chain:
30 𝐴1 𝑝1 (𝑠)
= +
(𝑠2 − 4𝑠 + 13)(𝑠 − 1) 𝑠−1 (𝑠2 − 4𝑠 + 13)

30
where 𝐴1 = 2
=3
(𝑠 − 4𝑠 + 13) 𝑠=1
1 −3𝑠2 + 12𝑠 − 9
and 𝑝1 (𝑠) = (30 − (3)(𝑠2 − 4𝑠 + 13)) =
𝑠−1 𝑠−1
= −3𝑠 + 9
1 Solutions 69

−3𝑠 + 9
Since the remainder term is a simple partial fraction, we con-
𝑠2 − 4𝑠 + 13
30
clude that the complete partial fraction decomposition is 2 =
(𝑠 − 4𝑠 + 13)(𝑠 − 1)
9 − 3𝑠 3
2
+
((𝑠 − 2) + 9) 𝑠 − 1

11. Note that 𝑠2 −4𝑠+8 = (𝑠−2)2 +2 so 𝑠 = 2±2𝑖 are the roots of 𝑠2 −4𝑠+8.
We will use the root 𝑠 = 2+2𝑖 to compute the (𝑠2 −4𝑠+8)-chain. Applying
Theorem 1 gives
25 𝐵1 𝑠 + 𝐶 1
= 2
(𝑠2 2
− 4𝑠 + 8) (𝑠 − 1) (𝑠 − 4𝑠 + 8)2
𝑝1 (𝑠)
+ 2
(𝑠 − 4𝑠 + 8)(𝑠 − 1)

25
where 𝐵1 (2 + 2𝑖) + 𝐶1 =
𝑠 − 1) 𝑠=2+2𝑖
25
= = 5 − 10𝑖
2𝑖 + 1
⇒ 𝐵1 = −5 and 𝐶1 = 15
25 − (−5𝑠 + 15)(𝑠 − 1)
and 𝑝1 (𝑠) =
𝑠2 − 4𝑠 + 8
2
(5)(𝑠 − 4𝑠 + 8)
=
𝑠2 − 4𝑠 + 8
= 5.
5
Now apply Theorem 1 to the remainder term .
(𝑠2 − 4𝑠 + 8)(𝑠 − 1)
5 𝐵2 𝑠 + 𝐶 2 𝑝2 (𝑠)
= 2 +
(𝑠2 − 4𝑠 + 8)(𝑠 − 1) (𝑠 − 4𝑠 + 8) 𝑠 − 1

5)
where 𝐵2 (2 + 2𝑖) + 𝐶2 = = 1 − 2𝑖
𝑠 − 1 𝑠=2+2𝑖
⇒ 𝐵2 = −1 and 𝐶2 = 3
5 − (3 − 𝑠)(𝑠 − 1)
and 𝑝2 (𝑠) =
𝑠2 − 4𝑠 + 8
(1)(𝑠2 − 4𝑠 + 8)
=
𝑠2 − 4𝑠 + 8
= 1.
25 −5𝑠 + 15
Thus the partial fraction expansion is = 2 +
(𝑠2 − 4𝑠 + 8)2 (𝑠 − 1) (𝑠 − 4𝑠 + 8)2
−𝑠 + 3 1
+
𝑠2 − 4𝑠 + 8 𝑠 − 1
70 1 Solutions

12. Note that 𝑠2 + 6𝑠 + 10 = (𝑠 + 3)2 + 1 so 𝑠 = −3 ± 𝑖 are the roots of


𝑠2 + 6𝑠 + 10. We will use the root 𝑠 = −3 + 𝑖 to compute the (𝑠2 + 6𝑠 + 10)-
chain. Applying Theorem 1 gives
𝑠 𝐵1 𝑠 + 𝐶 1
= 2
(𝑠2 + 6𝑠 + 10)2 (𝑠 + 3)2 (𝑠 + 6𝑠 + 10)2
𝑝1 (𝑠)
+
(𝑠2 + 6𝑠 + 10)(𝑠 + 3)2

𝑠
where 𝐵1 (−3 + 𝑖) + 𝐶1 = 2

(𝑠 + 3) ) 𝑠=−3+𝑖
= 3−𝑖
⇒ 𝐵1 = −1 and 𝐶1 = 0
𝑠 − (−𝑠)(𝑠 + 3)2
and 𝑝1 (𝑠) =
𝑠2 + 6𝑠 + 10
(𝑠)(𝑠2 + 6𝑠 + 10)
=
𝑠2 + 6𝑠 + 10
= 𝑠.
𝑠
Now apply Theorem 1 to the remainder term 2 .
(𝑠 + 6𝑠 + 10)(𝑠 + 3)2 )

𝑠 𝐵2 𝑠 + 𝐶 2 𝑝2 (𝑠)
= 2 +
(𝑠2 + 6𝑠 + 10)(𝑠 + 3)2 (𝑠 + 6𝑠 + 10) (𝑠 + 3)2

𝑠)
where 𝐵2 (−3 + 𝑖) + 𝐶2 = =3−𝑖
(𝑠 + 3)2 𝑠=−3+𝑖
⇒ 𝐵2 = −1 and 𝐶2 = 0
𝑠 − (−𝑠)(𝑠 + 3)2
and 𝑝2 (𝑠) =
𝑠2 + 6𝑠 + 10
(𝑠)(𝑠2 + 6𝑠 + 10)
=
𝑠2 + 6𝑠 + 10
= 𝑠.

The remainder term is


𝑠 (𝑠 + 3) − 3 −3 1
2
= 2
= 2
+ ,
(𝑠 + 3) (𝑠 + 3) (𝑠 + 3) 𝑠+3
so the partial fraction expansion of the entire rational function is
𝑠 −𝑠 𝑠
= 2 − 2
(𝑠2 + 6𝑠 + 10)2 (𝑠 + 3)2 (𝑠 + 6𝑠 + 10)2 𝑠 + 6𝑠 + 10
3 1
− +
(𝑠 + 3)2 𝑠+3
1 Solutions 71

13. Note that 𝑠2 +4𝑠+5 = (𝑠+2)2 +1 so 𝑠 = −2±𝑖 are the roots of 𝑠2 +4𝑠+5.
We will use the root 𝑠 = −2+𝑖 to compute the (𝑠2 +4𝑠+5)-chain. Applying
Theorem 1 gives
𝑠+1 𝐵1 𝑠 + 𝐶 1
= 2
(𝑠2 + 4𝑠 + 5)2 (𝑠2 + 4𝑠 + 6)2 (𝑠 + 4𝑠 + 5)2
𝑝1 (𝑠)
+
(𝑠2 + 4𝑠 + 5)(𝑠2 + 4𝑠 + 6)2

𝑠+1
where 𝐵1 (−2 + 𝑖) + 𝐶1 =
(𝑠2 + 4𝑠 + 6)2 ) 𝑠=−2+𝑖
= −1 + 𝑖
⇒ 𝐵1 = 1 and 𝐶1 = 1
𝑠 + 1 − (𝑠 + 1)(𝑠2 + 4𝑠 + 6)2
and 𝑝1 (𝑠) =
𝑠2 + 4𝑠 + 5
−(𝑠 + 1)((𝑠2 + 4𝑠 + 6)2 − 1)
=
𝑠2 + 4𝑠 + 5
−(𝑠 + 1)(𝑠2 + 4𝑠 + 7)(𝑠2 + 4𝑠 + 5)
=
𝑠2 + 4𝑠 + 5
2
= −(𝑠 + 1)(𝑠 + 4𝑠 + 7).

−(𝑠 + 1)(𝑠2 + 4𝑠 + 7)
Now apply Theorem 1 to the remainder term .
(𝑠2 + 4𝑠 + 5)(𝑠2 + 4𝑠 + 6)2 )

−(𝑠 + 1)(𝑠2 + 4𝑠 + 7) 𝐵2 𝑠 + 𝐶 2 𝑝2 (𝑠)


2 2 2
= 2 + 2
(𝑠 + 4𝑠 + 5)(𝑠 + 4𝑠 + 6) (𝑠 + 4𝑠 + 5) (𝑠 + 4𝑠 + 6)2

−(𝑠 + 1)(𝑠2 + 4𝑠 + 7)

where 𝐵2 (−2 + 𝑖) + 𝐶2 = = 2 − 2𝑖
(𝑠2 + 4𝑠 + 6)2
𝑠=−2+𝑖
⇒ 𝐵2 = −2 and 𝐶2 = −2
−(𝑠 + 1)(𝑠2 + 4𝑠 + 7) − (−2𝑠 − 2)(𝑠2 + 4𝑠 + 6)2
and 𝑝2 (𝑠) =
𝑠2 + 4𝑠 + 5
(𝑠 + 1)(2(𝑠2 + 4𝑠 + 6) + 1)(𝑠2 + 4𝑠 + 5)
=
𝑠2 + 4𝑠 + 5
2
= (𝑠 + 1)(2(𝑠 + 4𝑠 + 6) + 1).

The remainder term is


(𝑠 + 1)(2(𝑠2 + 4𝑠 + 6) + 1) 𝑠+1 2𝑠 + 2
2 2
= 2 2
+ 2
(𝑠 + 4𝑠 + 6) (𝑠 + 4𝑠 + 6) 𝑠 + 4𝑠 + 6

so the partial fraction expansion of the entire rational function is


72 1 Solutions

𝑠+1 𝑠+1 2𝑠 + 2
= 2 + 2
(𝑠2 + 4𝑠 + 5)2 (𝑠2 + 4𝑠 + 6)2 (𝑠 + 4𝑠 + 6)2 𝑠 + 4𝑠 + 6
𝑠+1 2𝑠 + 2
+ 2 − 2
(𝑠 + 4𝑠 + 5)2 𝑠 + 4𝑠 + 5

𝑢
14. Using the hint, let 𝑢 = 𝑠2 . Then the rational function becomes ,
(𝑢 + 5)3 (𝑢 + 6)2
which can be put in partial fraction form by the technique of Section 2.2.
Use Theorem 1 of Section 2.2 to compute the (𝑢 + 5)-chain:
𝑢 𝐴1 𝑝1 (𝑢)
= +
(𝑢 + 5)3 (𝑢 + 6)2 (𝑢 + 5)3 (𝑢 + 5)2 (𝑢 + 6)2

𝑢
where 𝐴1 = 2
= −5
(𝑢 + 6) 𝑢=−5
1
and 𝑝1 (𝑢) = (𝑢 − (−5)(𝑢 + 6)2 )
𝑢+5
5𝑢2 + 61𝑢 + 180 (5𝑢 + 36)(𝑢 + 5)
= = = 5𝑢 + 36.
𝑢+6 𝑢+5
Continuing gives
5𝑢 + 36 𝐴2 𝑝2 (𝑢)
= +
(𝑢 + 5)2 )(𝑢 + 6)2 (𝑢 + 5)2 (𝑢 + 5)(𝑢 + 6)2

5𝑢 + 36
where 𝐴2 = = 11
(𝑢 + 6)2 𝑢=−5
1
and 𝑝2 (𝑢) = (5𝑢 + 36 − (11)(𝑢 + 6)2 )
𝑢+5
−11𝑢2 − 127𝑢 − 360 −(11𝑢 + 72)(𝑢 + 5)
= =
𝑢+5 𝑢+5
= −(11𝑢 + 72).
Another recursion gives
−11𝑢 − 72 𝐴3 𝑝3 (𝑢)
= +
(𝑢 + 5)(𝑢 + 6)2 (𝑢 + 5) (𝑢 + 6)2

−11𝑢 − 72
where 𝐴3 = = −17
(𝑢 + 6)2 𝑢=−5
1
and 𝑝3 (𝑢) = (−11𝑢 − 72 − (−17)(𝑢 + 6)2 )
𝑢+5
17𝑢2 + 193𝑢 + 540 (17𝑢 + 108)(𝑢 + 5)
= =
𝑢+5 𝑢+5
= 17𝑢 + 108.
1 Solutions 73

Thus we get
𝑢 −5 11 17
= + −
(𝑢 + 5)3 (𝑢 + 6)2 (𝑢 + 5)3 (𝑢 + 5)2 𝑢+5
17𝑢 + 108
+
(𝑢 + 6)2
−5 11 17
= + −
(𝑢 + 5)3 (𝑢 + 5)2 𝑢+5
17 6
+ + .
(𝑢 + 6)2 𝑢+6

Replacing 𝑢 by 𝑠2 in the above expression, gives the following expression


𝑠2
for the partial fraction decomposition of 2 :
(𝑠 + 5)3 (𝑠2 + 6)2
−5 11 17 6 17
+ 2 − 2 + 2 + 2
(𝑠2 + 5) 3 (𝑠 + 5) 2 𝑠 + 5 (𝑠 + 6) 2 𝑠 +6

Section 2.4
1. ℒ−1 {−5/𝑠} = −5ℒ−1 {1/𝑠} = −5

2. ℒ−1 {3/(𝑠 − 4)} = 3ℒ−1 {1/(𝑠 − 4)} = 3𝑒4𝑡


{ }
3 4
3. ℒ−1 = 3ℒ−1 1/𝑠2 − 2ℒ−1 2/𝑠3 = 3𝑡 − 2𝑡2
{ } { }
2
− 3
𝑠 𝑠
{ } { }
4 1
4. ℒ−1 = 2ℒ−1 = 2𝑒−3𝑡/2
2𝑠 + 3 𝑠 + (3/2)
{ } { }
−1 3𝑠 −1 𝑠
5. ℒ = 3ℒ = 3 cos 2𝑡
𝑠2 + 4 𝑠2 + 2 2
{ √ }

{ }
−1 2 2 −1 3 2
6. ℒ 2
=√ ℒ √ = √ sin 3 𝑡
𝑠 +3 3 2
𝑠 + ( 3) 2 3

2𝑠 − 5
7. First, we have 𝑠2 + 6𝑠 + 9 = (𝑠 + 3)2 . Partial fractions gives 2 =
{ } 𝑠 + 6𝑠 + 9
−11 2 2𝑠 − 5
+ . So ℒ−1 = −11𝑡𝑒−3𝑡 + 2𝑒−3𝑡
(𝑠 + 3)2 𝑠+3 (𝑠 + 3)2
{ }
2𝑠 − 5 −11 2 −1 2𝑠 − 5
8. Partial fractions gives = + . Thus ℒ =
(𝑠 + 3)3 (𝑠 + 3)3 (𝑠 + 3)2 (𝑠 + 3)3
−11 2 −3𝑡
𝑡 𝑒 + 2𝑡𝑒−3𝑡
2
74 1 Solutions
{ }
6 6 −1 1 −1 6
9. = = + . So ℒ =
𝑠2 + 2𝑠 − 8 (𝑠 − 2)(𝑠 + 4) 𝑠+4 𝑠−2 𝑠2 + 2𝑠 − 8
𝑒2𝑡 − 𝑒−4𝑡
{ }
𝑠 𝑠 −2 3 −1 𝑠
10. 2 = = + . So ℒ =
𝑠 − 5𝑠 + 6 (𝑠 − 2)(𝑠 − 3) 𝑠−2 𝑠−3 (𝑠 − 2)(𝑠 + 3)
3𝑒3𝑡 − 2𝑒2𝑡
2𝑠2 − 5𝑠 + 1
{ 2 }
−1 3 2 −1 2𝑠 − 5𝑠 + 1
11. = + + . So ℒ =
(𝑠 − 2)4 (𝑠 − 2)4 (𝑠 − 2)3 (𝑠 − 2)2 (𝑠 − 2)4
−1 3 2𝑡 3 2 2𝑡
𝑡 𝑒 + 𝑡 𝑒 + 2𝑡𝑒2𝑡
6 2
{ }
2𝑠 + 6 2𝑠 + 6 −2 4 −1 2𝑠 + 6
12. 2 = = + . So ℒ =
𝑠 − 6𝑠 + 5 (𝑠 − 1)(𝑠 − 5) 𝑠−1 𝑠−5 𝑠62 − 6𝑠 + 5
5𝑡 𝑡
4𝑒 − 2𝑒
4𝑠2 4𝑠2
{ }
1 1 1 1 −1
13. = + + − . So ℒ =
(𝑠 − 1)2 (𝑠 + 1)2 (𝑠 − 1)2 𝑠 − 1 (𝑠 + 1)2 𝑠 + 1 (𝑠 − 1)2 (𝑠 + 1)2
𝑡𝑒𝑡 + 𝑒𝑡 + 𝑡𝑒−𝑡 − 𝑒−𝑡
9𝑡2
{ }
27 9 3 1 1 27
14. 2 = 3− 2+ − . So ℒ−1 3
= −3𝑡+1−𝑒−3𝑡
𝑠 (𝑠 + 3) 𝑠 𝑠 𝑠 𝑠−3 𝑠 (𝑠 + 3) 2
{ }
8𝑠 + 16 4 1 𝑠 2 −1 8𝑠 + 16
15. 2 = − + − . So ℒ =
(𝑠 + 4)(𝑠 − 2)2 (𝑠 − 2)2 𝑠 − 2 𝑠2 + 4 𝑠2 + 4 (𝑠2 + 4)(𝑠 − 2)2
2𝑡 2𝑡
4𝑡𝑒 − 𝑒 + cos 2𝑡 − sin 2𝑡
{ }
5𝑠 + 15 2 𝑠 3 −1 5𝑠 + 15
16. 2 = −2 2 + . So ℒ =
(𝑠 + 9)(𝑠 − 1) 𝑠−1 𝑠 + 9 𝑠2 + 9 (𝑠2 + 9)(𝑠 − 1)
𝑡
−2 cos 3𝑡 + sin 3𝑡 + 2𝑒
{ }
12 −6 3 4 1 12
17. 2 = 2 + − + . So ℒ−1 =
𝑠 (𝑠 + 1)(𝑠 − 2) 𝑠 𝑠 𝑠+1 𝑠−2 𝑠2 (𝑠 + 1)(𝑠 − 2)
3 − 6𝑡 + 𝑒2𝑡 − 4𝑒−𝑡
2𝑠 6 14 22 8 22
18. 3 2
= 3
+ 2
+ + 2
− . So
(𝑠 −{3) (𝑠 − 4) (𝑠}− 3) (𝑠 − 3) 𝑠−3 (𝑠 − 4) 𝑠−4
2𝑠
ℒ−1 = 3𝑡2 𝑒3𝑡 + 14𝑡𝑒3𝑡 + 22𝑒3𝑡 + 8𝑡𝑒4𝑡 − 22𝑒4𝑡
(𝑠 − 3)3 (𝑠 − 4)2
2𝑠 2𝑠
19. First we have 𝑠2 + 2𝑠 + 5 = (𝑠 + 1)2 + 4. So = =
𝑠2 + 2𝑠 + 5 (𝑠 + 1)2 + 4
2(𝑠 + 1) − 2 𝑠+1 2
=2 − . The First Translation princi-
(𝑠 + 1)2 + 4 { (𝑠 + 1)2 +}4 (𝑠 + 1) 2
{ +4 } { }
−1 2𝑠 −1 𝑠+1 −1 2
ple gives ℒ = 2ℒ −ℒ =
𝑠2 + 2𝑠 + 5 (𝑠 + 1)2 + 4 (𝑠 + 1)2 + 4
−𝑡 −𝑡
2𝑒 cos 2𝑡 − 𝑒 sin 2𝑡
1 Solutions 75
{ }
1 1 1
20. = . Thus ℒ−1 = 𝑒−3𝑡 sin 𝑡
𝑠2 + 6𝑠 + 10 (𝑠 + 3)2 + 1 (𝑠 + 3)2 + 1
{ }
𝑠−1 𝑠−4 1 −1 𝑠−1
21. = +3 . Thus ℒ =
𝑠2 − 8𝑠 + 17 (𝑠 − 4)2 + 1 (𝑠 − 4)2 + 1 𝑠2 − 8𝑠 + 17
𝑒4𝑡 cos 𝑡 + 3𝑒4𝑡 sin 𝑡


{ }
2𝑠 + 4 𝑠−2 8 −1 2𝑠 + 4
22. 2 =2 √ + 8 √ . Thus ℒ =
𝑠 − 4𝑠 + 12 (𝑠 − 2)√2 + ( 8)2 (𝑠 − 2)2 + ( 8)2 𝑠2 − 4𝑠 + 12
√ √
2𝑒2𝑡 cos 8 𝑡 + 8 𝑒2𝑡 sin 8 𝑡
{ }
𝑠−1 𝑠−1 −1 𝑠−1
23. 2 = . Thus ℒ = 𝑒𝑡 cos 3𝑡
𝑠 − 2𝑠 + 10 (𝑠 − 1)2 + 32 𝑠2 − 2𝑠 + 10
{ }
𝑠−5 𝑠−3 2 −1 𝑠−5
24. 2 = − . Thus ℒ =
𝑠 − 6𝑠 + 13 (𝑠 − 3)2 + 22 (𝑠 − 3)2 + 22 𝑠2 − 6𝑠 + 13
𝑒3𝑡 cos 2𝑡 − 𝑒3𝑡 sin 2𝑡
{ } { }
8𝑠 𝑠 8
25. ℒ−1 = 8ℒ −1
= (2𝑡 sin 2𝑡) = 2𝑡 sin 2𝑡
(𝑠2 + 4)2 (𝑠2 + 22 )2 2 ⋅ 22
{ } { }
−1 9 −1 3 3
26. ℒ = 3ℒ = (sin 3𝑡 − 3𝑡 cos 3𝑡) =
(𝑠2 + 9)2 (𝑠2 + 32 )2 2 ⋅ 32
1 1
sin 3𝑡 − 𝑡 cos 3𝑡
6 2
2
27. We first complete the square
{ 𝑠 + 4𝑠 + 5 } = (𝑠 + 2)2{+ 1. By the transla- }
2𝑠 (𝑠 + 2) − 2
tion principle we get ℒ−1 2
= 2ℒ −1
=
( { } (𝑠 + { 4𝑠 + 5)2 }) ((𝑠(+ 2)2 + 1)2 )
𝑠 1 1 1
2𝑒−2𝑡 ℒ−1 − 2ℒ −1
= 2𝑒 −2𝑡
𝑡 sin 𝑡 − 2( (sin 𝑡 − 𝑡 cos 𝑡) =
(𝑠2 + 1)2 (𝑠2 + 1)2 2 2
2𝑡𝑒−2𝑡 cos 𝑡 + (𝑡 − 2)𝑒−2𝑡 sin 𝑡

28. We first complete the square


{ 𝑠2 −6𝑠+10}= (𝑠−3)2 +1.{ By the translation }
−1 2𝑠 + 2 −1 (𝑠 − 3) + 3 + 1
principle we get ℒ 2 2
= 2ℒ 2 2
=
( { } (𝑠 − 6𝑠{+ 10) }) ((𝑠
( − 3) + 1) )
3𝑡 −1 𝑠 −1 1 3𝑡 1 1
2𝑒 ℒ + 4ℒ = 2𝑒 𝑡 sin 𝑡 + 4( (sin 𝑡 − 𝑡 cos 𝑡) =
(𝑠2 + 1)2 (𝑠2 + 1)2 2 2
−4𝑡𝑒3𝑡 cos 𝑡 + (𝑡 + 4)𝑒3𝑡 sin 𝑡

29. We first complete the square


{ 𝑠2 +8𝑠+17}= (𝑠+4)2 +1. { By the translation}
−1 2𝑠 −1 (𝑠 + 4) − 4
principle we get ℒ 2 2
= 2ℒ =
( { }(𝑠 + 8𝑠 + { 17) }) + 4)2 + 1)2
((𝑠 ( )
𝑠 1 1 1
2𝑒−4𝑡 ℒ−1 − 4ℒ −1
= 2𝑒 −4𝑡
𝑡 sin 𝑡 − 4( (sin 𝑡 − 𝑡 cos 𝑡) =
(𝑠2 + 1)2 (𝑠2 + 1)2 2 2
4𝑡𝑒−4𝑡 cos 𝑡 + (𝑡 − 4)𝑒−4𝑡 sin 𝑡
76 1 Solutions

2
30. We first complete the square
{ 𝑠 + 2𝑠 + 2 } = (𝑠 + 1){2 + 1. By the trans-
}
𝑠+1 𝑠+1
lation principle we get ℒ−1 = ℒ −1
=
( { }) (𝑠2 + 2𝑠 + 2)3 ((𝑠 + 1)2 + 1)3
𝑠 1( ) 1
𝑒−𝑡 ℒ−1 = 𝑒−𝑡 𝑡 sin 𝑡 − 𝑡2 cos 𝑡 = (𝑡𝑒−𝑡 sin 𝑡−𝑡2 𝑒−𝑡 cos 𝑡)
(𝑠2 + 1)3 8 8
2 2 2
31. We first complete the square { 𝑠 − 2𝑠 + 5 = } (𝑠 − 1) {+ 2 . By the transla-
}
−1 1 −1 1
tion principle we get ℒ =ℒ =
( { }) (𝑠2 (
− 2𝑠 +{5)3 2 2 3
}) ((𝑠 − 1) + 2 )
1 1 2
𝑒𝑡 ℒ−1 2 2 3
= 𝑒𝑡 ℒ−1
(𝑠 + 2 ) 2 (𝑠 + 22 )3
2
1
= 𝑒𝑡 (3 − (2𝑡)2 ) sin 2𝑡 − 6𝑡 cos 2𝑡
( )
2⋅8⋅2 4
1 (
(3 − 4𝑡2 )𝑒𝑡 sin 2𝑡 − 6𝑡𝑒𝑡 cos 2𝑡
)
=
256
32. We first complete the square
{ 𝑠2 −6𝑠+10}= (𝑠−3)2 +1. { By the translation
}
−1 8𝑠 −1 (𝑠 − 3) + 3
principle we get ℒ 2 3
= 8ℒ =
( { } (𝑠 − 6𝑠{+ 10) }) ((𝑠 − 3)2 + 1)3
𝑠 1
8𝑒3𝑡 ℒ−1 2 3
+ 3ℒ−1
( (𝑠 + 1) (𝑠 + 1)3
2
)
1 1
= 8𝑒3𝑡 𝑡 sin 𝑡 − 𝑡2 cos 𝑡 + 3 ((3 − 𝑡2 ) sin 𝑡 − 3𝑡 cos 𝑡)
( )
8 8
= (−3𝑡2 + 𝑡 + 9)𝑒3𝑡 sin 𝑡 − (𝑡2 + 9𝑡)𝑒3𝑡 cos 𝑡
2 2
33. We first complete the square
{ 𝑠 − 8𝑠 + 17 = } (𝑠 − 4) {+ 1. By the transla-
}
−1 𝑠−4 −1 𝑠−4
tion principle we get ℒ =ℒ =
( { }) (𝑠2 − 8𝑠 + 17)4 ((𝑠 − 4)2 + 1)4
𝑠 1 (
𝑒4𝑡 ℒ−1 = 𝑒4𝑡 (3𝑡 − 𝑡3 ) sin 𝑡 − 3𝑡2 cos 𝑡
)
2
(𝑠 + 1) 4 48
1 ( 3 4𝑡 2 4𝑡
)
= (−𝑡 + 3𝑡)𝑒 cos 𝑡 − 3𝑡 𝑒 cos 𝑡
48
2 2 2
34. We first complete the square
{ 𝑠 + 4𝑠 + 8 = } (𝑠 + 2) {+ 2 . By the transla-
}
2 2
tion principle we get ℒ−1 2 3
= ℒ −1
=
( { }) (𝑠 + 4𝑠 + 8) ((𝑠 + 2)2 + 22 )3
2 1 (
𝑒−2𝑡 ℒ−1 = 𝑒−2𝑡 (3 − (2𝑡)2 ) sin 2𝑡 − 6𝑡 cos 2𝑡
)
(𝑠2 + 22 )3 8 ⋅ 24
1 (
(3 − 4𝑡2 )𝑒−2𝑡 sin 2𝑡 − 6𝑡𝑒−2𝑡 cos 2𝑡
)
=
128
1 1/(𝑎 − 𝑏) −1/(𝑏 − 𝑎)
35. Compute the partial fraction = + .
(𝑠 − 𝑎)(𝑠 − 𝑏) 𝑠−𝑎 𝑠−𝑏
Then
𝑒𝑎𝑡 𝑒𝑏𝑡
{ } { }
1 1/(𝑎 − 𝑏) −1/(𝑏 − 𝑎)
ℒ−1 = ℒ−1 + = + .
(𝑠 − 𝑎)(𝑠 − 𝑏) 𝑠−𝑎 𝑠−𝑏 𝑎−𝑏 𝑏−𝑎
1 Solutions 77

36. Apply the inverse Laplace transform to the partial fraction expansion
𝑠 𝑎/(𝑎 − 𝑏) 𝑏/(𝑏 − 𝑎)
= + .
(𝑠 − 𝑎)(𝑠 − 𝑏) 𝑠−𝑎 𝑠−𝑏

37. Apply the inverse Laplace transform to the partial fraction expansion
1 1 1 1 1 1 1
= + + .
(𝑠 − 𝑎)(𝑠 − 𝑏)(𝑠 − 𝑐) (𝑎 − 𝑏)(𝑎 − 𝑐) 𝑠 − 𝑎 (𝑏 − 𝑎)(𝑏 − 𝑐) 𝑠 − 𝑏 (𝑐 − 𝑎)(𝑐 − 𝑏) (𝑠 − 𝑐)

38. Apply the inverse Laplace transform to the partial fraction expansion
𝑠 𝑎 1 𝑏 1 𝑐 1
= + + .
(𝑠 − 𝑎)(𝑠 − 𝑏)(𝑠 − 𝑐) (𝑎 − 𝑏)(𝑎 − 𝑐) 𝑠 − 𝑎 (𝑏 − 𝑎)(𝑏 − 𝑐) 𝑠 − 𝑏 (𝑐 − 𝑎)(𝑐 − 𝑏) (𝑠 − 𝑐)

39. Apply the inverse Laplace transform to the partial fraction expansion

𝑠2 𝑎2 1 𝑏2 1 𝑐2 1
= + + .
(𝑠 − 𝑎)(𝑠 − 𝑏)(𝑠 − 𝑐) (𝑎 − 𝑏)(𝑎 − 𝑐) 𝑠 − 𝑎 (𝑏 − 𝑎)(𝑏 − 𝑐) 𝑠 − 𝑏 (𝑐 − 𝑎)(𝑐 − 𝑏) (𝑠 − 𝑐)

40. Apply the inverse Laplace transform to the partial fraction expansion

𝑠𝑘 𝐴1 𝐴𝑛
= +⋅⋅⋅+
(𝑠 − 𝑟1 ) ⋅ ⋅ ⋅ (𝑠 − 𝑟𝑛 ) 𝑠 − 𝑟1 𝑠 − 𝑟𝑛
where
𝑠𝑘 𝑟𝑘

= ′ 𝑖 .

𝐴𝑖 =
𝑞(𝑠)/(𝑠 − 𝑟𝑖 ) 𝑠=𝑟𝑖
𝑞 (𝑟𝑖 )
The last equality is true since the product rule for derivatives implies that

𝑞 ′ (𝑟𝑖 ) = (𝑟1 − 𝑟1 ) ⋅ ⋅ ⋅ (𝑟𝑖 − 𝑟𝑖−1 )(𝑟𝑖 − 𝑟𝑖+1 ) ⋅ ⋅ ⋅ (𝑟𝑖 − 𝑟𝑛 ),

that is, the derivative of 𝑞(𝑠) evaluated at one of the roots 𝑟𝑖 is obtained
by deleting the term 𝑠 − 𝑟𝑖 from 𝑞(𝑠) and then evaluating at 𝑟𝑖 and this is
the same expression which is evaluated to put in the denominator of the
coefficient 𝐴𝑖 .

41. This is directly from Table 2.3.

42. Apply the inverse Laplace transform to the partial fraction expansion
𝑠 (𝑠 − 𝑎) + 𝑎 1 𝑎
= = + .
(𝑠 − 𝑎)2 (𝑠 − 𝑎)2 𝑠 − 𝑎 (𝑠 − 𝑎)2

43. This is directly from Table 2.3.


78 1 Solutions

44. Apply the inverse Laplace transform to the partial fraction expansion
𝑠 (𝑠 − 𝑎) + 𝑎 1 𝑎
= = + .
(𝑠 − 𝑎)3 (𝑠 − 𝑎)3 (𝑠 − 𝑎)2 (𝑠 − 𝑎)3

45. Apply the inverse Laplace transform to the partial fraction expansion
𝑠2 ((𝑠 − 𝑎) + 𝑎)2 1 2𝑎 𝑎2
3
= 3
= + 2
+ .
(𝑠 − 𝑎) (𝑠 − 𝑎) 𝑠 − 𝑎 (𝑠 − 𝑎) (𝑠 − 𝑎)3

46. To compute the partial fraction write 𝑠 = (𝑠 − 𝑎) + 𝑎 and compute 𝑠𝑘 =


((𝑠 − 𝑎) + 𝑎)𝑘 by the binomial theorem:
𝑘 ( )
∑ 𝑘
𝑠𝑘 = ((𝑠 − 𝑎) + 𝑎)𝑘 = 𝑎𝑘−𝑙 (𝑠 − 𝑎)𝑙 .
𝑙
𝑙=0

Thus,
𝑘 ( )
𝑠𝑘 ∑ 𝑘 𝑘−𝑙 1
= 𝑎 .
(𝑠 − 𝑎)𝑛 𝑙 (𝑠 − 𝑎)𝑛−𝑙
𝑙=0
Now apply the inverse Laplace transform.

Section 2.5
1. Yes.

2. No; 𝑡−2 is not a polynomial.


𝑡
3. Yes; = 𝑡𝑒−𝑡 .
𝑒𝑡
1
4. No; is not a polynomial.
𝑡
√ √
𝜋 2 2
5. Yes; 𝑡 sin(4𝑡 − ) = 𝑡( sin 4𝑡 − cos 4𝑡).
4 2 2
6. Yes; (𝑡 + 𝑒𝑡 )2 = 𝑡2 + 2𝑡𝑒𝑡 + 𝑒2𝑡 .

7. No.

8. Yes.
1
9. No; 𝑡 2 is not a polynomial.
sin 2𝑡
10. Yes; = 𝑒−2𝑡 sin 2𝑡.
𝑒2𝑡
1 Solutions 79

11. No.

12. ℬ𝑞 = 𝑒4𝑡
{ }

13. ℬ𝑞 = 𝑒−6𝑡
{ }

14. ℬ𝑞 = 1, 𝑒−5𝑡
{ }

15. ℬ𝑞 = 𝑒4𝑡 , 𝑒−𝑡


{ }

16. ℬ𝑞 = 𝑒3𝑡 , 𝑡𝑒3𝑡


{ }

17. ℬ𝑞 = 𝑒2𝑡 , 𝑒7𝑡


{ }

18. ℬ𝑞 = 𝑒3𝑡 , 𝑒−2𝑡


{ }

19. ℬ𝑞 = 𝑒−3𝑡 , 𝑒−6𝑡


{ }

20. ℬ𝑞 = 𝑒𝑡/2 , 𝑒4𝑡/3


{ }

{ √ √ }
21. ℬ𝑞 = 𝑒(−1+ 2)𝑡 , 𝑒(−1− 2)𝑡
{ √ √ }
22. ℬ𝑞 = 𝑒(2+ 3)𝑡 , 𝑒(2− 3)𝑡

23. ℬ𝑞 = 𝑒5𝑡 , 𝑡𝑒5𝑡


{ }

24. ℬ𝑞 = 𝑒−3𝑡/2 , 𝑡𝑒−3𝑡/2


{ }

25. ℬ𝑞 = {cos 3𝑡, sin 3𝑡}

26. ℬ𝑞 = {cos(5𝑡/2), sin(5𝑡/2)}

27. ℬ𝑞 = 𝑒−2𝑡 cos 3𝑡, 𝑒−2𝑡 sin 3𝑡


{ }

28. ℬ𝑞 = {𝑒𝑡 cos 2𝑡, 𝑒𝑡 sin 2𝑡}


{ √ √ }
29. ℬ𝑞 = 𝑒𝑡/2 cos 23 𝑡, 𝑒𝑡/2 sin 23 𝑡

30. ℬ𝑞 = {𝑒𝑡 , cos 𝑡, sin 𝑡}

31. ℬ𝑞 = 1, 𝑡, 𝑡2
{ }

√ √ }
32. ℬ𝑞 = 𝑒−2𝑡 , 𝑒𝑡 cos 3𝑡, 𝑒𝑡 sin 3𝑡
{

33. ℬ𝑞 = 𝑒𝑡/2 , 𝑒𝑡 , 𝑡𝑒𝑡


{ }

34. ℬ𝑞 = 𝑒−2𝑡 , 𝑒3𝑡 𝑡, 𝑒−3𝑡


{ }
80 1 Solutions

35. ℬ𝑞 = {𝑒𝑡 , 𝑒−𝑡 , cos 𝑡, sin 𝑡}


{ √ √ √ √ }
36. ℬ𝑞 = cos 3𝑡, sin 3𝑡, cos 2𝑡, sin 2𝑡

37. ℬ𝑞 = 𝑒2𝑡 , 𝑒−2𝑡 , 𝑡𝑒2𝑡 , 𝑡𝑒−2𝑡


{ }

38. ℬ𝑞 = 𝑒𝑡 , 𝑡𝑒𝑡 , 𝑡2 𝑒𝑡 , 𝑒−7𝑡 , 𝑡𝑒−7𝑡


{ }

39. ℬ𝑞 = cos 𝑡, 𝑡 cos 𝑡, 𝑡2 cos 𝑡, sin 𝑡, 𝑡 sin 𝑡, 𝑡2 sin 𝑡


{ }

𝑝1 (𝑠) 𝑝2 (𝑠)
40. 𝑟1 (𝑠) = with deg 𝑝1 (𝑠) < deg 𝑞1 (𝑠) and 𝑟2 (𝑠) = with
𝑞1 (𝑠) 𝑞2 (𝑠)
𝑝1 (𝑠)𝑝2 (𝑠)
deg 𝑝2 (𝑠) < deg 𝑞2 (𝑠). Thus, 𝑟1 (𝑠)𝑟2 (𝑠) = and
𝑞1 (𝑠)𝑞2 (𝑠)
deg(𝑝1 (𝑠)𝑝2 (𝑠) = deg 𝑝1 (𝑠)+deg 𝑝2 (𝑠) < deg 𝑞1 (𝑠)+deg 𝑞2 (𝑠) = deg(𝑞1 (𝑠)𝑞2 (𝑠)).

𝑝(𝑠)
41. 𝑟(𝑠) = with deg 𝑝(𝑠) < deg 𝑞(𝑠). But deg 𝑝(𝑠) = deg 𝑝(𝑠−𝑎) since, for
𝑞(𝑠)
any natural number 𝑛, (𝑠−𝑎)𝑛 = 𝑠𝑛 +lower degree terms. Thus translating
the highest power of 𝑠 found in 𝑝(𝑠) will produce the same highest power
of 𝑠 in 𝑝(𝑠−𝑎). Since the same is also true for 𝑞(𝑠), it follows that deg 𝑝(𝑠−
𝑝(𝑠 − 𝑎)
𝑎) = deg 𝑝(𝑠) < deg 𝑞(𝑠) = deg 𝑞(𝑠 − 𝑎). Thus, 𝑟(𝑠 − 𝑎) = is a
𝑞(𝑠 − 𝑎)
proper rational function.
𝑝(𝑠)
42. If 𝑟(𝑠) ∈ ℛ then 𝑟(𝑠) = where deg 𝑝(𝑠) = 𝑚 < 𝑛 = deg 𝑞(𝑠). Then
𝑞(𝑠)
𝑞(𝑠)𝑝′ (𝑠) − 𝑞 ′ (𝑠)𝑝(𝑠)
𝑟′ (𝑠) = ,
(𝑞(𝑠))2
and deg(𝑞(𝑠)𝑝′ (𝑠) − 𝑞 ′ (𝑠)𝑝(𝑠)) ≤ max(deg(𝑞(𝑠)𝑞 ′ (𝑠)), deg(𝑞 ′ (𝑠)𝑝(𝑠))) =
max(𝑛 + (𝑚 − 1), (𝑛 − 1) + 𝑚) = 𝑛 + 𝑚 − 1 < 2𝑛 = deg(𝑞(𝑠))2 . Hence
𝑟′ (𝑠) is a proper rational function.

43. Suppose 𝑟(𝑠) ∈ ℛ𝑞𝑛 but not in ℛ𝑞𝑛−1 . Then we can write 𝑟(𝑠) = 𝑞𝑝(𝑠)𝑛 (𝑠)
2 2
where 𝑝(𝑠) does not have a factor of 𝑞(𝑠) = (𝑠 − 𝑎) + 𝑏 . By the quotient
rule we have
𝑝′ 𝑞 𝑛 − 𝑛𝑞 𝑛−1 2(𝑠 − 𝑎)𝑝
𝑟′ (𝑠) =
𝑞 2𝑛
𝑝′ 𝑞 − 2𝑛(𝑠 − 𝑎)𝑝
= .
𝑞 𝑛+1
Thus 𝑟′ (𝑠) ∈ ℛ𝑞𝑛+1 . Suppose 𝑟′ (𝑠) ∈ ℛ𝑞𝑛 . Then the numerator 𝑝′ 𝑞 −
2𝑛(𝑠 − 𝑎)𝑝 has a factor of 𝑞. But this implies 𝑝 has a factor of 𝑞. But this
/ ℛ𝑞𝑛−1 . Hence 𝑟′ (𝑠) ∈
is impossible since 𝑟(𝑠) ∈ / ℛ𝑞 𝑛 .
1 Solutions 81

44. By exercise 43 this is true for 𝑛 = 1. Now apply induction. If 𝑛 is given


and we assume the result is true for derivatives of order 𝑛−1, then 𝑟 (𝑛−1) ∈
ℛ𝑞𝑛 but not in ℛ𝑞𝑛−1 . Another application of exercise 43 then shows that
)′
𝑟(𝑛) = 𝑟(𝑛−1) ∈ ℛ𝑞𝑛+1 but not in ℛ𝑞𝑛 .
(

45. First note the following trigonometric identities:


1
cos 𝑏𝑡 cos 𝑑𝑡 = (cos(𝑏 + 𝑑)𝑡 + cos(𝑏 − 𝑑)𝑡)
2
1
sin 𝑏𝑡 sin 𝑑𝑡 = (cos(𝑏 − 𝑑)𝑡 − cos(𝑏 + 𝑑)𝑡)
2
1
sin 𝑏𝑡 cos 𝑑𝑡 = (sin(𝑏 + 𝑑)𝑡 + sin(𝑏 − 𝑑)𝑡) .
2
Thus if 𝑡1 (𝑡) and 𝑡2 (𝑡) are two basic trig functions ( sin or cos) then
𝑡1 (𝑡)𝑡2 (𝑡) is a linear combination of basic trig functions. Now if 𝑓 (𝑡) =
𝑡𝑛 𝑒𝑎𝑡 𝑡1 (𝑡) and 𝑔(𝑡) = 𝑡𝑚 𝑒𝑐𝑡 𝑡2 (𝑡) then 𝑓 (𝑡)𝑔(𝑡) = 𝑡𝑛+𝑚 𝑒(𝑎+𝑐)𝑡 𝑡1 (𝑡)𝑡2 (𝑡).
Since 𝑡1 (𝑡)𝑡2 (𝑡) is a linear combinations of basic trig functions then 𝑓 (𝑡)𝑔(𝑡)
is a linear combination of exponential polynomial. Now if 𝑓 (𝑡) and 𝑔(𝑡)
is any exponential polynomial then they are each made up of linear com-
binations of simple exponential polynomials and their product is a sum
of terms of products of simple exponential polynomials, which we have
shown is the sum of possibly two simple exponential polynomials. It now
follows that 𝑓 (𝑡)𝑔(𝑡) is an exponential polynomial.

46. Observe that 𝑒𝑡−𝑡0 = 𝑒−𝑡0 𝑒𝑡 . So the translate of an exponential function


is a multiple of an exponential function. Also, if 𝑓 (𝑡) is a polynomial so is
𝑓 (𝑡−𝑡0 ). By the addition rule for cos we have cos 𝑏(𝑡−𝑡0 ) = cos 𝑏𝑡 cos 𝑏𝑡0 −
sin 𝑏𝑡 sin 𝑏𝑡0 and similarly for sin. It follows that all these translates remain
in ℰ. By Theorem ?? and Exercise ?? the result follows.

47. Since 𝑓 ∈ ℰ is a linear combination of terms of the form 𝑡𝑛 𝑒𝑎𝑡 cos 𝑏𝑡 and
𝑡𝑛 𝑒𝑎𝑡 sin 𝑏𝑡 it is enough to show that the derivative of each of these terms
is again in ℰ. But, by the product rule we have

(𝑡𝑛 𝑒𝑎𝑡 cos 𝑏𝑡)′ = 𝑛𝑡𝑛−1 𝑒𝑎𝑡 cos 𝑏𝑡 + 𝑎𝑡𝑛 𝑒𝑎𝑡 cos 𝑏𝑡 − 𝑏𝑡𝑛 𝑒𝑎𝑡 sin 𝑏𝑡,

a linear combination of simple exponential polynomials and hence back in


ℰ. A similar calculation holds for 𝑡𝑛 𝑒𝑎𝑡 sin 𝑏𝑡.

48. All exponential polynomials are continuous functions on ℝ. If 𝑓 (𝑡) =


𝑡 then 𝑓 is an exponential polynomial. However, 𝑓 −1 (𝑡) = 1/𝑡 is not
continuous at 0. Thus 𝑓 −1 is not an exponential polynomial.

Section 2.6
1.
82 1 Solutions
∫ 𝑡
𝑡∗𝑡 = 𝑥(𝑡 − 𝑥) 𝑑𝑥
0
∫ 𝑡
= (𝑡𝑥 − 𝑥2 ) 𝑑𝑥
0
) 𝑥=𝑡
𝑥2 𝑥3
(
= 𝑡 −
2 3 𝑥=0
𝑡2 𝑡3 𝑡3
=𝑡 − =
2 3 6

2.
∫ 𝑡
𝑡 ∗ 𝑡3 = 𝑡3 ∗ 𝑡 = 𝑥3 (𝑡 − 𝑥) 𝑑𝑥
0
∫ 𝑡
= (𝑡𝑥3 − 𝑥4 ) 𝑑𝑥
0
) 𝑥=𝑡
𝑥4 𝑥5
(
= 𝑡 −
4 5 𝑥=0
𝑡4 𝑡5
=𝑡 −
4 5
𝑡5
= .
20

3.
∫ 𝑡
3 ∗ sin 𝑡 = sin 𝑡 ∗ 3 = (sin 𝑥)(3) 𝑑𝑥
0
𝑥=𝑡
= −3 cos 𝑥∣𝑥=0
= −3(cos 𝑡 − cos 0)
= −3 cos 𝑡 + 3

4.
∫ 𝑡
4𝑡
(3𝑡 + 1) ∗ 𝑒 = (3𝑥 + 1)𝑒4(𝑡−𝑥) 𝑑𝑥
0
( ∫ 𝑡 ∫ 𝑡 )
4𝑡 −4𝑥 −4𝑥
=𝑒 3 𝑥𝑒 𝑑𝑥 + 𝑒 𝑑𝑥
( (0 0
) )
4𝑡 1 −4𝑡 1 −4𝑡 1 1 −4𝑡 1
=𝑒 3 − 𝑡𝑒 − 𝑒 + − 𝑒 +
4 16 16 4 4
7𝑒4𝑡 − 12𝑡 − 7
= .
16
1 Solutions 83

5. From the Convolution table we get


1
sin 2𝑡 ∗ 𝑒3𝑡 = (2𝑒3𝑡 − 2 cos 2𝑡 − 3 sin 2𝑡)
32 + 2 2
1
= (2𝑒3𝑡 − 2 cos 2𝑡 − 3 sin 2𝑡).
13

6. From linearity and the Convolution table we get


(2𝑡 + 1) ∗ cos 2𝑡 = 2𝑡 ∗ cos 2𝑡 + 1 ∗ cos 2𝑡
∫ 𝑡
1 − cos 2𝑡
=2 + ( cos 2𝑥 𝑑𝑥)
22 0
1 − cos 2𝑡 sin 2𝑡
= +
2 2

7. From the Convolution table we get


2
𝑡2 ∗ 𝑒−6𝑡 = (𝑒−6𝑡 − (−6 − 6𝑡 + (36𝑡2 )/2))
(−6)3
1
= (18𝑡2 − 6𝑡 − 6 + 𝑒−6𝑡 ).
108

8. From the Convolution table we get


1
cos 𝑡 ∗ cos 2𝑡 = (sin 𝑡 − 2 sin 2𝑡)
12 − 2 2
1
= (2 sin 2𝑡 − sin 𝑡).
3

9. From the Convolution table we get


𝑒2𝑡 − 𝑒−4𝑡
𝑒2𝑡 ∗ 𝑒−4𝑡 =
2 − (−4)
1 2𝑡
(𝑒 − 𝑒−4𝑡 ).
6

10.
ℒ {𝑡 ∗ 𝑡𝑛 } (𝑠) = ℒ {𝑡} (𝑠)ℒ {𝑡𝑛 } (𝑠)
1 𝑛!
= 2 𝑛+1
𝑠 𝑠
𝑛!
= 𝑛+3
𝑠
1 (𝑛 + 2)!
=
(𝑛 + 1)(𝑛 + 2) 𝑠𝑛+3
1
ℒ 𝑡𝑛+2 .
{ }
=
(𝑛 + 1)(𝑛 + 2)
84 1 Solutions

Thus
𝑡𝑛+2
𝑡 ∗ 𝑡𝑛 = .
(𝑛 + 1)(𝑛 + 2)

11.
1 𝑏
ℒ 𝑒𝑎𝑡 ∗ sin 𝑏𝑡 (𝑠) =
{ }
𝑠 − 𝑎 𝑠2 + 𝑏 2
( )
1 𝑏 𝑏𝑠 + 𝑏𝑎
= 2 − 2
𝑠 + 𝑏2 𝑠 − 𝑎 𝑠 + 𝑏2
1 ( { 𝑎𝑡 } )
= 2 𝑏ℒ 𝑒 − (𝑏ℒ {cos 𝑏𝑡} + 𝑎ℒ {sin 𝑏𝑡})
𝑠 + 𝑏2
Thus
1
𝑒𝑎𝑡 ∗ sin 𝑏𝑡 = (𝑏𝑒𝑎𝑡 − 𝑏 cos 𝑏𝑡 − 𝑎 sin 𝑏𝑡).
𝑎2 + 𝑏 2

12.
1 𝑠
ℒ 𝑒𝑎𝑡 ∗ cos 𝑏𝑡 (𝑠) =
{ }
𝑠 − 𝑎 𝑠2 + 𝑏 2
𝑎𝑠 − 𝑏2
( )
1 𝑎
= 2 −
𝑎 + 𝑏 2 𝑠 − 𝑎 𝑠2 + 𝑏 2
1 ( { 𝑎𝑡 } )
= 2 𝑎ℒ 𝑒 − (𝑎ℒ {cos 𝑏𝑡} − 𝑏ℒ {sin 𝑏𝑡})
𝑎 + 𝑏2
Thus
1
𝑒𝑎𝑡 ∗ sin 𝑏𝑡 = (𝑎𝑒𝑎𝑡 − 𝑎 cos 𝑏𝑡 + 𝑏 sin 𝑏𝑡).
𝑎2 + 𝑏2

13. First assume 𝑎 ∕= 𝑏. Then


𝑎 𝑏
ℒ {sin 𝑎𝑡 ∗ sin 𝑏𝑡} =
𝑠2+ 𝑎 𝑠 + 𝑏2
2 2
( )
1 𝑎𝑏 𝑎𝑏
= 2 − 2
𝑏 − 𝑎 2 𝑠2 + 𝑎 2 𝑠 + 𝑏2

From this it follows that


1
sin 𝑎𝑡 ∗ sin 𝑏𝑡 = (𝑏 sin 𝑎𝑡 − 𝑎 sin 𝑏𝑡).
𝑏2 − 𝑎 2
Now assume 𝑎 = 𝑏. Then
𝑎2
ℒ {sin 𝑎𝑡 ∗ sin 𝑎𝑡} =
(𝑠2 + 𝑎 2 )2
By Table 2.4 in Section 2.4 we get
1 Solutions 85

1
ℒ {sin 𝑎𝑡 ∗ sin 𝑎𝑡} = (sin 𝑎𝑡 − 𝑎𝑡 cos 𝑎𝑡).
2𝑎

14. First assume 𝑎 ∕= 𝑏. Then


𝑎 𝑠
ℒ {sin 𝑎𝑡 ∗ cos 𝑏𝑡} =
𝑠2 + 𝑎 2 𝑠2 + 𝑏 2
( )
1 𝑎𝑠 𝑎𝑠
= 2 −
𝑏 − 𝑎 2 𝑠2 + 𝑎 2 𝑠2 + 𝑏 2

From this it follows that


1
sin 𝑎𝑡 ∗ cos 𝑏𝑡 = (𝑎 cos 𝑎𝑡 − 𝑎 cos 𝑏𝑡).
𝑏2 − 𝑎2
Now assume 𝑎 = 𝑏. Then
𝑎𝑠
ℒ {sin 𝑎𝑡 ∗ cos 𝑎𝑡} =
(𝑠2 + 𝑎2 )2
By Table 2.4 in Section 2.4 we get
1
ℒ {sin 𝑎𝑡 ∗ cos 𝑎𝑡} = (𝑎𝑡 sin 𝑎𝑡).
2𝑎

15. First assume 𝑎 ∕= 𝑏. Then


𝑠 𝑠
ℒ {cos 𝑎𝑡 ∗ cos 𝑏𝑡} =
𝑠2 + 𝑎 2 𝑠2 + 𝑏 2
−𝑎2 𝑏2
( )
1
= 2 + 2
𝑏 − 𝑎 2 𝑠2 + 𝑎 2 𝑠 + 𝑏2

From this it follows that


1
cos 𝑎𝑡 ∗ cos 𝑏𝑡 = (−𝑎 sin 𝑎𝑡 + 𝑏 sin 𝑏𝑡).
𝑏2 − 𝑎 2
Now assume 𝑎 = 𝑏. Then
𝑠2
ℒ {cos 𝑎𝑡 ∗ cos 𝑎𝑡} =
(𝑠2 + 𝑎2 )2
1 𝑎2
= 2 2
− 2 .
𝑠 +𝑎 (𝑠 + 𝑎2 )2
By Table 2.4 we get
1 1 1
ℒ {cos 𝑎𝑡 ∗ cos 𝑎𝑡} = sin 𝑎𝑡− (sin 𝑎𝑡−𝑎𝑡 cos 𝑎𝑡) = (sin 𝑎𝑡+𝑎𝑡 cos 𝑎𝑡).
𝑎 2𝑎 2𝑎
86 1 Solutions

16. The key is to recognize the integral defining 𝑓 (𝑡) as the convolution
integral of two functions. Thus 𝑓 (𝑡) = (cos 2𝑡) ∗ 𝑡 so that 𝐹 (𝑠) =
𝑠 1 1
ℒ {(cos 2𝑡) ∗ 𝑡} = ℒ {cos 2𝑡} ℒ {𝑡} = 2 2
= 2
.
𝑠 +4𝑠 𝑠(𝑠 + 4)
2 2 4
17. 𝑓 (𝑡) = 𝑡2 ∗ sin 2𝑡 so 𝐹 (𝑠) = 3 2
= 3 2 .
𝑠 𝑠 +4 𝑠 (𝑠 + 4)
6 1 6
18. 𝑓 (𝑡) = 𝑡3 ∗ 𝑒−3𝑡 so 𝐹 (𝑠) = 4
= 4
𝑠 𝑠+3 𝑠 (𝑠 + 3)
6 1 6
19. 𝑓 (𝑡) = 𝑡3 ∗ 𝑒−3𝑡 so 𝐹 (𝑠) = 4
= 4
𝑠 𝑠+3 𝑠 (𝑠 + 3)
2 𝑠 2𝑠
20. 𝑓 (𝑡) = sin 2𝑡 ∗ cos 𝑡 so 𝐹 (𝑠) = = 2
𝑠2 2
+4𝑠 +1 (𝑠 + 4)(𝑠2 + 1)
2 2 4
21. 𝑓 (𝑡) = sin 2𝑡 ∗ sin 2𝑡 so 𝐹 (𝑠) = 2
𝑠2 + 2 2 𝑠2 + 2 2 (𝑠 + 4)2
22.
{ } { } { }
−1 1 −1 1 −1 1
ℒ =ℒ ∗ℒ
(𝑠 − 2)(𝑠 + 4) 𝑠−2 𝑠+4
= 𝑒2𝑡 ∗ 𝑒−4𝑡
𝑒2𝑡 − 𝑒−4𝑡
=
2 − −4
1 2𝑡
= (𝑒 − 𝑒−4𝑡 ).
6

23.
{ } { }
−1 1 −1 1
ℒ 2
=ℒ
𝑠 − 6𝑠 + 5 (𝑠 − 1)(𝑠 − 5)
{ } { }
−1 1 −1 1
=ℒ ∗ℒ
𝑠−1 𝑠−5
= 𝑒𝑡 ∗ 𝑒5𝑡
𝑒𝑡 − 𝑒5𝑡
=
1−5
1
= (−𝑒𝑡 + 𝑒5𝑡 )
4

24.
1 Solutions 87
{ } { } { }
1 1 1
ℒ−1 = ℒ−1 ∗ ℒ−1
(𝑠2 + 1)2 𝑠2
+1 𝑠2 +1
= sin 𝑡 ∗ sin 𝑡
1
= (sin 𝑡 − 𝑡 cos 𝑡)
2

25.
{ } { } { }
−1 𝑠 −1 1 −1 𝑠
ℒ =ℒ ∗ℒ
(𝑠2 + 1)2 𝑠2 + 1 𝑠2 + 1
= sin 𝑡 ∗ cos 𝑡
1
= 𝑡 sin 𝑡
2

26.
{ } { } { }
−1 1 −1 1 −1 1
ℒ =ℒ ∗ℒ
(𝑠 + 6)𝑠3 𝑠+6 𝑠3
2
𝑡
= 𝑒−6𝑡 ∗
2
1
= (−𝑒−6𝑡 + 1 − 6𝑡 + 18𝑡2 )
216

27.
{ } { } { }
2 1 2
ℒ−1 = ℒ−1 ∗ ℒ−1
(𝑠 − 3)(𝑠2 + 4) 𝑠−3 2
𝑠 +4
= 𝑒3𝑡 ∗ sin 2𝑡
1
= (2𝑒3𝑡 − 2 cos 2𝑡 − 3 sin 2𝑡)
13

28.
{ } { } { }
−1 𝑠 −1 1 −1 𝑠
ℒ =ℒ ∗ℒ
(𝑠 − 4)(𝑠2 + 1) 𝑠−4 2
𝑠 +1
= 𝑒4𝑡 ∗ cos 𝑡
1
= (4𝑒4𝑡 − 4 cos 𝑡 + sin 𝑡)
17

29.
{ } { } { }
−1 1 −1 1 −1 1
ℒ =ℒ ∗ℒ
(𝑠 − 𝑎)(𝑠 − 𝑏) 𝑠−𝑎 𝑠−𝑏
= 𝑒𝑎𝑡 ∗ 𝑒𝑏𝑡
𝑒𝑎𝑡 − 𝑒𝑏𝑡
= .
𝑎−𝑏
88 1 Solutions

30.
{ } { }
𝐺(𝑠) 1
ℒ−1 = ℒ−1 {𝐺(𝑠)} ∗ ℒ−1
𝑠+2 𝑠+2
= 𝑔(𝑡) ∗ 𝑒−2𝑡
∫ 𝑡
= 𝑔(𝑥)𝑒−2(𝑡−𝑥) 𝑑𝑥
0

31.
{ } { }
−1 𝐺(𝑠) −1 −1 𝑠
ℒ =ℒ {𝐺(𝑠)} ∗ ℒ √ 2
𝑠2 + 2 2
𝑠 + 2

= 𝑔(𝑡) ∗ cos( 2)𝑡
∫ 𝑡 √
= 𝑔(𝑥) cos 2(𝑡 − 𝑥) 𝑑𝑥
0

32. Hint: use the Input Integral Principle repeatedly.

33. We apply the input integral principle twice:


{ } ∫ 𝑡
−1 1
ℒ = sin 𝑥 𝑑𝑥
𝑠(𝑠2 + 1) 0
𝑡
= − cos 𝑥∣0
= − cos 𝑡 + 1

{ } 𝑡
1

ℒ−1 = 1 − cos 𝑥 𝑑𝑥
𝑠2 (𝑠2 + 1) 0
= 𝑡 − sin 𝑡 𝑑𝑥

1 1 1
34. The inverse Laplace transform of = is 𝑒2𝑡 ∗ 𝑒−2𝑡 =
𝑠2 − 4 𝑠−2𝑠+2
𝑒2𝑡 − 𝑒−2𝑡
. Thus
4
{ } 𝑡
1 1

ℒ−1 = (𝑒2𝑥 − 𝑒−2𝑥 ) 𝑑𝑥
𝑠(𝑠2 − 4) 4
(0 𝑡 𝑡 )
1 𝑒2𝑥 𝑒−2𝑥
= +
4 2 0 2 0
1 2𝑡
= (𝑒 − 1 + 𝑒−2𝑡 − 1)
8
1 1 1
= − + 𝑒2𝑡 + 𝑒−2𝑡
4 8 8
1 Solutions 89

Repeating the input integral principle we get


{ } ∫ 𝑡( )
1 1 1 2𝑥 1 −2𝑥
ℒ−1 = − + 𝑒 + 𝑒 𝑑𝑥
𝑠2 (𝑠2 − 4) 0 4 8 8
𝑡 1 1
= − + (𝑒2𝑡 − 1) − (𝑒−2𝑡 − 1)
4 16 16
−𝑡 1 1
= + 𝑒2𝑡 − 𝑒−2𝑡
4 16 16

35. We apply the input integral principle three times:


{ } ∫ 𝑡
−1 1
ℒ = 𝑒−3𝑥 𝑑𝑥
𝑠(𝑠 + 3) 0
𝑡
𝑒−3𝑥
=
−3 0
1
= (1 − 𝑒−3𝑡 ).
3

1 𝑡
{ }
1

ℒ−1 2
= 1 − 𝑒−3𝑥 𝑑𝑥
𝑠 (𝑠 + 3) 3 0
1 − 𝑒−3𝑡
( )
1
= 𝑡−
3 3
1
= (3𝑡 − 1 + 𝑒−3𝑡 ).
9

1 𝑡
{ }
1

−1
ℒ = 3𝑥 − 1 + 𝑒−3𝑥 𝑑𝑥
𝑠3 (𝑠 + 3) 9 0
( 2
𝑒−3𝑡 − 1
)
1 𝑡
= 3 −𝑡−
9 2 3
1
= (2 − 6𝑡 + 9𝑡2 − 2𝑒−3𝑡 )
54

36. We apply the input integral principle twice:


{ } ∫ 𝑡
−1 1
ℒ = 𝑥𝑒2𝑥 𝑑𝑥
𝑠(𝑠 − 2)2 0
𝑡
1 2𝑥 2𝑥

= (2𝑥𝑒 − 𝑒 )
4 0
1 2𝑡 2𝑡
= (2𝑡𝑒 − 𝑒 + 1)
4
90 1 Solutions
{ } 𝑡
1 1

ℒ−1 = 2𝑥𝑒2𝑥 − 𝑒2𝑥 + 1 𝑑𝑥
𝑠2 (𝑠 − 2)2 4 0
𝑡
1 2𝑥 2𝑥

= (𝑥𝑒 − 𝑒 + 𝑥)
4 0
1 2𝑡
= (𝑡𝑒 − 𝑒2𝑡 + 𝑡 + 1)
4
{ }
−1 1 1
37. First, ℒ = (sin 3𝑡 − 3𝑡 cos 3𝑡). Thus
(𝑠 + 9)2
2 54
{ } ∫ 𝑡
1 1
ℒ−1 = (sin 3𝑥 − 3𝑥 cos 3𝑥) 𝑑𝑥
𝑠(𝑠 + 9)2
2 54 0
( ( )) 𝑡
1 cos 3𝑥 cos 3𝑥
= − − 𝑥 sin 3𝑥 +
54 3 3
0
( )
1 2 cos 3𝑡 2
= − − 𝑡 sin 3𝑡 +
54 3 3
1
= (−2 cos 3𝑡 − 3𝑡 sin 3𝑡 + 2).
162

1 1
38. First, from Table 2.4, = 2 . We apply the input integral
𝑠3 + 𝑠 2 𝑠 (𝑠 + 1)
principle twice:
{ } 𝑡
1

−1
ℒ = 𝑒−𝑥 𝑑𝑥
𝑠(𝑠 + 1) 0
𝑡
= −𝑒−𝑥 0
= 1 − 𝑒−𝑡 .
{ } 𝑡
1

ℒ−1 = (1 − 𝑒−𝑥 ) 𝑑𝑥
𝑠2 (𝑠 + 1) 0
) 𝑡
= 𝑥 + 𝑒−𝑥 0
(

= 𝑡 + 𝑒−𝑡 − 1.

Section 2.7
1
(𝑡4 − 45𝑡2 + 105) sin 𝑡 + (10𝑡3 − 105𝑡) cos 𝑡
( )
1. 384

1
( 4
(𝑡 − 15𝑡2 ) cos 𝑡 − (6𝑡3 − 15𝑡) sin 𝑡
)
2. 384
1
(945 − 420𝑡2 + 15𝑡4 ) sin 𝑡 − (945𝑡 − 105𝑡3 + 𝑡4 ) cos 𝑡
( )
3. 3840
1
(105𝑡 − 45𝑡3 + 𝑡5 ) sin 𝑡 − (105𝑡2 − 10𝑡4 ) cos 𝑡
( )
4. 3840
1 Solutions 91

Section 3.1
12 𝑠+4 1 1 1
1. 𝑌 (𝑠) = (𝑠−2)(𝑠+1)(𝑠+2) + (𝑠+1)(𝑠+2) = 𝑠−2 + (𝑠+2) + 𝑠+1 and 𝑦(𝑡) = 𝑒2𝑡 +
−2𝑡 −𝑡
𝑒 −𝑒
−𝑠+4 25 −5 5 4
2. 𝑌 (𝑠) = (𝑠+1)(𝑠−5) + 𝑠2 (𝑠+1)(𝑠−5) = 𝑠2 − 𝑠+1 + 𝑠 and 𝑦(𝑡) = 4 − 5𝑡 − 5𝑒−𝑡
2𝑠+1 8 1 2 1
3. 𝑌 (𝑠) = 𝑠2 +4 + 𝑠(𝑠2 +4) = 𝑠2 +4 + 𝑠 and 𝑦(𝑡) = 2 sin 2𝑡 + 2
1 1
4. 𝑌 (𝑠) = (𝑠+2)2 + (𝑠+2)3 and 𝑦(𝑡) = 𝑡𝑒−2𝑡 + 21 𝑡2 𝑒−2𝑡
1 4𝑠 1 1
5. 𝑌 (𝑠) = (𝑠+2)2 + (𝑠2 +4)(𝑠+2)2 = 𝑠2 +4 and 𝑦(𝑡) = 2 sin 2𝑡
2𝑠−3 4 2 3 3
6. 𝑌 (𝑠) = (𝑠−1)(𝑠−2) + 𝑠(𝑠−1)(𝑠−2) = 𝑠 + 𝑠−3 − 𝑠−1 and 𝑦(𝑡) = 2 + 3𝑒2𝑡 − 3𝑒𝑡
−3𝑠+9 1 −7 4 1
7. 𝑌 (𝑠) = (𝑠−1)(𝑠−2) + (𝑠−1)2 (𝑠−2) = 𝑠−1 + 𝑠−2 − (𝑠−1)2 and 𝑦(𝑡) = −𝑡𝑒𝑡 +
4𝑒2𝑡 − 7𝑒𝑡
26 −1 1 13 1 1 −4𝑠−14 13 𝑡
8. 𝑌 (𝑠) = (𝑠2 +4)(𝑠+3)(𝑠−1) = 2 𝑠+3 + 10 𝑠−1 + 5 𝑠2 +4 and 𝑦(𝑡) = 20 𝑒 −
1 −3𝑡 4 7
2 𝑒 − 5 cos 2𝑡 − 5 sin 2𝑡
2 50 3 −3𝑠+4 7
9. 𝑌 (𝑠) = (𝑠+3) 2 + (𝑠2 +1)(𝑠+3)2 = 𝑠+3 + 𝑠2 +1 + (𝑠+3)2 and 𝑦(𝑡) = 3𝑒−3𝑡 +
7𝑡𝑒−3𝑡 − 3 cos 𝑡 + 4 sin 𝑡
𝑠−1 −1
10. 𝑌 (𝑠) = 𝑠2 +25 and 𝑦(𝑡) = 5 sin 5𝑡 + cos 5𝑡
𝑠+12
11. 𝑌 (𝑠) = 2(𝑠+4)2 and 𝑦(𝑡) = 12 𝑒−4𝑡 + 4𝑡𝑒−4𝑡
−𝑠 4 1 2 4 2𝑡
12. 𝑌 (𝑠) = (𝑠−2)2 + (𝑠−2) 3 = − 𝑠−2 + (𝑠−2)2 + (𝑠−2)3 and 𝑦(𝑡) = −𝑒 +2𝑡𝑒2𝑡 +
2𝑡2 𝑒2𝑡
√ (√ ) (√ )
𝑠+1 3 −𝑡 3𝑡 −𝑡
3𝑡
13. 𝑌 (𝑠) = 𝑠2 +𝑠+1 and 𝑦(𝑡) = 3 𝑒
2 sin 2 +𝑒 2 cos 2

𝑠+4 64𝑠 16 𝑠
14. 𝑌 (𝑠) = (𝑠+2) 2 + (𝑠2 +4)2 (𝑠+2)2 = (𝑠2 +4)2 + 𝑠2 +4 and 𝑦(𝑡) = −2𝑡 cos 2𝑡 +
cos 2𝑡 + sin 2𝑡
1 3 8 1
15. 𝑌 (𝑠) = 𝑠2 +4 + (𝑠2 +9)(𝑠 2 +4) = 5 𝑠2 +4 − 35 𝑠21+9 and 𝑦(𝑡) = 4
5 sin 2𝑡 − 15 sin 3𝑡

Section 3.2
1. no, not linear.

2. yes; (𝑫 2 − 3𝑫)(𝑦) = 𝑒𝑡 , 𝑞(𝑠) = 𝑠2 − 3𝑠, nonhomogeneous

3. no, third order.

4. no, not linear.


92 1 Solutions

5. no, not constant coefficient.

6. yes; (𝑫 2 + 2𝑫 + 3)(𝑦) = 𝑒−𝑡 , 𝑞(𝑠) = 𝑠2 + 2𝑠 + 3, nonhomogeneous

7. yes; (𝑫 2 − 7𝑫 + 10)(𝑦) = 0, 𝑞(𝑠) = 𝑠2 − 7𝑠 + 10, homogeneous

8. no, first order.

9. yes; 𝑫 2 (𝑦) = −2 + cos 𝑡, 𝑞(𝑠) = 𝑠2 , nonhomogeneous

10. yes; (2𝑫 2 − 12𝑫 + 18)(𝑦) = 0, 𝑞(𝑠) = 2𝑠2 − 12𝑠 + 18, homogeneous

11. (a) 6𝑒𝑡


(b) 0
(c) sin 𝑡 − 3 cos 𝑡

12. (a) 0
(b) 2 sin 𝑡
(c) −𝑒2𝑡

13. (a) −4𝑒−2𝑡


(b) 0
(c) sec2 𝑡 − 2 tan 𝑡

14. (a) 0
(b) 0
(c) 1

15. (a) 4𝑒2𝑡


(b) 0
(c) 0

16. 𝑦(𝑡) = cos 2𝑡 + 𝑐1 𝑒𝑡 + 𝑐2 𝑒4𝑡 where 𝑐1 , 𝑐2 are arbitrary constants.

17. 𝑦(𝑡) = 𝑡𝑒3𝑡 + 𝑐1 𝑒3𝑡 + 𝑐2 𝑒−2𝑡 where 𝑐1 , 𝑐2 are arbitrary constants.

18. 𝑦(𝑡) = cos 2𝑡 + 𝑒𝑡 − 𝑒4𝑡

19. 𝑦(𝑡) = 𝑡𝑒3𝑡 + 𝑒3𝑡 − 2𝑒−2𝑡

Section 3.3
1. 𝑦(𝑡) = 𝑐1 𝑒2𝑡 + 𝑐2 𝑒−𝑡

2. 𝑦(𝑡) = 𝑐1 𝑒−4𝑡 + 𝑐2 𝑒3𝑡

3. 𝑦(𝑡) = 𝑐1 𝑒−4𝑡 + 𝑐2 𝑒−6𝑡

4. 𝑦(𝑡) = 𝑐1 𝑒6𝑡 + 𝑐2 𝑒−2𝑡


1 Solutions 93

5. 𝑦(𝑡) = 𝑐1 𝑒−4𝑡 + 𝑐2 𝑡𝑒−4𝑡

6. 𝑦(𝑡) = 𝑐1 𝑒5𝑡 + 𝑐2 𝑒−2𝑡

7. 𝑦(𝑡) = 𝑐1 𝑒−𝑡 cos 2𝑡 + 𝑐2 𝑒−𝑡 sin 2𝑡

8. 𝑦(𝑡) = 𝑐1 𝑒3𝑡 + 𝑐2 𝑡𝑒3𝑡

9. 𝑦(𝑡) = 𝑐1 𝑒−9𝑡 + 𝑐2 𝑒−4𝑡

10. 𝑦(𝑡) = 𝑐1 𝑒−4𝑡 cos 3𝑡 + 𝑐2 𝑒−4𝑡 sin 3𝑡

11. 𝑦(𝑡) = 𝑐1 𝑒−5𝑡 + 𝑐2 𝑡𝑒−5𝑡

12. 𝑦(𝑡) = 𝑐1 𝑒7𝑡 + 𝑐2 𝑒−3𝑡


𝑒𝑡 −𝑒−𝑡
13. 𝑦 = 2

14. 𝑦 = 2𝑒5𝑡 + 3𝑒−2𝑡

15. 𝑦 = 𝑡𝑒5𝑡

16. 𝑦 = 𝑒−2𝑡 cos 3𝑡 − 𝑒−2𝑡 sin 3𝑡

Section 3.4
− 2)(𝑠 − 3) so ℬ𝑞𝑣 = 𝑒−𝑡 , 𝑒2𝑡 , 𝑒3𝑡 while ℬ𝑞 =
{ }
1. 𝑞(𝑠)𝑣(𝑠)
{ −𝑡 2𝑡 }= (𝑠 + 1)(𝑠
𝑒 , 𝑒 . Since 𝑒3𝑡 is the only function in the first set but not in the
second 𝑦𝑝 (𝑡) = 𝑐1 𝑒3𝑡 .

= }(𝑠 + 2)(𝑠 + 4)(𝑠 + 3) so ℬ𝑞𝑣 = 𝑒−2𝑡 , 𝑒−4𝑡 , 𝑒−3𝑡 while ℬ𝑞 =


{ }
2. 𝑞(𝑠)𝑣(𝑠)
{ −2𝑡 −4𝑡
𝑒 ,𝑒 . Since 𝑒−3𝑡 is the only function in the first set but not in the
second 𝑦𝑝 (𝑡) = 𝑐1 𝑒−3𝑡 .

3. 𝑞(𝑠)𝑣(𝑠) = (𝑠 − 2)2 (𝑠 − 3) so ℬ𝑞𝑣 = 𝑒2𝑡 , 𝑡𝑒2𝑡 , 𝑒3𝑡 while ℬ𝑞 = 𝑒2𝑡 , 𝑒3𝑡 .


{ } { }
Since 𝑡𝑒2𝑡 is the only function in the first set but not in the second 𝑦𝑝 (𝑡) =
𝑐1 𝑡𝑒2𝑡 .
3
ℬ𝑞𝑣 = 𝑒4𝑡 , 𝑡𝑒4𝑡 , 𝑡2 𝑒4𝑡 , 𝑒3𝑡 while ℬ𝑞 =
{ }
{ 4𝑡 3𝑡 }= (𝑠 − 4)4𝑡(𝑠 − 3)2 so
4. 𝑞(𝑠)𝑣(𝑠)
𝑒 , 𝑒 . Since 𝑡𝑒 and 𝑡 𝑒4𝑡 are the only functions in the first set but
not in the second 𝑦𝑝 (𝑡) = 𝑐1 𝑡𝑒4𝑡 + 𝑐2 𝑡2 𝑒4𝑡 .
2 2
{ 5𝑡 5𝑡 }
5. 𝑞(𝑠)𝑣(𝑠)
{ 5𝑡 5𝑡= } (𝑠 − 5) (𝑠 + 25) so ℬ𝑞𝑣 = 𝑒 , 𝑡𝑒 , cos 5𝑡, sin 5𝑡 while ℬ𝑞 =
𝑒 , 𝑡𝑒 . Since cos 5𝑡 and sin 5𝑡 are the only functions in the first set
that are not in the second 𝑦𝑝 (𝑡) = 𝑐1 cos 5𝑡 + 𝑐2 sin 5𝑡.

6. 𝑞(𝑠)𝑣(𝑠) = (𝑠2 + 1)(𝑠2 + 4) so ℬ𝑞𝑣 = {cos 𝑡, sin 𝑡, cos 2𝑡, sin 2𝑡} while ℬ𝑞 =
{cos 𝑡, sin 𝑡}. Since cos 2𝑡 and sin 2𝑡 are the only functions in the first set
that are not in the second 𝑦𝑝 (𝑡) = 𝑐1 cos 2𝑡 + 𝑐2 sin 2𝑡.
94 1 Solutions

7. 𝑞(𝑠)𝑣(𝑠) = (𝑠2 + 4)2 so ℬ𝑞𝑣 = {cos 2𝑡, sin 2𝑡, 𝑡 cos 2𝑡, 𝑡 sin 2𝑡} while ℬ𝑞 =
{cos 2𝑡, sin 2𝑡}. Since 𝑡 cos 2𝑡 and 𝑡 sin 2𝑡 are the only functions in the first
set that are not in the second 𝑦𝑝 (𝑡) = 𝑐1 𝑡 cos 2𝑡 + 𝑐2 𝑡 sin 2𝑡.

(𝑠2 + 4𝑠 + 5)(𝑠 − 1)}2 so ℬ𝑞𝑣 = 𝑒𝑡 , 𝑡𝑒𝑡 , 𝑒−2𝑡 cos 𝑡, 𝑒−2𝑡 sin 𝑡


{ }
8. 𝑞(𝑠)𝑣(𝑠) = {
while ℬ𝑞 = 𝑒−2𝑡 cos 𝑡, 𝑒−2𝑡 sin 𝑡 . Since 𝑒𝑡 and 𝑡𝑒𝑡 are the only functions
in the first set that are not in the second 𝑦𝑝 (𝑡) = 𝑐1 𝑒𝑡 + 𝑐2 𝑡𝑒𝑡 .

9. 𝑞(𝑠)𝑣(𝑠) = (𝑠2 + 4𝑠 + 5)(𝑠 − 1)2 so ℬ𝑞𝑣 = 𝑒𝑡 , 𝑡𝑒𝑡 , 𝑒−2𝑡 cos 𝑡, 𝑒−2𝑡 sin 𝑡
{ }

while ℬ𝑞 = {𝑒𝑡 , 𝑡𝑒𝑡 }. Since 𝑒−2𝑡 cos 𝑡 and 𝑒−2𝑡 sin 𝑡 are the only functions
in the first set that are not in the second 𝑦𝑝 (𝑡) = 𝑐1 𝑒−2𝑡 cos 𝑡 + 𝑐2 𝑒−2𝑡 sin 𝑡.

10. 𝑦 = 61 𝑒2𝑡 + 𝐴𝑒𝑡 + 𝐵𝑒−4𝑡

11. 𝑦 = −𝑡𝑒−2𝑡 + 𝐴𝑒−2𝑡 + 𝐵𝑒5𝑡

12. 𝑦 = 41 𝑒𝑡 + 𝐴𝑒−𝑡 + 𝐵𝑡𝑒−𝑡

13. 𝑦 = 2 + 𝐴𝑒−𝑡 + 𝐵𝑒−2𝑡

14. 𝑦 = 12 𝑒−3𝑡 + 𝐴𝑒−2𝑡 cos 𝑡 + 𝐵𝑒−2𝑡 sin 𝑡


1
15. 𝑦 = 4 + 15 𝑒𝑡 + 𝐴 cos(2𝑡) + 𝐵 sin(2𝑡)

16. 𝑦 = −𝑡2 − 2 + 𝐴𝑒𝑡 + 𝐵𝑒−𝑡

17. 𝑦 = 𝑒𝑡 + 𝐴𝑒2𝑡 + 𝐵𝑡𝑒2𝑡

18. 𝑦 = 21 𝑡2 𝑒2𝑡 + 𝐴𝑒2𝑡 + 𝐵𝑡𝑒2𝑡

19. 𝑦 = −𝑡 cos 𝑡 + 𝐴 sin 𝑡 + 𝐵 cos 𝑡

20. 𝑦 = 𝑡𝑒2𝑡 − 52 𝑒2𝑡 + 𝐴𝑒−3𝑡 + 𝐵𝑡𝑒−3𝑡


25 3 −3𝑡
21. 𝑦 = 6 𝑡 𝑒 + 𝐴𝑒−3𝑡 + 𝐵𝑡𝑒−3𝑡

22. 𝑦 = 14 𝑡𝑒−3𝑡 sin(2𝑡) + 𝐴𝑒−3𝑡 cos(2𝑡) + 𝐵𝑒−3𝑡 sin(2𝑡)

23. 𝑦 = −𝑡2 𝑒4𝑡 cos(3𝑡) + 𝑡𝑒4𝑡 sin(3𝑡) + 𝐴𝑒4𝑡 cos(3𝑡) + 𝐵𝑒4𝑡 sin 3𝑡
−1 3𝑡 10 6𝑡 135 −𝑡
24. 𝑦 = 12 𝑒 + 21 𝑒 + 84 𝑒

25. 𝑦 = 2𝑒−𝑡 − 2𝑒−𝑡 cos 2𝑡 + 4𝑒−𝑡 sin 2𝑡

26. 𝑦 = 2𝑒2𝑡 − 2 cos 𝑡 − 4 sin 𝑡


−1 −2𝑡 1
27. 𝑦 = 2 𝑒 + 𝑒2𝑡 + 2𝑡 − 2

28. 𝑫 − 5.

29. (𝑫 + 6)2 = 𝑫2 + 12𝑫 + 36.


1 Solutions 95

30. 𝑫2 + 9.

31. (𝑫 − 2)2 + 16 = 𝑫 2 − 4𝑫 + 20.

32. (𝑫2 + 1)3 = 𝑫3 + 3𝑫2 + 3𝑫 + 1.

33. (𝑫 − 𝑎)𝑛+1 .

Section 3.5
1 −6𝑡
1. 𝑦 = 32 𝑒 + 𝐴𝑒2𝑡 + 𝐵𝑒−2𝑡

2. 𝑦 = −𝑒𝑡 + 𝐴𝑒−3𝑡 + 𝐵𝑒5𝑡

3. 𝑦 = 𝑡𝑒−2𝑡 + 𝐴𝑒−2𝑡 + 𝐵𝑒−3𝑡

4. 𝑦 = 2 + 𝐴𝑒−𝑡 + 𝐵𝑒−2𝑡

5. 𝑦 = −𝑡𝑒−4𝑡 + 𝐴𝑒2𝑡 + 𝐵𝑒−4𝑡


3 11
6. 𝑦 = − 130 cos 𝑡 − 130 sin 𝑡

7. 𝑦 = −𝑡2 + 𝐴 + 𝐵𝑒−3𝑡 + 𝐶𝑒3𝑡

8. 𝑦 = 𝑡𝑒2𝑡 − 52 𝑒2𝑡 + 𝐴𝑒−3𝑡 + 𝐵𝑡𝑒−3𝑡


3 4𝑡
9. 𝑦 = −𝑡𝑒4𝑡 − 10 𝑒 + 𝐴𝑒−𝑡 + 𝐵𝑒6𝑡

10. 𝑦 = −𝑡2 𝑒4𝑡 cos(3𝑡) + 𝑡𝑒4𝑡 sin(3𝑡) + 𝐴𝑒4𝑡 cos(3𝑡) + 𝐵𝑒4𝑡 sin 3𝑡

11. 𝑦 = 61 𝑡3 𝑒2𝑡 + 𝐴𝑒2𝑡 + 𝐵𝑡𝑒2𝑡


1
12. 𝑦 = 2 sin 𝑡 + 𝐴𝑒−𝑡 + 𝐵𝑡𝑒−𝑡

13. 𝑦 = 12 𝑡𝑒𝑡 sin 𝑡

Section 4.1
1. no

2. yes, yes, homogeneous

3. yes, yes, nonhomogeneous

4. no

5. yes, yes, nonhomogeneous

6. yes, yes, nonhomogeneous


96 1 Solutions

7. no

8. yes, yes, nonhomogeneous

9. yes, no, homogeneous

10. no

11. yes, no, homogeneous

12. yes, no, homogeneous

13. 𝑳(1) = 1′′ + 1 = 1


𝑳(𝑡) = 𝑡′′ + 𝑡 = 𝑡
𝑳(𝑒−𝑡 ) = (𝑒−𝑡 )′′ + 𝑒−𝑡 = 2𝑒−𝑡
𝑳(cos 2𝑡) = (cos 2𝑡)′′ + cos 2𝑡 = −4 cos 2𝑡 + cos 2𝑡 = −3 cos 2𝑡.

14. 𝑳(1) = 1
𝑳(𝑡) = 𝑡
𝑳(𝑒−𝑡 ) = (𝑡 + 1)𝑒−𝑡
𝑳(cos 2𝑡) = (−4𝑡 + 1) cos 2𝑡

15. 𝑳(1) = −3
𝑳(𝑡) = 1 − 3𝑡
𝑳(𝑒−𝑡 ) = −2𝑒−𝑡
𝑳(cos 2𝑡) = −11 cos 2𝑡 − 2 sin 2𝑡

16. 𝑳(1) = 5
𝑳(𝑡) = 5𝑡 + 6
𝑳(𝑒−𝑡 ) = 0
𝑳(cos 2𝑡) = cos 2𝑡 − 12 sin 2𝑡

17. 𝑳(1) = −4
𝑳(𝑡) = −4𝑡
𝑳(𝑒−𝑡 ) = −3𝑒−𝑡
𝑳(cos 2𝑡) = −8 cos 2𝑡

18. 𝑳(1) = −1
𝑳(𝑡) = 0
𝑳(𝑒−𝑡 ) = (𝑡2 − 𝑡 − 1)𝑒−𝑡
𝑳(cos 2𝑡) = (−4𝑡2 − 1) cos 2𝑡 − 2𝑡 sin 2𝑡

19. 𝑳(𝑒𝑟𝑡 ) = 𝑎(𝑒𝑟𝑡 )′′ + 𝑏(𝑒𝑟𝑡 )′ + 𝑐𝑒𝑟𝑡 = 𝑎𝑟2 𝑒𝑟𝑡 + 𝑏𝑟𝑒𝑟𝑡 + 𝑐𝑒𝑟𝑡 = (𝑎𝑟2 + 𝑏𝑟 + 𝑐)𝑒𝑟𝑡
−4
20. 𝐶 = 3
−3 1
21. 𝐶1 = 4 and 𝐶2 = 2

22. no
1 Solutions 97

23. yes, 𝐶 = 1.

24. Parts (1) and (2) are done by computing 𝑦 ′′ + 𝑦 where 𝑦(𝑡) = 𝑡2 − 2,
𝑦(𝑡) = cos 𝑡, or 𝑦(𝑡) = sin 𝑡. Then by Theorem 3, every function of the
form 𝑦(𝑡) = 𝑡2 − 2 + 𝑐1 cos 𝑡 + 𝑐2 sin 𝑡 is a solution to 𝑦 ′′ + 𝑦 = 𝑡2 , where
𝑐1 and 𝑐2 are constants. If we want a solution to 𝑳(𝑦) = 𝑡2 with 𝑦(0) = 𝑎
and 𝑦 ′ (0) = 𝑏, then we need to solve for 𝑐1 and 𝑐2 :

𝑎 = 𝑦(0) = −2 + 𝑐1
𝑏 = 𝑦 ′ (0) = 𝑐2

These equations give 𝑐1 = 𝑎 + 2, 𝑐2 = 𝑏. Particular choices of 𝑎 and 𝑏 give


the answers for Part (3).

(3)a. 𝑦 = 21 𝑒𝑡 + 2𝑒2𝑡 − 32 𝑒3𝑡


(3)b. 𝑦 = 21 𝑒𝑡 − 2𝑒2𝑡 + 32 𝑒3𝑡
(3)c. 𝑦 = 21 𝑒𝑡 − 7𝑒2𝑡 + 11 3𝑡
2 𝑒
(3)d. 𝑦 = 21 𝑒𝑡 + (−1 + 3𝑎 − 𝑏)𝑒2𝑡 +
(1
− 2𝑎 + 𝑏 𝑒3𝑡
)
2

25.(3)a. 𝑦 = 21 𝑒𝑡 + 2𝑒2𝑡 − 32 𝑒3𝑡


1 𝑡
(3)b. 2𝑒 − 2𝑒2𝑡 + 32 𝑒3𝑡
1 𝑡 11 2𝑡
(3)c. 2𝑒 + 2 𝑒 − 7𝑒3𝑡
1 𝑡
(3)d. 2𝑒 + (3𝑎 − 𝑏 − 1)𝑒2𝑡 + (𝑏 − 2𝑎 + 21 )𝑒3𝑡

26.(3)a. 𝑦 = 61 𝑡5 + 10 2 5 3
3 𝑡 − 2𝑡
(3)b. 𝑦 = 16 𝑡5 − 2 2
3𝑡 + 2𝑡
1 3

(3)c. 𝑦 = 16 𝑡5 − 17 2 9 3
3 𝑡 + 2𝑡
(3)d. 𝑦 = 16 𝑡5 +
(1
+ − 12 − 2𝑎 + 𝑏 𝑡3
) 2 ( )
3 + 3𝑎 − 𝑏 𝑡

27. Write the equation in the standard form:


3 1
𝑦 ′′ + 𝑦 ′ − 2 𝑦 = 𝑡2 .
𝑡 𝑡
3 1
Then 𝑎1 (𝑡) = , 𝑎2 (𝑡) = − 2 , and 𝑓 (𝑡) = 𝑡2 . These three functions are
𝑡 𝑡
all continuous on the intervals (0, ∞) and (−∞, 0). Thus, Theorem 6
shows that if 𝑡0 ∈ (0, ∞) then the unique solution is also defined on the
interval (0, ∞), and if 𝑡0 ∈ (−∞, 0), then the unique solution is defined
on (−∞, 0).

28. Maximal intervals are (−∞, −1), (−1, 1), (1, ∞)

29. (𝑘𝜋, (𝑘 + 1)𝜋) where 𝑘 ∈ ℤ


98 1 Solutions

30. (−∞, ∞)

31. (3, ∞)

32. (−∞, −2), (−2, 0), (0 2), (2, ∞)

33. The initial condition occurs at 𝑡 = 0 which is precisely where 𝑎2 (𝑡) = 𝑡2


has a zero. Theorem 6 does not apply.

34. In this case 𝑦(𝑡0 ) = 0 and 𝑦 ′ (𝑡0 ) = 0. The function 𝑦(𝑡) = 0, 𝑡 ∈ 𝐼 is a


solution to the initial value problem. By the uniqueness part of Theorem
6 𝑦 = 0 is the only solution.

35. The assumptions say that 𝑦1 (𝑡0 ) = 𝑦2 (𝑡0 ) and 𝑦1′ (𝑡0 ) = 𝑦2′ (𝑡0 ). Both 𝑦1
and 𝑦2 therefore satisfies the same initial conditions. By the uniqueness
part of Theorem 6 𝑦1 = 𝑦2 .

Section 4.2
1. dependent; 2𝑡 and 5𝑡 are multiples of each other.

2. independent

3. independent

4. dependent; 𝑒2 𝑡 + 1 = 𝑒1 𝑒2𝑡 and 𝑒2𝑡−3 = 𝑒−3 𝑒2𝑡 , they are multiples of each
other.

5. independent

6. dependent; ln 𝑡2 = 2 ln 𝑡 and ln 𝑡5 = 5 ln 𝑡, they are multiples of each other.

7. dependent; sin 2𝑡 = 2 sin 𝑡 cos 𝑡, they are multiples of each other.

8. dependent; cosh 𝑡 = 21 (𝑒𝑡 + 𝑒−𝑡 ) and 3𝑒𝑡 (1 + 𝑒−2𝑡 ) = 3(𝑒𝑡 + 𝑒−𝑡 ), they are
multiples of each other.

9. 1. Suppose 𝑎𝑡3 + 𝑏 𝑡3 = 0 on (−∞, ∞). Then for 𝑡 = 1 and 𝑡 = −1 we
get
𝑎+𝑏 = 0
−𝑎 + 𝑏 = 0.
These equations imply 𝑎 = 𝑏 = 0. So 𝑦1 and 𝑦2 are linearly indepen-
dent. {
′ 2 ′ −3𝑡2 if 𝑡 < 0
2. Observe that 𝑦1 (𝑡) = 3𝑡 and 𝑦2 (𝑡) = 2
If 𝑡 < 0
3𝑡 if 𝑡 ≥ 0.
( 3
𝑡 −𝑡3
)
then 𝑤(𝑦1 , 𝑦2 )(𝑡) = = 0. If 𝑡 ≥ 0 then 𝑤(𝑦1 , 𝑦2 )(𝑡) =
3𝑡2 −3𝑡2
1 Solutions 99

𝑡3 𝑡3
( )
= 0. It follows that the Wronskian is zero for all 𝑡 ∈
3𝑡2 3𝑡2
(−∞, ∞).
3. The condition that the coefficient function 𝑎2 (𝑡) be nonzero in Theo-
rem 4 and Proposition 6 is essential. Here the coefficient function, 𝑡2 ,
of 𝑦 ′′ is zero at 𝑡 = 0, so Proposition 6 does not apply on (−∞, ∞).
The largest open intervals on which 𝑡2 is nonzero are (−∞, 0) and
(0, ∞). On each of these intervals 𝑦1 and 𝑦2 are linearly dependent.
4. Consider the cases 𝑡 < 0 and 𝑡 ≥ 0. The verification is then straight-
forward.
5. Again the condition that the coefficient function 𝑎2 (𝑡) be nonzero is
essential. The Uniqueness and Existence theorem does not apply.

Section 4.3
1. The indicial polynomial is 𝑞(𝑠) = 𝑠2 + 𝑠 − 2 = (𝑠 +{2)(𝑠 −}1). There are
two distinct roots 1 and −2. The fundamental set is 𝑡, 𝑡−2 . The general
solution is 𝑦(𝑡) = 𝑐1 𝑡 + 𝑐2 𝑡−2 .

2. The indicial polynomial is 𝑞(𝑠) = 2𝑠2 − 7𝑠 + 3 = (2𝑠 − } 3). There are


{ 1)(𝑠 −
1 1
two distinct roots 2 and 3. The fundamental set is 𝑡 2 , 𝑡3 . The general
1
solution is 𝑦(𝑡) = 𝑐1 𝑡 2 + 𝑐2 𝑡3 .

3. The indicial polynomial is 𝑞(𝑠) = 9𝑠2 − 6𝑠 + 1 = (3𝑠 −{1)2 . There}is one


1 1
root, 1/3, with multiplicity 2. The fundamental set is 𝑡 3 , 𝑡 3 ln 𝑡 . The
1 1
general solution is 𝑦(𝑡) = 𝑐1 𝑡 3 + 𝑐2 𝑡 3 ln 𝑡.
√ √
4. The indicial polynomial is 𝑞(𝑠) = 𝑠2 − 2 = (𝑠 − 2)(𝑠 +
{ √ 2). There are
√ √ √ }
two distinct roots 2 and − 2. The fundamental set is 𝑡 2 , 𝑡− 2 . The
√ √
2
general solution is 𝑦(𝑡) = 𝑐1 𝑡 + 𝑐 2 𝑡− 2
.

5. The indicial polynomial is 𝑞(𝑠) = 4𝑠2 − 𝑠 + 1 {= (2𝑠 − 1)}2 . The root is


1 1 1
2 with multiplicity 2. The fundamental set is 𝑡 2 , 𝑡 2 ln 𝑡 . The general
1 1
solution is 𝑦(𝑡) = 𝑐1 𝑡 2 + 𝑐2 𝑡 2 ln 𝑡.

6. The indicial polynomial is 𝑞(𝑠) = 𝑠2 − 4𝑠


{ − 21 =} (𝑠 − 7)(𝑠 + 3). The roots
are 7 and −3. The fundamental set is 𝑡7 , 𝑡−3 . The general solution is
𝑦(𝑡) = 𝑐1 𝑡7 + 𝑐2 𝑡−3 .

8. The indicial polynomial is 𝑞(𝑠) = 𝑠2 − 𝑠 + 1 = 𝑠2 − 𝑠 + 1/4 + 3/4√=


√ √
1+𝑖 3
2 2
(𝑠 − 1/2) + ( 3/2) . There are two complex}roots, 2 and 1−𝑖2 3 .
{ 1
√ 1

3 3
The fundamental set is 𝑡 2 sin 2 𝑡, 𝑡
2 cos 2 𝑡 . The general solution is
1
√ 1

3 3
𝑦(𝑡) = 𝑐1 𝑡 sin
2
2 𝑡 + 𝑐2 𝑡 cos
2
2 𝑡.
100 1 Solutions

9. The indicial polynomial is 𝑞(𝑠) = 𝑠2 − 4 = (𝑠 − 2)(𝑠


{ + 2). }There are two
distinct roots, 2 and −2. The fundamental set is 𝑡2 , 𝑡−2 . The general
solution is 𝑦(𝑡) = 𝑐1 𝑡2 + 𝑐2 𝑡−2 .

10. The indicial polynomial is 𝑞(𝑠) = 𝑠2 + 4. There are two complex roots,
2𝑖 and −2𝑖. The fundamental set is {cos(2 ln 𝑡), sin(2 ln 𝑡)}. The general
solution is 𝑦(𝑡) = 𝑐1 cos(2 ln 𝑡) + 𝑐2 sin(2 ln 𝑡).

11. The indicial polynomial is 𝑞(𝑠) = 𝑠2 − 4𝑠 + 13 = (𝑠 − 2)2 + 9. There


are
{ 2 two complex roots, }2 + 3𝑖 and 2 − 3𝑖. The fundamental set is
𝑡 cos(3 ln 𝑡), 𝑡2 sin(3 ln 𝑡) . The general solution is 𝑦(𝑡) = 𝑐1 𝑡2 cos(3 ln 𝑡)+
𝑐2 𝑡2 sin(3 ln 𝑡).
1
12. 𝑦 = (𝑡 − 𝑡−2 )
3
13. 𝑦 = 2𝑡1/2 − 𝑡1/2 ln 𝑡

14. 𝑦 = −3 cos(2 ln 𝑡) + 2 sin(2 ln 𝑡)

15. No solution is possible.

Section 4.4
( )
𝑠−𝑎
1. ln
𝑠−𝑏

𝑠2 + 𝑏 2
( )
2. ln 2
𝑠 + 𝑎2
( 2
𝑠 + 𝑎2
) ( )
−1 𝑏 −1 𝑎
( )
3. 𝑠 ln + 2𝑏 tan − 2𝑎 tan
𝑠2 + 𝑏 2 𝑠 𝑠
(𝑎)
4. tan−1
𝑠
5. 𝑦 = 𝑎𝑡 + 𝑏𝑒𝑡

6. 𝑦 = 𝑎(𝑡 + 1) + 𝑏𝑒−𝑡
1 𝐶
7. 𝑌 ′ (𝑠) + 𝑌 (𝑠) = 0 and 𝑌 (𝑠) = and 𝑦(𝑡) = 𝐶𝑒−𝑡
𝑠+1 𝑠+1
−𝑦0 𝑦0
8. 𝑌 ′ (𝑠) = , 𝑌 (𝑠) = , and 𝑦(𝑡) = 𝑒−2𝑡 .
(𝑠 + 2)2 𝑠+2

4𝑠 3𝑦0 𝑦0 (𝑠3 + 3𝑠) + 𝐶


9. 𝑌 ′ (𝑠) + 𝑌 (𝑠) = 2 . Solving gives 𝑌 (𝑠) = and
𝑠2+1 𝑠 +1 (𝑠2 + 1)2
𝑦(𝑡) = 𝐴(𝑡 cos 𝑡 − sin 𝑡) + 𝐵(𝑡 sin 𝑡 + cos 𝑡).
1 Solutions 101

2𝑠2 + 𝑠 − 2 2(𝑦0 𝑠 + 𝑦1 ) 2𝑠2 − 2𝑠 + 1 2𝑠 − 1


10. 𝑌 ′ (𝑠) + = 𝑌 (𝑠) = 𝑦0 + 2 𝑦1
𝑠(𝑠 − 1) 𝑠(𝑠 − 1) 2𝑠2 (𝑠 − 1) 1𝑠 (𝑠 − 1)
𝑦(𝑡) = (−𝑡 + 1 + 𝑒𝑡 ) + (𝑡 − 1 + 𝑒𝑡 )𝑦1 The general solution can be written
𝐴(𝑡 − 1) + 𝐵𝑒𝑡
−𝑦0 𝑦0
11. 𝑌 ′ (𝑠) = 2
, 𝑌 (𝑠) = , and 𝑦(𝑡) = 𝑦0 𝑒−𝑡 .
(𝑠 + 1) 𝑠+1
6𝑠 𝐶
12. 𝑌 ′ (𝑠)+ , and 𝑦(𝑡) = 𝐶 (3 − 𝑡2 ) sin 𝑡 − 3𝑡 cos 𝑡
( )
𝑌 (𝑠) = 0, 𝑌 (𝑠) = 2
𝑠2+1 (𝑠 + 1) 3

𝑠−2 −𝑠𝑦0 − 𝑦1 𝑠2 + 3𝑠 + 3 𝑠+2


13. 𝑌 ′ (𝑠) − 𝑌 (𝑠) = 2
, 𝑌 (𝑠) = 𝑦0 + 𝑦1 , and
𝑠+1 (𝑠 + 1) (𝑠 + 1)3 (𝑠 + 1)3
𝑡2 𝑡2
𝑦(𝑡) = 𝑦0 (𝑒−𝑡 + 𝑡𝑒−𝑡 + 𝑒−𝑡 ) + 𝑦1 (𝑡𝑒−𝑡 + 𝑒−𝑡 ). The general solution can
2 2
2
𝑡
be written 𝑦(𝑡) = 𝐴𝑒−𝑡 + 𝐵(𝑡𝑒−𝑡 + 𝑒−𝑡 ).
2
( )
1 1 𝑠
14. 𝑌 ′ (𝑠) = 𝑦0 ( − ) Then 𝑌 (𝑠) = 𝑦0 ln + 𝐶. Take 𝐶 = 0.
𝑠 𝑠−1 𝑠−1
1 − 𝑒𝑡
Hence 𝑦(𝑡) = 𝑦0 .
𝑡
( )
−𝑦0 𝑦0 𝑠+1
15. 𝑌 ′ (𝑠) = 2 and 𝑌 (𝑠) = ln + 𝐶. Take 𝐶 = 0 Then 𝑦(𝑡) =
𝑠 −1 2 𝑠−1
𝑦0 𝑒𝑡 − 𝑒−𝑡
.
2 𝑡
( )
−𝑦0 𝑠−2
16. 𝑌 ′ (𝑠) = 2 and 𝑌 (𝑠) = 𝑦0 ln + 𝐶. Take 𝐶 = 0. Then
(𝑠
( 3𝑡− 5𝑠 + )6) 𝑠−3
𝑒 − 𝑒2𝑡
𝑦(𝑡) = 𝑦0 .
𝑡
−𝑦0 −𝑦0
17. 𝑌 ′ (𝑠) = so 𝑌 (𝑠) = (tan−1 (𝑠/3) + 𝐶). Since lim𝑠→∞ 𝑌 (𝑠) = 0
𝑠2 + 9 3
𝜋 𝑦0 3 𝑦0 sin 3𝑡
we have 𝐶 = − . Hence 𝑌 (𝑠) = tan−1 ( ) and 𝑦(𝑡) = .
2 3 𝑠 3 𝑡
( )
′ −𝑦0 𝑠+1
18. 𝑌 (𝑠) = 2 and hence 𝑦(𝑠) = 𝑦0 ln + 𝐶. But 𝐶 = 0 and
𝑠 +𝑠 𝑠
−𝑡
1−𝑒
𝑦(𝑡) = 𝑦0 .
𝑡
19. We use the formula
𝑛 ( ) 𝑘
𝑑𝑛 ∑ 𝑛 𝑑 𝑑𝑛−𝑘
(𝑓 (𝑡)𝑔(𝑡)) = 𝑓 (𝑡) ⋅ 𝑔(𝑡).
𝑑𝑡𝑛 𝑚 𝑑𝑡𝑘 𝑑𝑡𝑛−𝑘
𝑘=0

Observe that
102 1 Solutions

𝑑𝑘 −𝑡
𝑒 = (−1)𝑘 𝑒−𝑡
𝑑𝑡𝑘
and
𝑑𝑛−𝑘 𝑛
𝑡 = 𝑛(𝑛 − 1) ⋅ ⋅ ⋅ (𝑘 + 1)𝑡𝑘 .
𝑑𝑡𝑛−𝑘
It now follows that
1 𝑡 𝑑𝑛 −𝑡 𝑛
𝑒 (𝑒 𝑡 )
𝑛! 𝑑𝑡𝑛
𝑛 ( )
1 𝑡 ∑ 𝑛 𝑑𝑘 −𝑡 𝑑𝑛−𝑘 𝑛
= 𝑒 𝑒 𝑡
𝑛! 𝑘 𝑑𝑡𝑘 𝑑𝑡𝑛−𝑘
𝑘=0
𝑛 ( )
𝑡
∑ 𝑛 𝑛(𝑛 − 1) ⋅ ⋅ ⋅ (𝑘 + 1) 𝑘
=𝑒 (−1)𝑘 𝑒−𝑡 𝑡
𝑘 𝑛!
𝑘=0
𝑛 ( ) 𝑘
∑ 𝑛 𝑡
= (−1)𝑘
𝑘 𝑘!
𝑘=0
= ℓ𝑛 (𝑡).

20. This follows in a similar manner to the proof of Equation (2) given in
Theorem 11.

22.

ℒ {ℓ𝑛 (𝑎𝑡)} (𝑠)


𝑛 ( ) { 𝑘}
∑ 𝑛 𝑡
= (−1)𝑘 𝑎𝑘 ℒ
𝑘 𝑘!
𝑘=0
𝑛 ( )
∑ 𝑛 1
= (−1)𝑘 𝑎𝑘 𝑘+1
𝑘 𝑠
𝑘=0
𝑛 ( )
1 ∑ 𝑛
= 𝑛+1 (−𝑎)𝑘 𝑠𝑛−𝑘
𝑠 𝑘
𝑘=0
(𝑠 − 𝑎)𝑛
= .
𝑠𝑛+1
The last line follows from the binomial theorem. Note: the dilation prin-
ciple gives the same formula for 𝑎 > 0.

23. Hint: Take the Laplace transform of each side. Use the previous exercise
and the binomial theorem.
∫𝑡
24. We have that ℓ𝑛 ∗ 1(𝑡) = 0 ℓ𝑛 (𝑥) 𝑑𝑥. By the convolution theorem
1 Solutions 103
𝑛
1 (𝑠 − 1)
ℒ {ℓ𝑛 ∗ 1} (𝑠) =
𝑠 𝑠𝑛+1 )
𝑠 − 1 (𝑠 − 1)𝑛
(
= 1−
𝑠 𝑠𝑛+1
(𝑠 − 1)𝑛 (𝑠 − 1)𝑛+1
= 𝑛+1

𝑠 𝑠𝑛+2
= ℒ {ℓ𝑛 } (𝑠) − ℒ {ℓ𝑛+1 } (𝑠).
The result follows by inversion.

25. We compute the Laplace transform of both sides. We’ll do a piece at a


time.
ℒ {(2𝑛 + 1)ℓ𝑛 } (𝑠)
(𝑠 − 1)𝑛
= (2𝑛 + 1) 𝑛+1
𝑠
(𝑠 − 1)𝑛−1
= (2𝑛 + 1)(𝑠(𝑠 − 1)).
𝑠𝑛+2

ℒ {−𝑡ℓ𝑛 } (𝑠)
)′
(𝑠 − 1)𝑛
(
=
𝑠𝑛+1
(𝑠 − 1)𝑛−1
= (𝑛 + 1 − 𝑠).
𝑠𝑛+2

−𝑛ℒ {ℓ𝑛−1 } (𝑠)


(𝑠 − 1)𝑛−1
= −𝑛
𝑠𝑛
𝑛−1
(𝑠 − 1)
= (−𝑛𝑠2 ).
𝑠𝑛+2
(𝑠 − 1)𝑛−1
We have written each so that the common factor is . The co-
𝑠𝑛+2
efficients are
𝑛 + 1 − 𝑠 + (2𝑛 + 1)(𝑠(𝑠 − 1)) − 𝑛𝑠2
= (𝑛 + 1)(𝑠2 − 2𝑠 + 1)
= (𝑛 + 1)(𝑠 − 1)2
The right hand side is now
(𝑠 − 1)𝑛−1
( )
1
(𝑛 + 1)(𝑠 − 1)2
𝑛+1 𝑠𝑛+2
(𝑠 − 1)𝑛+1
=
𝑠𝑛+2
= ℒ {ℓ𝑛+1 } (𝑠).
104 1 Solutions

Taking the inverse Laplace transform completes the verification.


∫𝑡
26. We have that ℓ𝑛 ∗ ℓ𝑚 (𝑡) = 0 ℓ𝑛 (𝑥)ℓ𝑚 (𝑡 − 𝑥) 𝑑𝑥. By the convolution the-
orem

ℒ {ℓ𝑛 ∗ ℓ𝑚 } (𝑠)
(𝑠 − 1)𝑚 (𝑠 − 1)𝑛
=
𝑠𝑚+1 𝑠(𝑛+1

(𝑠 − 1)𝑚+𝑛
)
𝑠−1
= 1 −
𝑠𝑚+𝑛+1 𝑠
𝑚+𝑛
(𝑠 − 1) (𝑠 − 1)𝑚+𝑛+1
= 𝑚+𝑛+1

𝑠 𝑠𝑚+𝑛+2
= ℒ {ℓ𝑚+𝑛 (𝑠)} − ℒ {ℓ𝑚+𝑛+1 } (𝑠).

The result follows by inversion.


∫∞
27. First of all 0 𝑒−𝑡 ℓ𝑛 (𝑡) 𝑑𝑡 = ℒ {ℓ𝑛 } (1) = 0. Thus
∫ ∞
𝑒−𝑥 ℓ𝑛 (𝑥) 𝑑𝑡
𝑡
∫ 𝑡
=− 𝑒−𝑥 ℓ𝑛 (𝑥) 𝑑𝑥
0
∫ ∞
= −𝑒−𝑡 𝑒𝑡−𝑥 ℓ𝑛 (𝑥) 𝑑𝑥
0
= −𝑒−𝑡 (𝑒𝑡 ∗ ℓ𝑛 (𝑡)).

By the convolution theorem

ℒ 𝑒𝑡 ∗ ℓ𝑛 (𝑠)
{ }

1 (𝑠 − 1)𝑛
=
𝑠 − 1 𝑠𝑛+1
(𝑠 − 1)𝑛−1
=
𝑠𝑛+1
(𝑠 − 1)𝑛−1
( )
𝑠−1
= 1 −
𝑠𝑛 𝑠
𝑛−1
(𝑠 − 1) (𝑠 − 1)𝑛
= 𝑛

𝑠 𝑠𝑛+1
= ℒ {ℓ𝑛−1 (𝑡)} − ℒ−1 {ℓ𝑛 (𝑡)} .
−1

It follows by inversion that 𝑒𝑡 ∗ℓ𝑛 = ℓ𝑛−1 −ℓ𝑛 and substituting this formula
into the previous calculation gives the needed result.
1 Solutions 105

Section 4.5
1. Let 𝑦2 (𝑡) = 𝑡2 𝑢(𝑡). Then 𝑡4 𝑢′′ + 𝑡3 𝑢′ = 0, which gives 𝑢′ = 𝑡−1 and
𝑢(𝑡) = ln 𝑡. Substituting gives 𝑦2 (𝑡) = 𝑡2 ln 𝑡. The general solution can be
written 𝑦(𝑡) = 𝑐1 𝑡2 + 𝑐2 𝑡2 ln 𝑡.
−1
2. 𝑦2 (𝑡) = 3𝑡2 . The general solution can be written 𝑦(𝑡) = 𝑐1 𝑡 + 𝑐2 𝑡12 .

3. Let 𝑦2 (𝑡) = (1 − 𝑡2 )𝑢(𝑡). Substitution gives (1 − 𝑡2 )2 𝑢′′ − 4𝑡(1 − 𝑡2 )𝑢′ = 0


′′
and hence 𝑢𝑢′ = −2 1−𝑡 −2𝑡 ′ 1
2 . From this we get 𝑢 = (1−𝑡)2 . Integrating 𝑢 by

( )
partial fractions give 𝑢 = −1 𝑡 1 1+𝑡
2 1−𝑡2 + 4 ln 1−𝑡 and hence
( )
−1 1 1+𝑡
𝑦2 (𝑡) = 𝑡 + (1 − 𝑡2 ) ln .
2 4 1−𝑡

()
𝑡 1+𝑡
4. 𝑦2 (𝑡) = −1 + 2 ln . The general solution can be written 𝑦(𝑡) =
1−𝑡
( ( ))
𝑡 1+𝑡
𝑐1 𝑡 + 𝑐2 −1 + 2 ln 1−𝑡 .
1 5 3
5. Let 𝑦2 (𝑡) = 𝑡 2 𝑢(𝑡). Then 4𝑡√2 𝑢′′ + 4𝑡 2 𝑢′ = 0 leads to 𝑢′ = 1/𝑡 and hence

𝑢(𝑡) = ln√𝑡. Thus 𝑦√ 2 (𝑡) = 𝑡 ln 𝑡. The general solution can be written


𝑦(𝑡) = 𝑐1 𝑡 + 𝑐 + 2 𝑡 ln 𝑡.

6. 𝑦2 (𝑡) = 1. The general solution can be written 𝑦(𝑡) = 𝑐1 𝑡 + 𝑐2 .

7. 𝑦2 (𝑡) = 𝑡𝑒𝑡 . The general solution can be written 𝑦(𝑡) = 𝑐1 𝑡 + 𝑐2 𝑡𝑒𝑡 .

8. Let 𝑦2 (𝑡) = 𝑡2 cos 𝑡 𝑢(𝑡). Then 𝑡4 cos 𝑡 𝑢′′ − 2𝑡4 sin 𝑡 𝑢′ = 0 which gives
𝑢′ (𝑡) = sec2 𝑡 and hence 𝑢(𝑡) = tan 𝑡. Thus 𝑦2 (𝑡) = 𝑡2 sin 𝑡. The general
solution can be written 𝑦(𝑡) = 𝑐1 𝑡2 cos 𝑡 + 𝑐2 𝑡2 sin 𝑡.

9. 𝑦2 (𝑡) = −1 2 2
2 cos 𝑡 . The general solution can be written 𝑦(𝑡) = 𝑐1 sin 𝑡 +
2
𝑐2 cos 𝑡 .

10. 𝑦2 (𝑡) = 𝑡𝑒2𝑡 . The general solution can be written 𝑦(𝑡) = 𝑐1 𝑒2𝑡 + 𝑐2 𝑡𝑒2𝑡 .

11. 𝑦2 (𝑡) = −1 − 𝑡 tan 𝑡. The general solution can be written 𝑦(𝑡) = 𝑐1 tan 𝑡 +
𝑐2 (1 + 𝑡 tan 𝑡).

12. 𝑦2 (𝑡) = 𝑡 − 1. The general solution can be written 𝑦(𝑡) = 𝑐1 𝑒−𝑡 + 𝑐2 (𝑡 − 1).

13. 𝑦2 (𝑡) = − sec 𝑡. The general solution can be written 𝑦(𝑡) = 𝑐1 tan 𝑡 +
𝑐2 sec 𝑡.
2
14. Let 𝑦2 (𝑡) = 𝑡𝑢(𝑡). Then (𝑡3 + 𝑡)𝑢′′ + 2𝑢′ = 0 which gives 𝑢′ = 𝑡 𝑡+1
2 and
𝑢 = 𝑡 − 1𝑡 . Thus 𝑦2 (𝑡) = 𝑡2 − 1. The general solution can be written
𝑦(𝑡) = 𝑐1 𝑡 + 𝑐2 (𝑡2 − 1).
106 1 Solutions

𝑡 sin 2𝑡 sin 2𝑡
15. 𝑦2 = −1− 1+cos . The general solution can be written 𝑦(𝑡) = 𝑐1 1+cos 2𝑡 +
( )2𝑡
𝑡 sin 2𝑡
𝑐2 1 + 1+cos 2𝑡 .

16. 𝑦2 (𝑡) = 𝑡 sin 𝑡. The general solution can be written 𝑦(𝑡) = 𝑐1 𝑡 cos 𝑡 +
𝑐2 𝑡 sin 𝑡.

Section 4.6
1. sin 𝑡 and cos 𝑡 form a fundamental set for the homogeneous solutions. Let
𝑦(𝑝 (𝑡) = 𝑢1 cos)𝑡(+ 𝑢) 2 sin 𝑡.(Then )the matrix equation
cos 𝑡 sin 𝑡 𝑢′1 0
= implies 𝑢′1 (𝑡) = − sin2 𝑡 = 21 (cos 2𝑡 − 1)
− sin 𝑡 cos 𝑡 𝑢′2 sin 𝑡
and 𝑢′2 (𝑡) = cos 𝑡 sin 𝑡 = 12 (sin 2𝑡). Integration give 𝑢1 (𝑡) = 14 (sin(2𝑡) −
2𝑡) = 12 (sin 𝑡 cos 𝑡−𝑡) and 𝑢2 (𝑡) = −1 −1
4 cos 2𝑡 = 4 (2 cos 2𝑡−1). This implies
1 1 1
𝑦𝑝 (𝑡) = 4 sin 𝑡 − 2 𝑡 cos 𝑡. Since 4 sin 𝑡 is a homogeneous solution we can
write the general solution in the form 𝑦(𝑡) = −1 2 𝑡 cos 𝑡 + 𝑐1 cos 𝑡 + 𝑐2 sin 𝑡.
We observe that a particular solution is the imaginary part of a solution
to 𝑦 ′′ + 𝑦 = 𝑒𝑖𝑡 . We use the incomplete partial fraction method and get
𝑝(𝑠)
𝑌 (𝑠) = (𝑠−𝑖)12 (𝑠+𝑖) . This can be written 𝑌 (𝑠) = 2𝑖 1 1
(𝑠−𝑖)2 + (𝑠−𝑖)(𝑠+𝑖) . From
( )
1 −1 1
this we get 𝑦𝑝 (𝑡) = Im 2𝑖 ℒ { (𝑠−𝑖) 2} = Im −𝑖 2 𝑡𝑒
𝑖𝑡
= −1
2 𝑡 cos 𝑡. The
−1
general solution is 𝑦(𝑡) = 2 𝑡 cos 𝑡
+ 𝑐1 cos 𝑡 + 𝑐2 sin 𝑡.

2. A fundamental set for 𝑦 ′′ − 4𝑦 = 0 is 𝑒2𝑡 , 𝑒−2𝑡 . Let 𝑦𝑝 (𝑡) = 𝑢1 (𝑡)𝑒2𝑡 +


{ }

𝑢2 (𝑡)𝑒−2𝑡 . Then
) (the )matrix
( equation
𝑒2𝑡 𝑒−2𝑡 𝑢′1
( )
0 4𝑡
= implies 𝑢′1 (𝑡) = 14 and 𝑢′2 (𝑡) = −𝑒4 and
2𝑒2𝑡 −2𝑒−2𝑡 𝑢′2 𝑒2𝑡
4𝑡
𝑒2𝑡 𝑒2𝑡
hence 𝑢1 (𝑡) = 4𝑡 and 𝑢2 (𝑡) = −𝑒 𝑡 2𝑡
16 . Now 𝑦𝑝 (𝑡) = 4 𝑒 − 16 . Since 16
is a homogeneous solution we can write the general solution as 𝑦(𝑡) =
𝑡 2𝑡 2𝑡 −2𝑡
4 𝑒 + 𝑐1 𝑒 + 𝑐2 𝑒 . On the other hand, the incomplete partial fraction
1
𝑝(𝑠)
method gives 𝑌 (𝑠) = (𝑠−2)12 (𝑠+2 = (𝑠−2) 4
2 + (𝑠−2)(𝑠+2) . From this we see

that a particular solution is 𝑦𝑝 (𝑡) = 14 𝑡𝑒2𝑡 . The general solution is 𝑦(𝑡) =


1 2𝑡 2𝑡 −2𝑡
4 𝑡𝑒 + 𝑐1 𝑒 + 𝑐2 𝑒 .

3. The functions 𝑒𝑡 cos 2𝑡 and 𝑒𝑡 sin 2𝑡 form a fundamental set. Let 𝑦𝑝 (𝑡) =
𝑐1 𝑒𝑡 cos 2𝑡 + 𝑐2 𝑒𝑡 sin 2𝑡.
( Then ) the
( matrix equation
𝑢′1
)
0
𝑡 𝑡
𝑊 (𝑒 cos 2𝑡, 𝑒 sin 2𝑡) ′ = implies that 𝑢′1 (𝑡) = −1
2 sin 2𝑡 and
𝑢2 𝑒𝑡
𝑢′2 (𝑡) = 12 cos 2𝑡. Hence, 𝑢1 (𝑡) = 41 𝑐𝑜𝑠2𝑡 and 𝑢2 (𝑡) = 14 sin 2𝑡. From this
we get 𝑦𝑝 (𝑡) = 41 𝑒𝑡 cos2 2𝑡 + 14 𝑒𝑡 sin2 2𝑡 = 41 𝑒𝑡 . On the other hand, the
method of undetermined coefficients implies that a particular solution is
of the form 𝑦𝑝 (𝑡) = 𝐶𝑒𝑡 . Substitution gives 4𝐶𝑒𝑡 = 𝑒𝑡 and hence 𝐶 = 41 .
It follows that 𝑦𝑝 (𝑡) = 41 𝑒𝑡 . Furthermore, the general solution is 𝑦(𝑡) =
1 𝑡 𝑡 𝑡
4 𝑒 + 𝑐1 𝑒 cos 2𝑡 + 𝑐2 𝑒 sin 2𝑡.
1 Solutions 107

is (1, 𝑒−3𝑡
{ }
4. A
( fundamental set . The matrix equation
1 𝑒−3𝑡 𝑢′1
) ( ) )
0
= −3𝑡 implies 𝑢′1 (𝑡) = 13 𝑒−3𝑡 and 𝑢′2 (𝑡) = −1 3 . Thus
0 −3𝑒−3𝑡 𝑢′2 𝑒
−3𝑡 −3𝑡
𝑢1 (𝑡) = −𝑒9 , 𝑢2 (𝑡) = −𝑡 3 , and 𝑦𝑝 (𝑡) =
−𝑒
9 − 3𝑡 𝑒−3𝑡 . Observe though
−3𝑡
that −𝑒9 is a homogeneous solution and so the general solution can
be written 𝑦(𝑡) = − 3𝑡 𝑒−3𝑡 + 𝑐1 + 𝑐2 𝑒−3𝑡 . The incomplete partial fraction
−1
1 𝑝(𝑠) −1 −3𝑡
method gives 𝑌 (𝑠) = (𝑠+3) 2 𝑠 = (𝑠+3)2 + (𝑠+3)𝑠 which implies that 3 𝑡𝑒
3

is a particular solution. The general solution is as above.

set is( 𝑒𝑡), 𝑒2𝑡 . The matrix equation


{ }
5. A
( fundamental
𝑒𝑡 𝑒2𝑡 𝑢′1
) ( )
0
𝑡 2𝑡 = implies 𝑢′1 (𝑡) = −𝑒2𝑡 and 𝑢′2 (𝑡) = 𝑒𝑡 . Hence
𝑒 2𝑒 𝑢′2 𝑒3𝑡
𝑢1 (𝑡) = −1 2𝑡 𝑡 −1 2𝑡 𝑡
2 𝑒 , 𝑢2 (𝑡) = 𝑒 , and 𝑦𝑝 (𝑡) = 2 𝑒 𝑒 + 𝑒 𝑒
𝑡 2𝑡
= 21 𝑒3𝑡 . The
1 3𝑡
general solution is 𝑦(𝑡) = 2 𝑒 + 𝑐1 𝑒𝑡 + 𝑐2 𝑒2𝑡 . The method of undetermined
coefficients implies that a particular solution is of the form 𝑦𝑝 = 𝐶𝑒3𝑡 .
Substitution gives 2𝐶𝑒3𝑡 = 3𝑒3𝑡 and hence 𝐶 = 12 . The general solution is
as above.

6. sin 𝑡 and cos 𝑡 form a fundamental set for the homogeneous solutions. Let
𝑦(𝑝 (𝑡) = 𝑢1 cos)𝑡(+ 𝑢) 2 sin 𝑡.(Then )the matrix equation
cos 𝑡 sin 𝑡 𝑢′1 0
= implies that 𝑢′1 (𝑡) = cos 𝑡 − sec 𝑡 and
− sin 𝑡 cos 𝑡 𝑢′2 tan 𝑡
𝑢′2 (𝑡) = sin 𝑡. From this we get 𝑢1 (𝑡) = sin 𝑡 − ln ∣sec 𝑡 + tan 𝑡∣ and 𝑢2 (𝑡) =
− cos 𝑡. Therefore 𝑦𝑝 (𝑡) = − cos 𝑡 ln ∣sec 𝑡 + tan 𝑡∣. The general solution is
thus 𝑦(𝑡) = − cos 𝑡 ln ∣sec 𝑡 + tan 𝑡∣ + 𝑐1 cos 𝑡 + 𝑐2 sin 𝑡.
𝑡 𝑡
7. A
( fundamental ) ( set)is {𝑒(, 𝑡𝑒)}. The matrix equation
𝑡 𝑡 ′ 0
𝑒 𝑡𝑒 𝑢1
= 𝑒𝑡 implies 𝑢′1 (𝑡) = −1 and 𝑢′2 (𝑡) = 1𝑡 . Hence,
𝑒𝑡 𝑒𝑡 + 𝑡𝑒𝑡 𝑢′2 𝑡
𝑢1 (𝑡) = −𝑡, 𝑢2 (𝑡) = ln 𝑡, and 𝑦𝑝 (𝑡) = −𝑡𝑒𝑡 + 𝑡 ln 𝑡𝑒𝑡 . Since −𝑡𝑒𝑡 is a
homogeneous solution we can write the general solution as 𝑦(𝑡) = 𝑡 ln 𝑡𝑒𝑡 +
𝑐1 𝑒𝑡 + 𝑐2 𝑡𝑒𝑡 .

8. A
( fundamental ) (set′ )
is {cos
(𝑡, sin)𝑡}. The matrix equation
cos 𝑡 sin 𝑡 𝑢1 0
= implies 𝑢′1 (𝑡) = − tan 𝑡 and 𝑢′2 (𝑡) = 1.
− sin 𝑡 cos 𝑡 𝑢′2 sec 𝑡
Hence 𝑢1 (𝑡) = ln(cos 𝑡), 𝑢2 (𝑡) = 𝑡, and 𝑦𝑝 (𝑡) = cos 𝑡 ln(cos 𝑡) + 𝑡 sin 𝑡. The
general solution is 𝑦(𝑡) = cos 𝑡 ln(cos 𝑡) + 𝑡 sin 𝑡 + 𝑐1 cos 𝑡 + 𝑐2 sin 𝑡.

9. The associated homogeneous equation is Cauchy-Euler} with indicial equa-


tion 𝑠2 −3𝑠+2 = (𝑠−2)(𝑠−1). It follows that 𝑡, 𝑡2 forms a fundamental
{

set. We put the given equation is standard form to get 𝑦 ′′ − 2𝑡 𝑦 ′ + 𝑡22 𝑦 = 𝑡2 .


2
Thus
( 2) 𝑓 (𝑡)
( = )𝑡 . The
( )matrix equation

𝑡𝑡 𝑢1 0
= 2 implies 𝑢′1 (𝑡) = −𝑡2 and 𝑢′2 (𝑡) = 𝑡. Hence 𝑢1 (𝑡) =
1 2𝑡 𝑢′2 𝑡
108 1 Solutions
3 2 3
−𝑡 𝑡2 2 𝑡4
3 , 𝑢2 (𝑡) = 𝑡2 , and 𝑦𝑝 (𝑡) = −𝑡3 𝑡 + 2𝑡 = 6. It follows that the general
4
solution is 𝑦(𝑡) = 𝑡6 + 𝑐1 𝑡 + 𝑐2 𝑡2 .
′′ 1 ′ 1
10. (In standard
) ( ′ ) form
( we get ) 𝑦 − 𝑡 𝑦 = 3𝑡 − 𝑡 . The matrix equation
2
1𝑡 𝑢1 0
= implies 𝑢′1 (𝑡) = −3 2 1 ′ 3 1
2 𝑡 + 2 and 𝑢2 (𝑡) = 2 − 2𝑡2 .
0 2𝑡 𝑢′2 3𝑡 − 1𝑡
Hence 𝑢1 (𝑡) = −1 3 1 3 1 3
2 𝑡 + 2 𝑡, 𝑢2 (𝑡) = 2 𝑡 + 2𝑡 , and 𝑦𝑝 (𝑡) = 𝑡 + 𝑡. The general
3 2
solution is 𝑦(𝑡) = 𝑡 + 𝑡 + 𝑐1 + 𝑐2 𝑡 .

11. The homogeneous equation is Cauchy-Euler with indicial equation 𝑠2 −


2𝑠 + 1 = (𝑠 − 1)2 . It follows that {𝑡, 𝑡 ln 𝑡} is a fundamental set. After
writing in standard form we see the forcing function 𝑓 (𝑡) is 1𝑡 . The matrix
equation
( )( ′) ( )
𝑡 𝑡 ln 𝑡 𝑢1 0
= 1 implies 𝑢′1 (𝑡) = − ln 𝑡
and 𝑢′2 (𝑡) = 1𝑡 . Hence
1 ln 𝑡 + 1 𝑢′2 𝑡
𝑡
2
2 2 2
𝑢1 (𝑡) = − ln 2
𝑡
, 𝑢2 (𝑡) = ln 𝑡, and 𝑦𝑝 (𝑡) = −𝑡 𝑡
2 ln 𝑡 + 𝑡 ln 𝑡 = 2 ln 𝑡. The
𝑡 2
general solution is 𝑦(𝑡) = 2 ln 𝑡 + 𝑐1 𝑡 + 𝑐2 𝑡 ln 𝑡.

12. A fundamental set is 𝑒2𝑡 , 𝑡𝑒2𝑡 . The matrix equation


{ }
( 2𝑡
𝑡𝑒2𝑡
)( ′)
0
( )
𝑒 𝑢1
= 2𝑡 implies 𝑢′1 (𝑡) = 𝑡2−𝑡 ′
+1 and 𝑢2 (𝑡) =
2𝑒2𝑡 𝑒2𝑡 (1 + 2𝑡) 𝑢′2 𝑒
2
𝑡 +1
1 −1 2
𝑡2 +1 . Hence 𝑢1 (𝑡) = 2 ln(𝑡 +1), 𝑢2 (𝑡) = tan
−1
𝑡, and 𝑦𝑝 (𝑡) = −1 2𝑡
2 𝑒 ln(𝑡 +
2

1)+𝑡 tan−1 𝑒2𝑡 . The general solution is 𝑦(𝑡) = −1 2𝑡 2


2 𝑒 ln(𝑡 +1)+𝑡 tan
−1 2𝑡
𝑒 +
2𝑡 2𝑡
𝑐1 𝑒 + 𝑐2 𝑡𝑒 .

13. (The matrix equation )( ′) ( )


tan 𝑡 sec 𝑡 𝑢1 0
= implies 𝑢′1 (𝑡) = 𝑡 and 𝑢′2 (𝑡) = −𝑡 sin 𝑡.
sec2 𝑡 sec 𝑡 tan 𝑡 𝑢′2 𝑡
2 2
Hence 𝑢1 (𝑡) = 𝑡2 , 𝑢2 (𝑡) = 𝑡 cos 𝑡 − sin 𝑡, and 𝑦𝑝 (𝑡) = 𝑡2 tan 𝑡 + (𝑡 cos 𝑡 −
2
sin 𝑡) sec 𝑡 = 𝑡2 tan 𝑡 + 𝑡 − tan 𝑡. Since tan 𝑡 is a homogeneous solution we
2
can write the general solution as 𝑦(𝑡) = 𝑡2 tan 𝑡 + 𝑡 + 𝑐1 tan 𝑡 + 𝑐2 sec 𝑡.

14. When put in standard form one sees that 𝑓 (𝑡) = 𝑡𝑒−𝑡 . The matrix equa-
tion
𝑡 − 1 𝑒−𝑡
( )( ′ ) ( )
𝑢1 0
= implies 𝑢′1 (𝑡) = 𝑒−𝑡 and 𝑢′2 (𝑡) = 1 − 𝑡.
1 −𝑒−𝑡 𝑢′2 𝑡𝑒−𝑡
2 2
Hence 𝑢1 (𝑡) = −𝑒−𝑡 , 𝑢2 (𝑡) = 𝑡− 𝑡2 , and 𝑦𝑝 (𝑡) = −(𝑡−1)𝑒−𝑡 +(𝑡− 𝑡2 )𝑒−𝑡 =
−𝑡2 −𝑡 −𝑡 −𝑡
2 𝑒 +𝑒 . Since 𝑒 is a homogeneous solution we can write the general
−𝑡2 −𝑡
solution as 𝑦(𝑡) = 2 𝑒 + 𝑐1 (𝑡 − 1) + 𝑐2 𝑒−𝑡 .

15. After put in standard form the forcing function 𝑓 is 4𝑡4 . The matrix equa-
tion
cos 𝑡2 sin 𝑡2
( )( ′) ( )
𝑢1 0
= implies 𝑢′1 (𝑡) = −2𝑡3 sin 𝑡2 and
−2𝑡 sin 𝑡2 2𝑡 cos 2𝑡 𝑢′2 4𝑡4
1 Solutions 109

𝑢′2 (𝑡) = 2𝑡3 cos 𝑡2 . Integration by parts gives 𝑢1 (𝑡) = 𝑡2 cos 𝑡2 − sin 𝑡2 and
𝑢2 (𝑡) = 𝑡2 sin 𝑡2 + cos 𝑡2 . Hence 𝑦𝑝 (𝑡) = 𝑡2 cos 𝑡2 − cos 𝑡2 sin 𝑡2 + 𝑡2 sin 𝑡2 +
cos 𝑡2 sin 𝑡2 = 𝑡2 . The general solution is 𝑦(𝑡) = 𝑡2 + 𝑐1 cos 𝑡2 + 𝑐2 sin 𝑡2 .
𝑡 −𝑡
16. (A fundamental
) ( ′ )set is
( {𝑒 , 𝑒 ) }. The matrix equation
𝑡 −𝑡 0
𝑒 𝑒 𝑢1 −1 𝑒𝑡
= 1 implies 𝑢′1 (𝑡) = 12 1+𝑒
1 ′
𝑡 and 𝑢2 (𝑡) = 2 1+𝑒𝑡 .
𝑒𝑡 −𝑒−𝑡 𝑢′2 1+𝑒 −𝑡

Hence 𝑢1 (𝑡) = 21 (𝑡 − ln(1 + 𝑒𝑡 ), 𝑢2 (𝑡) = −1 𝑡 𝑡


2 (𝑒 − ln(1 + 𝑒 )), and 𝑦𝑝 (𝑡) =
1 𝑡 𝑡 −𝑡 𝑡
2 (𝑡𝑒 − 1 − (𝑒 − 𝑒 ) ln(1 + 𝑒 )). (Note: in the integrations of 𝑢′1 and 𝑢′2
use the substitution 𝑢 = 𝑒𝑡 .) The general solution can now be written
𝑦(𝑡) = 12 (𝑡𝑒𝑡 − 1 − (𝑒𝑡 − 𝑒−𝑡 ) ln(1 + 𝑒𝑡 )) + 𝑐1 𝑒𝑡 + 𝑐2 𝑒−𝑡 .

18. 𝑦𝑝 (𝑡) = 𝑎1 𝑓 (𝑡) ∗ sin 𝑎𝑡

19. 𝑦𝑝 (𝑡) = 𝑎1 𝑓 (𝑡) ∗ sinh 𝑎𝑡

20. 𝑦𝑝 (𝑡) = 𝑎1 𝑓 (𝑡) ∗ 𝑡𝑒−𝑎𝑡


1
21. 𝑦𝑝 (𝑡) = 𝑏−𝑎 𝑓 (𝑡) ∗ (𝑒𝑏𝑡 − 𝑒𝑎𝑡 )

Section 5.1
1. Graph (c)

2. Graph (g)

3. Graph (e)

4. Graph (a)

5. Graph (f)

6. Graph (d)

7. Graph (h)

8. Graph (b)
110 1 Solutions

Graphs for problems 1 through 8

1 2

0 𝑡 0 𝑡
0 1 2 3 4 5 6 0 1 2 3 4 5 6
(a) (b)
4
1
2
0 𝑡
1 2 3 4 5 6
0 𝑡
0 1 2 3 4 5 6
−1
(c) (d)

1 1

0 𝑡 0 𝑡
0 1 2 3 4 5 6 0 1 2 3 4 5 6
(e) (f)

1
1

0 𝑡
1 2 3 4 5 6
0 𝑡
0 1 2 3 4 5 6
−1
(g) (h)
∫5 ∫2 2
∫3 ∫53
) 2 (
9. 0 𝑓 (𝑡) 𝑑𝑡 = 0 (𝑡 − 4) 𝑑𝑡 + 2 0 𝑑𝑡 + 3 (−𝑡 + 3) 𝑑𝑡 = 𝑡 /3 − 4𝑡 0 +
0+
( 2
) 5
−𝑡 /2 + 3𝑡 3 = (8/3 − 8) + (−25/2 + 15) − (−9/2 + 9) = −22/3.
∫2 ∫1 ∫2 1 2
10. 𝑓 (𝑢) 𝑑𝑢 = 0 (2 − 𝑢) 𝑑𝑢 + 1 𝑢3 𝑑𝑢 = (2𝑢 − 𝑢2 /2) 0 + 𝑢4 /4 1 = 3/2 +
0
4 − 1/4 = 21/4.
∫ 2𝜋 ∫𝜋 ∫ 2𝜋
11. 0 ∣sin 𝑥∣ 𝑑𝑥 = 0 sin 𝑥 𝑑𝑥 + 𝜋 − sin 𝑥 𝑑𝑥 = − cos 𝑥∣𝜋0 + cos 𝑥∣2𝜋𝜋 = 4.
∫3 ∫1 ∫21
∫31
12. 0
𝑓 (𝑤) 𝑑𝑤 = 0
𝑤 𝑑𝑤 + 1 𝑤
𝑑𝑤 + 2 2
𝑑𝑤 = 1/2 + ln 2 + 1/2 = 1 + ln 2
1 Solutions 111
∫5 ∫3 ∫4 ∫6
13. 2
𝑓 (𝑡) 𝑑𝑡 = 2
(3 − 𝑡) 𝑑𝑡 + 3
2(𝑡 − 3) 𝑑𝑡 + 4
2 𝑑𝑡 = 1/2 + 1 + 4 = 11/2
∫6 ∫2 ∫4 ∫6
14. 0
𝑓 (𝑡) 𝑑𝑡 = 0
(1 − 𝑡) 𝑑𝑡 + 2
(3 − 𝑡) 𝑑𝑡 + 4
(5 − 𝑡) 𝑑𝑡 = 0 + 0 + 0 = 0.
∫6 ∫1 ∫2 ∫6
15. 0
𝑓 (𝑢) 𝑑𝑢 = 0
𝑢 𝑑𝑢 + 1
(2 − 𝑢) 𝑑𝑢 + 2
1 𝑑𝑢 = 1/2 + 1/2 + 4 = 5.
∫6 ∫2 ∫3 ∫6
16. 0
𝑓 (𝑡) 𝑑𝑡 = 0
𝑡2 𝑑𝑡 + 2
4 𝑑𝑡 + 3
(7 − 𝑡) 𝑑𝑡 = 8/3 + 4 + 15/2 = 85/6

17. A is true since 𝑦(𝑡) satisfies the differential equation on each subinterval.
B is true since the left and right limits agree at 𝑡 = 2. C is not true since
𝑦(0) = 1 ∕= 2.

18. A is true since 𝑦(𝑡) satisfies the differential equation on each subinterval.
B is true since lim𝑡→2− 𝑦(𝑡) = 1 + 𝑒−8 = lim𝑡→2+ 𝑦(𝑡). C is true since
𝑦(0) = 2.

19. A is true since 𝑦(𝑡) satisfies the differential equation on each subinterval.
B is false since lim𝑡→2− 𝑦(𝑡) = 1 + 𝑒−8 while lim𝑡→2+ 𝑦(𝑡) = 1. C is false
since B is false.

20. A is false since 2𝑒−4𝑡 does not satisfy the differential equation 𝑦 ′ + 4𝑦 = 4
on the interval [0, 2). Since 𝐴 is false, B and C are necessarily false.

21. A is true since 𝑦(𝑡) satisfies the differential equation on each subinterval.
B is true since lim𝑡→1− 𝑦(𝑡) = −2𝑒 + 𝑒2 = lim𝑡→1+ 𝑦(𝑡). C is false since
lim𝑡→1− 𝑦 ′ (𝑡) = −3𝑒 + 2𝑒2 while lim𝑡→1+ 𝑦 ′ (𝑡) = 3𝑒2 − 2𝑒. D is false since
C is false.

22. A is true since 𝑦(𝑡) satisfies the differential equation on each subinterval.
B is false since lim𝑡→1− 𝑦(𝑡) = −2𝑒 + 𝑒2 while lim𝑡→1+ 𝑦(𝑡) = (1/2)𝑒2 − 3𝑒.
C and D are false since B is false. You cannot have a continuous derivative
if the function is not continuous.

23. A is true since 𝑦(𝑡) satisfies the differential equation on each subinterval.
B is true since lim𝑡→1− 𝑦(𝑡) = −2𝑒 + 𝑒2 = lim𝑡→1+ 𝑦(𝑡). C is true since
lim𝑡→1− 𝑦 ′ (𝑡) = −3𝑒 + 2𝑒2 = lim𝑡→1+ 𝑦 ′ (𝑡). D is true since 𝑦(0) = 𝑦 ′ (0) =
0.

24. A is true since 𝑦(𝑡) satisfies the differential equation on each subinter-
val. B is true since lim𝑡→1− 𝑦(𝑡) = −𝑒2 = lim𝑡→1+ 𝑦(𝑡). C is true since
lim𝑡→1− 𝑦 ′ (𝑡) = −𝑒 − 2𝑒2 = lim𝑡→1+ 𝑦 ′ (𝑡). D is false since 𝑦(0) = −2 ∕= 0.

25. The general solution of 𝑦 ′ − 𝑦 = 1 on the interval [0, 2) is found by using


the integrating factor 𝑒−𝑡 . The general solution is 𝑦(𝑡) = −1 + 𝑐𝑒𝑡 and
the initial condition 𝑦(0) = 0 gives 𝑐 = 1, so that 𝑦(𝑡) = −1 + 𝑒𝑡 for
𝑡 ∈ [0, 2). Continuity of 𝑦(𝑡) at 𝑡 = 2 will then give 𝑦(2) = lim𝑡→2− 𝑦(𝑡) =
−1 + 𝑒2 , which will provide the initial condition for the next interval
112 1 Solutions

[2, 4). The general solution of 𝑦 ′ − 𝑦 = −1 on [2, 4) is 𝑦(𝑡) = 1 + 𝑘𝑒𝑡 .


Thus −1 + 𝑒2 = 𝑦(2) = 1 + 𝑘𝑒2 and solve for 𝑘 to get 𝑘 = −2𝑒−2 + 1,
so that 𝑦(𝑡) = 1 + (−2𝑒−2 + 1)𝑒𝑡 for 𝑡 ∈ [2, 4). Continuity will then give
𝑦(4) = 1 + (−2𝑒−2 + 1)𝑒4 , which will provide the initial condition for
the next interval [4, ∞). The general solution to 𝑦 ′ − 𝑦 = 0 on [4, ∞)
is 𝑦(𝑡) = 𝑏𝑒𝑡 and the constant 𝑏 is obtained from the initial condition
𝑏𝑒4 = 𝑦(4) = 1 + (−2𝑒−2 + 1)𝑒4 , which gives 𝑏 = 𝑒−4 − 2𝑒−2 + 1, so
that 𝑦(𝑡) = (𝑒−4 − 2𝑒−2 + 1)𝑒𝑡 for 𝑡 ∈ [4, ∞). Putting these three pieces
together, we find that the solution is

𝑡
⎨−1 + 𝑒
 if 0 ≤ 𝑡 < 2,
𝑡−2
𝑦(𝑡) = 1 − 2𝑒 + 𝑒𝑡 if 2 ≤ 𝑡 < 4
⎩ 𝑡−4
− 2𝑒𝑡−2 + 𝑒𝑡 if 4 ≤ 𝑡 < ∞.

𝑒

26. The general solution of 𝑦 ′ + 3𝑦 = 𝑡 on the interval [0, 1) is found by using


the integrating factor 𝑒3𝑡 . The general solution is 𝑦(𝑡) = 31 𝑡 − 19 + 𝑐𝑒−3𝑡
and the initial condition gives 𝑐 = 19 . By continuity we must have 𝑦(1) =
1 1 1 −3
3 − 9 + 9𝑒 = 92 + 19 𝑒−3 , which will provide the initial condition for
the next interval [1, ∞). The general solution of 𝑦 ′ + 3𝑦 = 1 on (1, ∞) is
𝑦(𝑡) = 31 + 𝑘𝑒−3𝑡 . Setting 𝑦(1) = 31 − 19 + 19 𝑒−3 = 29 + 19 𝑒−3 found from
the interval [0, 1) equal to 𝑦(1) = 31 + 𝑘𝑒−3 found from the interval [1, ∞)
and solving for 𝑘 gives 𝑘 = 91 − 19 𝑒3 . Thus the complete solution is
{
1
𝑡 − 19 + 19 𝑒−3𝑡 if 0 ≤ 𝑡 < 1,
𝑦(𝑡) = 13 1 1 3 −3𝑡
3 + ( 9 − 9 𝑒 )𝑒 if 1 ≤ 𝑡 < ∞.

27. The general solution of 𝑦 ′ − 𝑦 = 𝑓 (𝑡) on any interval is found by using


the integrating factor 𝑒−𝑡 . The general solution on the interval [0, 1) is
𝑦(𝑡) = 𝑎𝑒𝑡 and since the initial condition is 𝑦(0) = 0, the solution on
[0, 1) is 𝑦(𝑡) = 0. Continuity then given 𝑦(1) = 0, which will be the initial
condition for the interval [1, 2). The general solution of 𝑦 ′ − 𝑦 = 𝑡 − 1
on the interval [1, 2) is 𝑦(𝑡) = −𝑡 + 𝑏𝑒𝑡 and the initial condition 𝑦(1) = 0
gives 0 = −1+𝑏𝑒1 so that 𝑏 = 𝑒−1 . Thus 𝑦(𝑡) = −𝑡+𝑒−1 𝑒𝑡 = −𝑡+𝑒𝑡−1 for
𝑡 ∈ [0, 2). Continuity of 𝑦(𝑡) at 𝑡 = 2 will then give 𝑦(2) = lim𝑡→2− 𝑦(𝑡) =
−2+𝑒1 , which will provide the initial condition for the next interval [2, 3).
The general solution of 𝑦 ′ − 𝑦 = 3 − 𝑡 on [2, 3) is 𝑦(𝑡) = 𝑡 − 2 + 𝑐𝑒𝑡 . Thus
−2 + 𝑒1 = 𝑦(2) = 𝑐𝑒2 and solve for 𝑐 to get 𝑐 = −2𝑒−2 + 𝑒−1 , so that
𝑦(𝑡) = 𝑡 − 2 + (−2𝑒−2 + 𝑒−1 )𝑒𝑡 = 𝑡 − 2 − 2𝑒𝑡−2 + 𝑒𝑡−1 for 𝑡 ∈ [2, 3).
Continuity will then give 𝑦(3) = 1 − 2𝑒1 + 𝑒2 , which will provide the initial
condition for the next interval [3, ∞). The general solution to 𝑦 ′ − 𝑦 = 0
on [4, ∞) is 𝑦(𝑡) = 𝑘𝑒𝑡 and the constant 𝑘 is obtained from the initial
condition 𝑘𝑒3 = 𝑦(3) = 1 − 2𝑒1 + 𝑒2 , which gives 𝑐 = 𝑒−3 − 2𝑒−2 + 𝑒−1 ,
1 Solutions 113

so that 𝑦(𝑡) = (𝑒−3 − 2𝑒−2 + 𝑒−1 )𝑒𝑡 = 𝑒𝑡−3 − 2𝑒𝑡−2 + 𝑒𝑡−1 for 𝑡 ∈ [3, ∞).
Putting these three pieces together, we find that the solution is

0
 if 0 ≤ 𝑡 < 1,
⎨−𝑡 + 𝑒𝑡−1

if 1 ≤ 𝑡 < 2,
𝑦(𝑡) = 𝑡−2 𝑡−1
𝑡 − 2 − 2𝑒
 +𝑒 if 2 ≤ 𝑡 < 3

⎩ 𝑡−3
𝑒 − 2𝑒𝑡−2 + 𝑒𝑡−1 if 3 ≤ 𝑡 < ∞.

28. The general solution of 𝑦 ′ + 𝑦 = 𝑓 (𝑡) on any interval is found by using the
integrating factor 𝑒𝑡 . Using this integrating factor the general solution of
𝑦 ′ + 𝑦 = sin 𝑡 on the interval [0, 𝜋) is found to be 𝑦(𝑡) = (sin 𝑡 − cos 𝑡)/2 +
𝑐𝑒−𝑡 . The initial condition actually occurs at the end of this interval, but
by continuity we can substitute 𝑡 = 𝜋 in this formula to get −1 = 𝑦(𝜋) =
1/2 + 𝑐𝑒−𝜋 so 𝑐 = −(3/2)𝑒𝜋 . Hence on the interval [0, 𝜋] the solution is
𝑦(𝑡) = (sin 𝑡 − cos 𝑡)/2 − (3/2)𝑒𝜋 𝑒−𝑡 = (sin 𝑡 − cos 𝑡)/2 − (3/2)𝑒−(𝑡−𝜋). The
general solution of 𝑦 ′ + 𝑦 = 0 on the interval [𝜋, ∞) is 𝑦(𝑡) = 𝑘𝑒−𝑡 and
𝑦(𝜋) = −1 this gives −1 = 𝑘𝑒−𝜋 so 𝑘 = −𝑒𝜋 and 𝑦(𝑡) = −𝑒−(𝑡−𝜋) on
[𝜋, ∞). Putting these two pieces together, we find that the solution is
{
(sin 𝑡 − cos 𝑡)/2 − (3/2)𝑒−(𝑡−𝜋) if 0 ≤ 𝑡 < 𝜋,
𝑦(𝑡) =
−𝑒−(𝑡−𝜋) if 𝜋 ≤ 𝑡 < ∞.

29. The characteristic polynomial of the equation 𝑦 ′′ −𝑦 = 𝑓 (𝑡) is 𝑠2 −1 = (𝑠−


1)(𝑠 + 1) so the homogeneous equation has the solution 𝑦ℎ (𝑡) = 𝑎𝑒𝑡 + 𝑏𝑒−𝑡
for constants 𝑎 and 𝑏. On the interval [0, 1] the equation 𝑦 ′′ − 𝑦 = 𝑡
has a particular solution 𝑦𝑝 (𝑡) = −𝑡 so the general solution has the form
𝑦(𝑡) = −𝑡 + 𝑎𝑒𝑡 + 𝑏𝑒−𝑡 . The initial conditions give 0 = 𝑦(0) = 𝑎 + 𝑏 and
1 = 𝑦 ′ (0) = −1 + 𝑎 − 𝑏. Solving gives 𝑎 = 1, 𝑏 = −1 so 𝑦(𝑡) = −𝑡 + 𝑒𝑡 − 𝑒−𝑡
on [0, 1). By continuity it follows that 𝑦(1) = −1 + 𝑒1 − 𝑒−1 and 𝑦 ′ (1) =
−1 + 𝑒1 + 𝑒−1 and these constitute the initial values for the equation
𝑦 ′′ − 𝑦 = 0 on the interval [1, ∞). The general solution on this interval is
𝑦(𝑡) = 𝑎𝑒𝑡 +𝑏𝑒−𝑡 and at 𝑡 = 1 we get 𝑦(1) = 𝑎𝑒1 +𝑏𝑒−1 = −1+𝑒1 −𝑒−1 and
𝑦 ′ (1) = 𝑎𝑒1 − 𝑏𝑒−1 = −1 + 𝑒1 + 𝑒−1 . Solving for 𝑎 and 𝑏 gives 𝑎 = 1 − 𝑒−1
and 𝑏 = −1 so that 𝑦(𝑡) = (1 − 𝑒−1 )𝑒𝑡 − 𝑒−𝑡 = 𝑒𝑡 − 𝑒𝑡−1 − 𝑒−1 . Putting
the two pieces together gives
{
−𝑡 + 𝑒𝑡 − 𝑒−𝑡 if 0 ≤ 𝑡 < 1,
𝑦(𝑡) = 𝑡 𝑡−1 −1
𝑒 −𝑒 −𝑒 1 ≤ 𝑡 < ∞.

30. The characteristic polynomial of the equation 𝑦 ′′ − 4𝑦 ′ + 4𝑦 = 𝑓 (𝑡) is


𝑠2 − 4𝑠 + 4 = (𝑠 − 2)2 so the homogeneous equation has the solution
𝑦ℎ (𝑡) = 𝑎𝑒2𝑡 + 𝑏𝑡𝑒2𝑡 for constants 𝑎 and 𝑏, which is valid on the interval
114 1 Solutions

[0, 2). The initial conditions 𝑦(0) = 1 and 𝑦 ′ (0) = 0 imply that 𝑐1 = 1
and 𝑐2 = −2. So 𝑦(𝑡) = 𝑒2𝑡 − 2𝑡𝑒2𝑡 on [0, 2). By continuity it follows that
𝑦(2) = 𝑒4 − 4𝑒4 = −3𝑒4 and 𝑦 ′ (2) = −8𝑒4 and these constitute the initial
values for the equation 𝑦 ′′ − 4𝑦 ′ + 4𝑦 = 4 on the interval [2, ∞). The
general solution on this interval is 𝑦(𝑡) = 1 + 𝑎𝑒2𝑡 + 𝑏𝑡𝑒2𝑡 and at 𝑡 = 2 we
get 𝑦(2) = 1 + 𝑎𝑒4 + 2𝑏𝑒4 = −3𝑒4 and 𝑦 ′ (1) = (2𝑎 + 𝑏)𝑒4 + 4𝑏𝑒4 = −8𝑒4 .
Solving for 𝑎 and 𝑏 gives 𝑎 = 1 − 5𝑒−4 and 𝑏 = −2 + 2𝑒−4 so that 𝑦(𝑡) =
1 + (1 − 5𝑒−4)𝑒2𝑡 + (−2 + 2𝑒−4)𝑡𝑒2𝑡 = 1 + 𝑒2𝑡 − 5𝑒2(𝑡−2) − 2𝑡𝑒2𝑡 + 2𝑡𝑒2(𝑡−2) .
Putting the two pieces together gives
{
𝑒2𝑡 − 2𝑡𝑒2𝑡 if 0 ≤ 𝑡 < 2,
𝑦(𝑡) =
1 + 𝑒2𝑡 − 5𝑒2(𝑡−2) − 2𝑡𝑒2𝑡 + 2𝑡𝑒2(𝑡−2) 2 ≤ 𝑡 < ∞.

33. 1. ∣𝑓 (𝑡)∣ = ∣sin(1/𝑡)∣ ≤ 1 for all 𝑡 ∕= 0, while ∣𝑓 (0)∣ = ∣0∣ = 0 ≤ 1.


2. It is enough to observe that lim𝑡→0+ does not exist. But letting 𝑡𝑛 =
1
𝑛𝜋 gives 𝑓 (𝑡𝑛 ) = sin 𝑛𝜋 = 0 for all positive integers 𝑛, while letting
2
𝑡𝑛 = (4𝑛+1)𝜋 gives 𝑓 (𝑡𝑛 ) = sin(1/𝑡𝑛 ) = sin((4𝑛 + 1)𝜋/2) = sin(2𝑛𝜋 +
𝜋
2 ) = 1 so there is one sequence 𝑡𝑛 → 0 with 𝑓 (𝑡𝑛 ) → 0 while another
sequence 𝑡𝑛 → 0 with 𝑓 (𝑡𝑛 ) → 1 so 𝑓 (𝑡) cannot be continuous at 0.
3. To be piecewise continuous, 𝑓 (𝑡) would have to have a limit at 𝑡 ap-
proaches 0 from above, and this is not true as shown in part 2.

Section 5.2

⎨0
 if 𝑡 < 2,
1. 𝑓 (𝑡) = 3ℎ(𝑡 − 2) − ℎ(𝑡 − 5) = 3 if 2 ≤ 𝑡 < 5, Thus, the graph is

2 if 𝑡 ≥ 5.

𝑦
3
2
1
0 𝑡
0 1 2 3 4 5 6 7 8


 0 if 𝑡 < 2,

⎨2 if 2 ≤ 𝑡 < 3,
2. 𝑓 (𝑡) = 2ℎ(𝑡 − 2) − 3ℎ(𝑡 − 3) + 4ℎ(𝑡 − 4) = Thus, the
−1 if 2 ≤ 𝑡 < 4,


3 if 𝑡 ≥ 4.

graph is
1 Solutions 115
𝑦
3
2
1
𝑡
1 2 3 4 5 6 7
−1

3. This function is 𝑔(𝑡 − 1)ℎ(𝑡 − 1) where 𝑔(𝑡) = 𝑡, so the graph of 𝑓 (𝑡) is


the graph of 𝑔(𝑡) = 𝑡 translated 1 unit to the right and then truncated at
𝑡 = 1, with the graph before 𝑡 = 1 replaced by the line 𝑦 = 0. Thus the
graph is
𝑦
2
1
0 𝑡
0 1 2 3

4. This function is 𝑔(𝑡 − 2)ℎ(𝑡 − 2) where 𝑔(𝑡) = 𝑡2 , so the graph of 𝑓 (𝑡) is


the graph of 𝑔(𝑡) = 𝑡2 translated 1 unit to the right and then truncated
at 𝑡 = 2, with the graph before 𝑡 = 2 replaced by the line 𝑦 = 0. Thus the
graph is
𝑦
2

0 𝑡
0 1 2 3

5. This function is just 𝑡2 truncated at 𝑡 = 2, with the graph before 𝑡 = 2


replaced by the line 𝑦 = 0. Thus the graph is
𝑦
8

0 𝑡
0 1 2 3
where the dashed line is the part of the 𝑡2 graph that has been truncated.
It is only shown for emphasis and it is not part of the graph.
116 1 Solutions

6. This is the sin 𝑡 function, truncated at 𝑡 = 𝜋. The graph is


𝑦
1

0 𝑡
𝜋 2𝜋 3𝜋
−1

7. This is the function cos 2𝑡 shifted 𝜋 units to the right and then truncated
at 𝑡 = 𝜋. The graph is
𝑦
1

0 𝑡
𝜋 2𝜋 3𝜋
−1

8. 𝑓 (𝑡) = 𝑡2 𝜒[0, 1) (𝑡) + (2 − 𝑡)𝜒[1, 3) (𝑡) + 3𝜒[3, ∞) (𝑡)



𝑡2
⎨ if 0 ≤ 𝑡 < 1,
= 2 − 𝑡 if 1 ≤ 𝑡 < 3, Thus, the graph is

3 if 𝑡 ≥ 3.

𝑦
3
2
1
𝑡
1 2 3 4 5
−1

9. (a) (𝑡 − 2)𝜒[2, ∞) (𝑡); (b) (𝑡 − 2)ℎ(𝑡 − 2);


(c) ℒ {(𝑡 − 2)ℎ(𝑡 − 2)} = 𝑒−2𝑠 ℒ {𝑡} = 𝑒−2𝑠 /𝑠2 .

10. (a) 𝑡𝜒[2, ∞) (𝑡); (b) 𝑡ℎ(𝑡 − 2); ( )


1 2
(c) ℒ {𝑡ℎ(𝑡 − 2)} = 𝑒−2𝑠 ℒ {𝑡 + 2} = 𝑒−2𝑠 + .
𝑠2 𝑠
11. (a) (𝑡 + 2)𝜒[2, ∞) (𝑡); (b) (𝑡 + 2)ℎ(𝑡 − 2); ( )
−2𝑠 −2𝑠 1 4
(c) ℒ {(𝑡 + 2)ℎ(𝑡 − 2)} = 𝑒 ℒ {(𝑡 + 2) + 2} = 𝑒 + .
𝑠2 𝑠
1 Solutions 117

12. (a) (𝑡 − 4)2 𝜒[4, ∞) (𝑡); (b) (𝑡 − 4)2 ℎ(𝑡 − 4);


2
(c) ℒ (𝑡 − 4)2 ℎ(𝑡 − 4) = 𝑒−2𝑠 ℒ 𝑡2 = 𝑒−4𝑠 3 .
{ } { }
𝑠
13. (a) 𝑡2{𝜒[4, ∞) (𝑡); (b) 2
} 𝑡 −4𝑠ℎ(𝑡 −{4);
(c) ℒ 𝑡(ℎ(𝑡 − 4) = 𝑒 ) ℒ (𝑡 + 4)2 = 𝑒−4𝑠 ℒ 𝑡2 + 8𝑡 + 16
2
} { }

2 8 16
= 𝑒−4𝑠 + 2+ .
𝑠3 𝑠 𝑠

14. (a) (𝑡2 − 4)𝜒[4, ∞) (𝑡); (b) (𝑡2 − 4)ℎ(𝑡 − 4);


{ 2 } −4𝑠
{ 2
} −4𝑠
{2 }
(c) ℒ (𝑡( − 4)ℎ(𝑡 − 4) )= 𝑒 ℒ (𝑡 + 4) − 4 = 𝑒 ℒ 𝑡 + 8𝑡 + 12
2 8 12
= 𝑒−4𝑠 + 2+ .
𝑠3 𝑠 𝑠

15. (a) (𝑡{− 4)2 𝜒[2, ∞) (𝑡); (b)


} (𝑡−2𝑠− 4)2{ℎ(𝑡 − 2);
2 2 −2𝑠
} {2 }
(c) ℒ (𝑡( − 4) ℎ(𝑡 − 2)) = 𝑒 ℒ ((𝑡 + 2) − 4) = 𝑒 ℒ 𝑡 − 4𝑡 + 4
2 4 4
= 𝑒−2𝑠 − + .
𝑠3 𝑠2 𝑠

16. (a) 𝑒𝑡−4 𝜒[4, ∞) (𝑡); (b) 𝑒𝑡−4 ℎ(𝑡 − 4);


1
(c) ℒ 𝑒𝑡−4 ℎ(𝑡 − 4) = 𝑒−4𝑠 ℒ {𝑒𝑡 } = 𝑒−4𝑠
{ }
.
𝑠−1
17. (a) 𝑒𝑡 𝜒[4, ∞) (𝑡); (b) 𝑒𝑡 ℎ(𝑡 −{4); }
(c) ℒ {𝑒𝑡 ℎ(𝑡 − 4)} = 𝑒−4𝑠 ℒ 𝑒𝑡+4 = 𝑒−4𝑠 𝑒4 ℒ {𝑒𝑡 }
1
= 𝑒−4(𝑠−1) .
𝑠−1
18. (a) 𝑒𝑡−4 𝜒[6, ∞) (𝑡); (b) 𝑒𝑡−4 ℎ(𝑡 − 6);
(c) ℒ 𝑒𝑡−4 ℎ(𝑡 − 6) = 𝑒−6𝑠 ℒ 𝑒(𝑡+6)−4 = 𝑒−6𝑠 𝑒2 ℒ {𝑒𝑡 }
{ } { }
1
= 𝑒−6𝑠+2 .
𝑠−1
19. (a) 𝑡𝑒𝑡 𝜒[4, ∞) (𝑡); (b) 𝑡𝑒𝑡 ℎ(𝑡 −
{ 4);
(c) ℒ {𝑡𝑒𝑡 ℎ(𝑡 −4𝑠 𝑡+4
= 𝑒−4𝑠 𝑒4 ℒ {𝑡𝑒𝑡 + 4𝑒𝑡 }
}
( − 4)} = 𝑒 ℒ (𝑡
) + 4)𝑒
1 4
= 𝑒−4(𝑠−1) + .
(𝑠 − 1)2 𝑠−1
20. (a) 𝜒[0,4) (𝑡) − 𝜒[4,5) (𝑡); (b) 1 − 2ℎ(𝑡 − 4) + ℎ(𝑡 − 5);
1 𝑒−4𝑠 𝑒−5𝑠
(c) ℒ {1 − 2ℎ(𝑡 − 4) + ℎ(𝑡 − 5)} = − 2 + .
𝑠 𝑠 𝑠
21. (a) 𝑡𝜒[0,1) (𝑡) + (2 − 𝑡)𝜒[1,∞) (𝑡); (b) 𝑡 + (2 − 2𝑡)ℎ(𝑡 − 1);
(c) ℒ {𝑡 + (2 − 2𝑡)ℎ(𝑡 − 1)} = ℒ {𝑡} + 𝑒−𝑠 ℒ {(2 − 2(𝑡 + 1))}
1 2𝑒−𝑠
= ℒ {𝑡} + 𝑒−𝑠 ℒ {−2𝑡} = 2 − 2 .
𝑠 𝑠
118 1 Solutions

22. (a) 𝑡𝜒[0,1) (𝑡) + (2 − 𝑡)𝜒[1,2) (𝑡) + 𝜒[2,∞) (𝑡);


(b) 𝑡 − (2 − 2𝑡)ℎ(𝑡 − 1) + (𝑡 − 1)ℎ(𝑡 − 2);
(c) ℒ {𝑡 + (2 − 2𝑡)ℎ(𝑡 − 1) + (𝑡 − 1)ℎ(𝑡 − 2)}
= ℒ {𝑡} + 𝑒−𝑠 ℒ {(2 − 2(𝑡 + 1))} + 𝑒−2𝑠 ℒ {(𝑡 + 2) − 1}
= ℒ {𝑡} + 𝑒−𝑠 ℒ {−2𝑡} −2𝑠
(+ 𝑒 ℒ){𝑡 + 1}
−𝑠
1 2𝑒 1 1
= 2 − 2 + 𝑒−2𝑠 + .
𝑠 𝑠 𝑠2 𝑠

23. (a) 𝑡2 𝜒[0, 2) (𝑡) + 4𝜒[2, 3) (𝑡) + (7 − 𝑡)𝜒[3, ∞) (𝑡);


(b) 𝑡2{+ (4 − 𝑡2 )ℎ(𝑡 − 2) + (3 − 𝑡)ℎ(𝑡 − 3); }
2
(c) ℒ{ 𝑡} + (4 − 𝑡2{ )ℎ(𝑡 − 2) + (3}− 𝑡)ℎ(𝑡 − 3)
= ℒ 𝑡2 + 𝑒−2𝑠 ℒ 4 − (𝑡 + 2)2 + 𝑒−3𝑠 ℒ {3 − (𝑡 + 3)}
𝑒−3𝑠
( )
2 2 4
= 3 − 𝑒−2𝑠 3
+ 2
− 2 .
𝑠 𝑠 𝑠 𝑠
24. (a) 𝜒[0,2) (𝑡) + (3 − 𝑡)𝜒[2,3) (𝑡) + 2(𝑡 − 3)𝜒[3,4) (𝑡) + 2𝜒[4,∞) (𝑡);
(b) 1 + (2 − 𝑡)ℎ(𝑡 − 2) + (3𝑡 − 9)ℎ(𝑡 − 3) − (2𝑡 − 4)ℎ(𝑡 − 4);
(c) ℒ {1 + (2 − 𝑡)ℎ(𝑡 − 2) + (3𝑡 − 9)ℎ(𝑡 − 3) − (2𝑡 − 4)ℎ(𝑡 − 4)}
= ℒ {1}+𝑒−2𝑠 ℒ {2 − (𝑡 + 2)}+𝑒 −3𝑠
ℒ {3(𝑡 −4𝑠
) + 3) − 9}−𝑒 ℒ {2(𝑡 + 4) − 4}
−2𝑠 −3𝑠
(
1 𝑒 3𝑒 2 4
= − 2 + − 𝑒−4𝑠 + .
𝑠 𝑠 𝑠2 𝑠2 𝑠
∑∞
25. (a) 𝑛=0 ∑ (𝑡 − 𝑛)𝜒[𝑛,𝑛+1) (𝑡);
(b) 𝑡 − ∞ ∑∞ℎ(𝑡 − 𝑛);
𝑛=1 ∑∞
(c) ℒ {𝑡 − 𝑛=1 ℎ(𝑡 − 𝑛)} = ℒ {𝑡} − 𝑛=1 ℒ {ℎ(𝑡 − 𝑛)}
1 𝑒−𝑛𝑠 1 1 ∑∞ 𝑛
= 2− ∞ (𝑒−𝑠 )

𝑛=1 = 2−
𝑠 𝑠 𝑠 𝑠 𝑛=1
1 𝑒−𝑠
= 2− .
𝑠 𝑠(1 − 𝑒−𝑠 )
∑∞ ∑∞
26. (a) 𝑛=0 𝜒[2𝑛,2𝑛+1) (𝑡); (b) 𝑛=0 (−1)𝑛 ℎ(𝑡 − 𝑛);
1
(c) .
𝑠(1 + 𝑒−𝑠 )
∑∞ ∑∞
27. (a) 𝑛=0 (2𝑛 + 1 − 𝑡)𝜒[2𝑛,2𝑛+2) (𝑡); (b) −(𝑡 + 1) + 2 𝑛=0 ℎ(𝑡 − 2𝑛);
1 1 2
(c) − 2 − + .
𝑠 𝑠 𝑠(1 − 𝑒−2𝑠 )
{ −3𝑠 } { }
𝑒 1
28. ℒ−1 = ℎ(𝑡 − 3) ℒ−1
𝑠−1 𝑠 − 1 𝑡→𝑡−3
{
0 if 0 ≤ 𝑡 < 3,
= ℎ(𝑡 − 3) (𝑒𝑡 )∣𝑡→𝑡−3 = 𝑒𝑡−3 ℎ(𝑡 − 3) =
𝑒𝑡−3 if 𝑡 ≥ 3.
1 Solutions 119
−3𝑠
{ } { }
𝑒 1
29. ℒ−1 = ℎ(𝑡 − 3) ℒ−1
𝑠2 𝑠2 𝑡→𝑡−3
{
0 if 0 ≤ 𝑡 < 3,
= ℎ(𝑡 − 3) (𝑡)∣𝑡→𝑡−3 = (𝑡 − 3)ℎ(𝑡 − 3) =
𝑡 − 3 if 𝑡 ≥ 3.

𝑒−3𝑠
{ } { }
−1 −1 1
30. ℒ = ℎ(𝑡 − 3) ℒ
(𝑠 − 1)3 (𝑠 − 1)3 𝑡→(𝑡−3)
= ℎ(𝑡 − 3) 12 𝑡2 𝑒𝑡 𝑡→𝑡−3 = 21 (𝑡 − 3)2 𝑒𝑡−3 ℎ(𝑡 − 3)
( )
{
0 if 0 ≤ 𝑡 < 3,
= 1 2 𝑡−3
2 (𝑡 − 3) 𝑒 if 𝑡 ≥ 3.

𝑒−𝜋𝑠
{ } { }
−1 −1 1
31. ℒ 2
= ℎ(𝑡 − 𝜋) ℒ 2

𝑠 +1 𝑠 + 1 𝑡→𝑡−𝜋
= ℎ(𝑡 − 𝜋) (sin 𝑡)∣𝑡→𝑡−𝜋 = ℎ(𝑡 − 𝜋) sin(𝑡 − 𝜋)
{ {
0 if 0 ≤ 𝑡 < 𝜋, 0 if 0 ≤ 𝑡 < 𝜋,
= =
sin(𝑡 − 𝜋) if 𝑡 ≥ 𝜋 − sin 𝑡 if 𝑡 ≥ 𝜋.

𝑠𝑒−3𝜋𝑠
{ } { }
−1 −1 𝑠
32. ℒ = ℎ(𝑡 − 3𝜋) ℒ
𝑠2 + 1 𝑠2 + 1 𝑡→𝑡−3𝜋
= ℎ(𝑡 − 3𝜋) (cos 𝑡)∣𝑡→𝑡−3𝜋 = ℎ(𝑡 − 3𝜋) cos(𝑡 − 3𝜋)
{ {
0 if 0 ≤ 𝑡 < 3𝜋, 0 if 0 ≤ 𝑡 < 3𝜋,
= =
cos(𝑡 − 3𝜋) if 𝑡 ≥ 3𝜋 − cos 𝑡 if 𝑡 ≥ 3𝜋.

𝑒−𝜋𝑠
{ } { }
1
33. ℒ−1 2
= ℎ(𝑡 − 𝜋) ℒ −1
2

𝑠 + 2𝑠 + 5 𝑠 + 2𝑠 + 5 𝑡→𝑡−𝜋
{ }
1
= ℎ(𝑡 − 𝜋) ( 21 𝑒−𝑡 sin 2𝑡) 𝑡→𝑡−𝜋

= ℎ(𝑡 − 𝜋) ℒ−1
(𝑠 + 1)2 + 22 𝑡→𝑡−𝜋
{
1 −(𝑡−𝜋) 0 if 0 ≤ 𝑡 < 𝜋,
= 2𝑒 sin 2(𝑡 − 𝜋)ℎ(𝑡 − 𝜋) = 1 −(𝑡−𝜋)
2𝑒 sin 2𝑡 if 𝑡 ≥ 𝜋.

𝑒−𝑠 𝑒−2𝑠
{ }
34. ℒ−1 +
𝑠2 3
(𝑠{− 1)} { }
1 1
= ℎ(𝑡 − 1) ℒ−1 2
+ ℎ(𝑡 − 2) ℒ −1
3

𝑠
𝑡→𝑡−1 (𝑠 − 1) 𝑡→𝑡−2
= ℎ(𝑡 − 1) (𝑡)∣𝑡→𝑡−1 + ℎ(𝑡 − 2) ( 21 𝑡2 𝑒𝑡 ) 𝑡→𝑡−2

= (𝑡 − 1)ℎ(𝑡 − 1) + 21 (𝑡 − 2)2 𝑒𝑡−2 ℎ(𝑡 − 2)



0
⎨ if 0 ≤ 𝑡 < 1,
= 𝑡−1 if 1 ≤ 𝑡 < 2,
𝑡 − 1 + 12 (𝑡 − 2)2 𝑒𝑡−2 if 𝑡 ≥ 2.


120 1 Solutions

𝑒−2𝑠
{ } { }
1
35. ℒ−1 2
= ℎ(𝑡 − 2) ℒ −1
2

𝑠 +4 𝑠 + 4 𝑡→𝑡−2
= ℎ(𝑡 − 2) ( 21 sin 2𝑡) 𝑡→𝑡−2 = 12 ℎ(𝑡 − 2) sin 2(𝑡 − 2)

{
0 if 0 ≤ 𝑡 < 2,
= 1
2 sin 2(𝑡 − 2) if 𝑡 ≥ 2.

𝑒−2𝑠
{ } { }
1
36. ℒ−1 = ℎ(𝑡 − 2) ℒ−1
𝑠2 − 4 2
𝑠 − 4 𝑡→𝑡−2
{ ( )}
1 1 1
= ℎ(𝑡 − 2) ℒ−1 −
4 𝑠−2 𝑠+2
𝑡→𝑡−2(
1 2𝑡 1

= ℎ(𝑡 − 2) ( 4 (𝑒 − 𝑒 ) 𝑡→𝑡−2 = 4 ℎ(𝑡 − 2) 𝑒2(𝑡−2) − 𝑒−2(𝑡−2)
−2𝑡
)
{
0 if 0 ≤ 𝑡 < 2,
= 1 ( 2(𝑡−2) −2(𝑡−2)
)
4 𝑒 −𝑒 if 𝑡 ≥ 2.

𝑠𝑒−4𝑠
{ } { }
𝑠
37. ℒ−1 2
= ℎ(𝑡 − 4) ℒ −1
2

𝑠 + 3𝑠 + 2 𝑠 + 3𝑠 + 2 𝑡→𝑡−4
{ }
−1 2 1
= ℎ(𝑡 − 4) ℒ −
𝑠 + 2 𝑠 + 1 𝑡→𝑡−4

= ℎ(𝑡 − 4) (2𝑒−2𝑡 − 𝑒−𝑡 ) 𝑡→𝑡−4 = ℎ(𝑡 − 4) 2𝑒−2(𝑡−4) − 𝑒−(𝑡−4)
( )
{
0 if 0 ≤ 𝑡 < 4,
=
2𝑒−2(𝑡−4) − 𝑒−(𝑡−4) if 𝑡 ≥ 4.

𝑒−2𝑠 + 𝑒−3𝑠
{ } { } { }
−1 −1 1 −1 1
38. ℒ = ℎ(𝑡−2) ℒ +ℎ(𝑡−3) ℒ
𝑠2 − 3𝑠 + 2 𝑠2 − 3𝑠 + 2 𝑡→𝑡−2 𝑠2 − 3𝑠 + 2 𝑡→𝑡−3
{ }
1 1
= ℎ(𝑡 − 2) ℒ−1 −
𝑠 − 2 𝑠 − 1 𝑡→𝑡−2
{ }
1 1
+ ℎ(𝑡 − 3) ℒ−1 −
𝑠 − 2 𝑠 − 1 𝑡→𝑡−3

= ℎ(𝑡 − 2) (𝑒2𝑡 − 𝑒𝑡 ) 𝑡→𝑡−2 + ℎ(𝑡 − 3) (𝑒2𝑡 − 𝑒𝑡 ) 𝑡→𝑡−3
= ℎ(𝑡 − 2) 𝑒2(𝑡−2) − 𝑒𝑡−2 + ℎ(𝑡 − 3) 𝑒2(𝑡−3) − 𝑒𝑡−3
( ) ( )

1 − 𝑒−5𝑠
{ } { } { }
−1 −1 1 −1 1
39. ℒ = ℒ − ℎ(𝑡 − 5) ℒ
𝑠2 𝑠2 𝑠2 𝑡→𝑡−5
{
𝑡 if 0 ≤ 𝑡 < 5,
= 𝑡 − ℎ(𝑡 − 5) (𝑡)∣𝑡→𝑡−5 = 𝑡 − (𝑡 − 5)ℎ(𝑡 − 5) =
5 if 𝑡 ≥ 5.
1 Solutions 121
−3𝑠
{ } { } { }
1+𝑒 1 1
40. ℒ−1 = ℒ−1 + ℎ(𝑡 − 3) ℒ−1
𝑠4 𝑠4 𝑠4
𝑡→𝑡−3
= 61 𝑡3 + ℎ(𝑡 − 3) 61 𝑡3 𝑡→𝑡−3 = 16 𝑡3 + 16 (𝑡 − 3)3 ℎ(𝑡 − 3)
( )
{
1 3
𝑡 if 0 ≤ 𝑡 < 3,
= 16 3 1 3
6 𝑡 + 6 (𝑡 − 3) if 𝑡 ≥ 3.
{ } { }
2𝑠 + 1 2𝑠 + 1
41. ℒ−1 𝑒−𝜋𝑠 2 = ℎ(𝑡 − 𝜋)ℒ−1 2

𝑠 + 6𝑠 + 13 𝑠 + 6𝑠 + 13 𝑡→𝑡−𝜋
{ }
2(𝑠 + 3) − 5
= ℎ(𝑡 − 𝜋) ℒ−1
(𝑠 + 3)2 + 22 𝑡→𝑡−𝜋
{ }
2(𝑠 + 3)
= ℎ(𝑡 − 𝜋) ℒ−1 2 2

(𝑠 + 3) + 2 𝑡→𝑡−𝜋
{ }
−5
+ ℎ(𝑡 − 𝜋) ℒ−1 2 2

(𝑠 + 3) + 2 𝑡→𝑡−𝜋
= ℎ(𝑡 − 𝜋) (2𝑒−3𝑡 cos 2𝑡 − 25 𝑒−3𝑡 sin 2𝑡) 𝑡→𝑡−𝜋

−3(𝑡−𝜋)
2 cos 2(𝑡 − 𝜋) − 25 sin 2(𝑡 − 𝜋)
( )
= ℎ(𝑡
{ − 𝜋)𝑒
0 if 0 ≤ 𝑡 < 𝜋,
= 5
𝑒−3(𝑡−𝜋) 2 cos 2𝑡 −
( )
2 sin 2𝑡 if 𝑡 ≥ 𝜋.

42. This is the same calculation as the previous exercise with the additional
term added:
{ }
−1 2𝑠 + 1 5
ℒ 2
= 2𝑒−3𝑡 cos 2𝑡 − 𝑒−3𝑡 sin 2𝑡.
𝑠 + 6𝑠 + 13 2
{ }
−1 −𝜋𝑠 2𝑠 + 1
Combining this with the previous calculation gives ℒ (1 − 𝑒 ) 2 =
{ } 𝑠 + 6𝑠 + 13
2𝑠 + 1
ℒ−1 2
𝑠{ + 6𝑠 + 13 }
−1 2𝑠 + 1
−ℒ 𝑒−𝜋𝑠 2
𝑠 + 6𝑠 + 13)
= 𝑒−3𝑡 2 cos 2𝑡 − 25 sin 2𝑡
(

− ℎ(𝑡 − 𝜋)𝑒−3(𝑡−𝜋) 2 cos 2(𝑡 − 𝜋) − 25 sin 2(𝑡 − 𝜋)


( )

Section 5.3
1. 𝑦 = − 23 ℎ(𝑡 − 1) 1 − 𝑒−2(𝑡−1)
( )

2. 𝑦 = 𝑒−2𝑡 − 1 + 2ℎ(𝑡 − 1) 1 − 𝑒−2(𝑡−1)


( )

3. 𝑦 = ℎ(𝑡 − 1) 1 − 𝑒−2(𝑡−1) − ℎ(𝑡 − 3) 1 − 𝑒−2(𝑡−3)


( ) ( )

4. 𝑦 = 41 𝑒−2𝑡 − 14 + 12 𝑡 + ℎ(𝑡 − 1) 41 𝑒−2(𝑡−1) − 14 + 12 (𝑡 − 1) − 1


( )
2 ℎ(𝑡 −
1) 1 − 𝑒−2(𝑡−1)
( )
122 1 Solutions

5. − 91 ℎ(𝑡 − 3) (−1 + cos 3(𝑡 − 3))

6. 𝑦 = − 32 𝑒𝑡 + 5 4𝑡 1 1
− 5) −3 + 4𝑒𝑡−5 − 𝑒4(𝑡−5)
( )
12 𝑒 + 4 + 12 ℎ(𝑡

7. 𝑦 = 31 ℎ(𝑡 − 1) 1 − 3𝑒−2(𝑡−1) + 2𝑒−3(𝑡−1)


( )

+ 31 ℎ(𝑡 − 3) −1 + 3𝑒−3(𝑡−3) − 2𝑒−3(𝑡−3)


( )

1
8. 𝑦 = cos 3𝑡 + 24 ℎ(𝑡
− 2𝜋) (3 sin 𝑡 − sin 3𝑡)

9. 𝑦 = 𝑡𝑒−𝑡 + ℎ(𝑡 − 3) 1 − (𝑡 − 2)𝑒−(𝑡−3)


( )

10. 𝑦 = 𝑡𝑒−𝑡 − 14 ℎ(𝑡 − 3) −𝑒𝑡 − 5𝑒−𝑡+6 + 2𝑡𝑒−𝑡+6


( )

1 −5𝑡
− 14 𝑒−𝑡 + 51 + 20
1
ℎ(𝑡 − 2) 4 + 𝑒−5(𝑡−2) − 5𝑒−(𝑡−2) + 20 1
( )
11. 𝑦 = 20 𝑒 ℎ(𝑡 −
−5(𝑡−4) −(𝑡−4) 1 −5(𝑡−6) −(𝑡−6)
( ) ( )
4) 4 + 𝑒 − 5𝑒 + 20 ℎ(𝑡 − 6) 4 + 𝑒 − 5𝑒

Section 5.4
1. 𝑦 = ℎ(𝑡 − 1)𝑒−2(𝑡−1)

2. 𝑦 = (1 + ℎ(𝑡 − 1))𝑒−2(𝑡−1)

3. 𝑦 = ℎ(𝑡 − 1)𝑒−2(𝑡−1) − ℎ(𝑡 − 3)𝑒−2(𝑡−3)


{
1
1 sin 2𝑡 if 0 ≤ 𝑡 < 𝜋,
4. 𝑦 = 2 (1 + ℎ(𝑡 − 𝜋)) sin 2𝑡 = 2
sin 2𝑡 if 𝑡 ≥ 𝜋.
{
1
sin 2𝑡 if 𝜋 ≤ 𝑡 < 2𝜋,
5. 𝑦 = 21 𝜒[𝜋, 2𝜋) sin 2𝑡 = 2
0 otherwise.

6. 𝑦 = cos 2𝑡 + 21 𝜒[𝜋, 2𝜋) sin 2𝑡

7. 𝑦 = (𝑡 − 1)𝑒−2(𝑡−1) ℎ(𝑡 − 1)

8. 𝑦 = (𝑡 − 1) 𝑒−2𝑡 + 𝑒−2(𝑡−1) ℎ(𝑡 − 1)


( )

9. 𝑦 = 3ℎ(𝑡 − 1)𝑒−2(𝑡−1) sin(𝑡 − 1)

10. 𝑦 = 𝑒−2𝑡 (sin 𝑡 − cos 𝑡) + 3ℎ(𝑡 − 1)𝑒−2(𝑡−1) sin(𝑡 − 1)

11. 𝑦 = 𝑒−2𝑡 cos 4𝑡 + 21 sin 4𝑡


( )

+ 14 sin 4𝑡 ℎ(𝑡 − 𝜋)𝑒−2(𝑡−𝜋) − ℎ(𝑡 − 2𝜋)𝑒−2(𝑡−2𝜋)


( )

1
𝑒 − 𝑒−𝑡 − 6𝑡𝑒−𝑡 + 16 ℎ(𝑡 − 3) 𝑒5(𝑡−3) − 𝑒−(𝑡−3)
( 5𝑡 ) ( )
12. 𝑦 = 18
1 Solutions 123

Section 6.1
⎡ ⎤ ⎡ ⎤
[ ] [ ] 1 −1 −1 2 0 8
−3 1 63
2. 𝐴𝐵 = , 𝐴𝐶 = , 𝐵𝐴 = ⎣5 −2 18 ⎦, 𝐶𝐴 = ⎣−2 3 7⎦
−3 5 46
0 1 5 3 −1 7
⎡ ⎤
[ ] 3 −1 7
3 4
3. 𝐴(𝐵 + 𝐶) = 𝐴𝐵 + 𝐴𝐶 = , (𝐵 + 𝐶)𝐴 = ⎣3 1 25⎦
1 13
5 0 12
⎡ ⎤
−2 5
4. 𝐶 = ⎣−13 −8⎦
7 0
⎡ ⎤
6 4 −1 −8
5. 𝐴𝐵 = ⎣0 2 −8 2 ⎦
2 −1 9 −5
[ ]
2 3 −8
6. 𝐵𝐶 =
−2 0 24
⎡ ⎤
8 0
⎢ 4 −5⎥
7. 𝐶𝐴 = ⎢ ⎣ 8 14 ⎦

10 11
⎡ ⎤
6 0 2
⎢ 4 2 −1⎥
8. 𝐵 𝑡 𝐴𝑡 = ⎢⎣−1 −8 9 ⎦

−8 2 −5
⎡ ⎤
8 9 −48
9. 𝐴𝐵𝐶 = ⎣ 4 0 −48⎦ .
−2 3 40
⎡ ⎤
1 4 3 1
⎢0 0 0 0⎥
10. 𝐴𝐵 = −4 and 𝐵𝐴 = ⎢ ⎣−1 −4 −3 −1⎦

−2 −8 −6 −2
[ ]
1 0
14.
1 −1
⎡ ⎤
0 0 1
15. ⎣3 −5 −1⎦
0 0 5
124 1 Solutions
[ ]
𝑎𝑏 0
16. 𝐴𝐵 − 𝐵𝐴 = . It is not possible to have 𝑎𝑏 = 1 and −𝑎𝑏 = 1
0 −𝑎𝑏
since 1 ∕= −1.
] [ [ ]
01 00
17. (a) Choose, for example, 𝐴 = and 𝐵 = .
00 10
(b) (𝐴 + 𝐵)2 = 𝐴2 + 2𝐴𝐵 + 𝐵 2 precisely when 𝐴𝐵 = 𝐵𝐴.
[ ] [ ]
11 12
18. 𝐴2 = , 𝐴3 =
12 23
[ ]
1𝑛
19. 𝐵 𝑛 =
01
[ 𝑛 ]
𝑎 0
20. 𝐴𝑛 =
0 𝑏𝑛
[ ] [ ] [ ]
01 𝑣2 1𝑐
21. (a) 𝐴 = ; the two rows of 𝐴 are switched. (b) 𝐴 =
[ 1 0 ] 𝑣 1 01
𝑣1 + 𝑐𝑣2
; to the first row is added 𝑐 times the second row while the
𝑣2
second row is unchanged, (c) to the second row is added 𝑐 times the first
row while the first row is unchanged. (d) the first row is multiplied by 𝑎
while the second row is unchanged, (e) the second row is multiplied by 𝑎
while the first row is unchanged.

Section 6.2
⎡ ⎤ ⎡ ⎤ ⎡ ⎤
1 4 3 ⎡ ⎤ 2 14 3 2
⎢1 𝑥
1 −1⎥
⎥, x = ⎣ 𝑦 ⎦, b = ⎢4⎥, and [𝐴∣b] = ⎢ 1 1 −1 4 ⎥.
⎢ ⎥ ⎢ ⎥
1. (a) 𝐴 = ⎢
⎣2 0 1⎦ ⎣1 ⎦ ⎣2 0 1 1⎦
𝑧
0 1 −1 6 0 1 −1 6
⎡ ⎤
[ ] 𝑥1 [ ] [ ]
2 −3 4 1 ⎢ 𝑥2 ⎥ 0 2 −3 4 1 0
(b) 𝐴 = , x = ⎣ ⎦, b =
⎢ ⎥ , and [𝐴∣b] = .
3 8 −3 −6 𝑥3 1 3 8 −3 −6 1
𝑥4

𝑥1 − 𝑥3 + 4𝑥4 + 3𝑥5 = 2
5𝑥1 + 3𝑥2 − 3𝑥3 − 𝑥4 − 3𝑥5 = 1
2.
3𝑥1 − 2𝑥2 + 8𝑥3 + 4𝑥4 − 3𝑥5 = 3
−8𝑥1 + 2𝑥2 + 2𝑥4 + 𝑥5 = −4
⎡ ⎤
10 1
3. 𝑝2,3 (𝐴) = ⎣0 1 4 ⎦
000
1 Solutions 125

4. RREF
[ ]
1 0 −5 −2 −1
5. 𝑡2,1 (−2)(𝐴) =
01 3 1 1
⎡ ⎤
010 3
6. 𝑚2 (1/2)(𝐴) = ⎣0 0 1 3 ⎦
0000

7. RREF
⎡ ⎤
1010 3
8. 𝑡1,3 (−3)(𝐴) = ⎣0 1 3 4 1 ⎦
00000
⎡ ⎤
100 2
9. ⎣0 1 0 1 ⎦
0 0 1 −1
⎡ ⎤
1 0 0 −11 −8
10. ⎣0 1 0 −4 −2⎦
001 9 6

0 1 0 72 14
⎡ ⎤
⎢0 0 1 3 1 ⎥
11. ⎢ 2⎥
⎣0 0 0 0 0 ⎦
00000
⎡ ⎤
1200 3
⎢0 0 1 0 2 ⎥
12. ⎢⎣0 0 0 1 0 ⎦

00000
⎡ ⎤
100 1 1 1
13. ⎣0 1 0 −1 3 1 ⎦
001 2 11
⎡ ⎤
10 2
⎢0 1 1 ⎥
⎢ ⎥
14. ⎢⎢0 0 0 ⎥

⎣0 0 0 ⎦
000
⎡ ⎤
1010
⎢0 1 3 0 ⎥
15. ⎢⎣0 0 0 1 ⎦

0000
126 1 Solutions
⎡ ⎤
14003
16. ⎣0 0 1 0 1⎦
00013
⎡ ⎤
1 0 0 0 0
⎢0 ‘1 −1 0 0 ⎥
17. ⎢
⎣0 0 0 1 −1⎦

0 0 0 0 0
⎡ ⎤ ⎡ ⎤ ⎡ ⎤
𝑥 −1 −3
18. ⎣𝑦⎦ = ⎣ 1 ⎦ + 𝛼⎣ 1 ⎦
𝑧 0 5
⎡ ⎤ ⎡ ⎤ ⎡ ⎤ ⎡ ⎤
𝑥1 4 −1 −2
⎢ 𝑥2 ⎥ ⎢−1⎥ ⎢−3⎥ ⎢−1⎥
19. ⎢ ⎥ = ⎢ ⎥+ 𝛼⎢ ⎥ +𝛽⎢ ⎥
⎣ 𝑥3 ⎦ ⎣ 0 ⎦ ⎣1⎦ ⎣0⎦
𝑥4 0 0 1
[ ] [ ]
𝑥 −2
20. =𝛼
𝑦 1
⎡ ⎤ ⎡ ⎤ ⎡ ⎤
𝑥1 3 −4
⎢ 𝑥2 ⎥ ⎢ 0 ⎥ ⎢1⎥
21. ⎢ ⎥ = ⎢ ⎥+ 𝛼⎢ ⎥
⎣ 𝑥3 ⎦ ⎣−2⎦ ⎣0⎦
𝑥4 5 0
⎡ ⎤ ⎡ ⎤
𝑥 14/3
22. ⎣ 𝑦 ⎦ = ⎣ 1/3 ⎦
𝑧 −2/3

23. no solution
⎡ ⎤ ⎡ ⎤
0 1
24. ⎣ 3 ⎦ + 𝛼 ⎣ 0 ⎦
4 0
⎡ ⎤ ⎡ ⎤ ⎡ ⎤
5 1 1
25. The equation ⎣−1⎦ = 𝑎 ⎣ 1 ⎦ + 𝑏 ⎣−1⎦ has solution 𝑎 = 2 and 𝑏 = 3. By
⎡4 ⎤ 2 0
5
Proposition 6 ⎣−1⎦ is a solution.
4

26. 𝑘 = 2

27. 𝑘 = 2
1 Solutions 127
⎡ ⎤
−7/2
28. (a) If x𝑖 is the solution set for 𝐴x = b𝑖 then x1 = ⎣ 7/2 ⎦, x2 =
−3/2
⎡ ⎤ ⎡ ⎤
−3/2 7
⎣ 3/2 ⎦, and x3 = ⎣−6⎦.
−1/2 3
(b) The augmented matrix [𝐴∣𝑏1 ∣𝑏2 ∣𝑏3 ] reduces to
⎡ ⎤
1 0 0 −7/2 −3/2 7
⎣0 1 0 7/2 3/2 −6 ⎦ .
0 0 1 −3/2 −1/2 3

The last three columns correspond in order to the solutions.

Section 6.3
[ ]
4 −1
1.
−3 1
[ ]
3 −2
2.
−4 3

3. not invertible
[ ]
−2 1
4.
−3/2 1/2

5. not invertible
⎡ ⎤
1 −1 1
6. ⎣0 1 −2⎦
0 0 1
⎡ ⎤
−6 5 13
7. ⎣ 5 −4 −11⎦
−1 1 3
⎡ ⎤
−1/5 2/5 2/5
8. ⎣−1/5 −1/10 2/5 ⎦
−3/5 1/5 1/5
⎡ ⎤
−29 39/2 −22 13
⎢ 7 −9/2 5 −3 ⎥
9. ⎢
⎣−22 29/2 −17 10 ⎦

9 −6 7 −4
128 1 Solutions
⎡ ⎤
−1 0 0 −1
⎢ 0 −1 0 −1 ⎥
10. 21 ⎢
⎣ 0 0 −1 −1 ⎦

−1 −1 −1 −1
⎡ ⎤
0 0 −1 1
⎢1 0 0 0⎥
11. ⎢
⎣ 0 1 1 −1⎦

−1 −1 0 1

12. not invertible


[ ]
5
13. b =
−3
⎡ ⎤
−2
14. b = ⎣ 6 ⎦
−3
⎡ ⎤
16
1 ⎣
15. b = 10 11 ⎦
18
⎡ ⎤
1
16. b = ⎣ 1 ⎦
1
⎡ ⎤
19
⎢ −4 ⎥
17. b = ⎢⎣ 15 ⎦

−6
⎡ ⎤
3
⎢1⎥
18. b = ⎢⎣−4⎦ .

19. (𝐴𝑡 )−1 = (𝐴−1 )𝑡

20. (𝐸(𝜃))−1 = 𝐸(−𝜃)

21. 𝐹 (𝜃)−1 = 𝐹 (−𝜃)


[ ] [ ] [ ]
10 12 12
22. Example. 𝐴 = ,𝐵= ,𝐶= .
00 34 56
1 Solutions 129

Section 6.4
1. 1

2. 0

3. 10

4. 8

5. −21

6. 6

7. 2

8. 15

9. 0
[ ]
1 −2 + 𝑠 2
10. 𝑠2 −3𝑠 𝑠 = 0, 3
1 −1 + 𝑠
[ ]
1 𝑠−3 1
11. 𝑠2 −6𝑠+8 𝑠 = 2, 4
1 𝑠−3
[ ]
1 𝑠−1 1
12. 𝑠2 −2𝑠+𝑠 𝑠=1±𝑖
−1 𝑠 − 1

(𝑠 − 1)2
⎡ ⎤
3 𝑠−1
1
13. (𝑠−1)3
⎣ 0 (𝑠 − 1)2 0 ⎦ 𝑠=1
0 3(𝑠 − 1) (𝑠 − 1)2
⎡ 2 ⎤
𝑠 − 2𝑠 + 10 −3𝑠 − 6 3𝑠 − 12
1 ⎣ −3𝑠 + 12 𝑠2 − 2𝑠 − 8 3𝑠 − 12 ⎦ 𝑠 = −2, 1, 4
14. 𝑠3 −3𝑠2 −6𝑠+8
3𝑠 + 6 −3𝑠 − 6 𝑠2 − 2𝑠 − 8
⎡ 2 ⎤
𝑠 + 𝑠 4𝑠 + 4 0
1 ⎣−𝑠 − 1 𝑠2 + 𝑠 0 ⎦ 𝑠 = −1, ±2𝑖
15. 𝑠3 +𝑠2 +4𝑠+4
𝑠 − 4 4𝑠 + 4 𝑠2 + 4
[ ]
9 −4
16.
−2 1
17. no inverse
[ ]
1 6 −4
18. 10
−2 3
130 1 Solutions
⎡ ⎤
4 −4 4
1⎣
19. 8 −1 3 −1

−5 −1 3
⎡ ⎤
27 −12 3
1 ⎣
20. 21
−13 5 4 ⎦
−29 16 −4
⎡ ⎤
2 −98 9502
1⎣
21. 6 0 3 −297

0 0 6
⎡ ⎤
−13 76 −80 35
1 ⎢−14 76 −80 36 ⎥
⎢ ⎥
22. 2 ⎣ 6 −34 36 −16 ⎦

7 −36 38 −17
⎡ ⎤
55 −95 44 −171
1 ⎢50 −85 40 −150 ⎥
⎢ ⎥
23. 15 ⎣70 −125 59 −216 ⎦

65 −115 52 −198
24. no inverse

Section 7.1
1. nonlinear

2. linear,( constant
) coefficient,
( 2) non homogeneous
1 1 𝑡
y′ = y+
−1 1 1

3. linear,( homogeneous,
) but not constant coefficient
′ sin 𝑡 −1
y = y
1 cos 𝑡

4. nonlinear, because of the presence of sin 𝑦1 or cos 𝑦2 .

5. linear,⎛constant
⎞ coefficient, homogeneous
1000
⎜2 0 0 1⎟
y′ = ⎜
⎝0 0 0 1⎠ y

0120

6. linear,( constant
) coefficient,
( ) nonhomogeneous
1
′ −1 5
y = 2 1 y+
−1 2 −5
1 Solutions 131

7. First note that 𝑦1 (0) = [0 and] 𝑦2 (0)[ = 1, so ]the initial [ condition is


′ 𝑡 3𝑡
]
𝑦 (𝑡) 𝑒 − 3𝑒 5 −2
satisfied. Then 𝒚 ′ (𝑡) = 1
= while 𝒚(𝑡) =
[ 𝑡 𝑦2′ (𝑡)[ 2𝑒𝑡]− 3𝑒3𝑡 4[−1 ]
5(𝑒 − 𝑒3𝑡 ) − 2(2𝑒𝑡 − 𝑒3𝑡 ) 𝑒𝑡 − 3𝑒3𝑡
]
5 −2
𝑡 3𝑡 𝑡 3𝑡 = . Thus 𝒚 ′ (𝑡) = 𝒚, as
4(𝑒 − 𝑒 ) − (2𝑒 − 𝑒 ) 2𝑒𝑡 − 3𝑒3𝑡 4 −1
required.
[ ] [ ]
𝑦 𝑦
In solutions, 11–16, 𝒚 = 1 = ′ .
𝑦2 𝑦
11. Let 𝑦1 = 𝑦 and 𝑦2 = 𝑦 ′ . Then 𝑦1′[ =]𝑦 ′ = 𝑦2 and 𝑦2′ = 𝑦 ′′ = −5𝑦 ′ −6𝑦+𝑒2𝑡 =
𝑦
−6𝑦1 − 5𝑦2 + 𝑒2𝑡 . Letting 𝒚 = 1 , this can be expressed in vector form
𝑦2
as [ ] [ ]
0 1 0
𝒚′ = 𝒚 + 2𝑡
−6 −5 𝑒
[ ]
1
𝒚(0) = .
−2
( )
′ 0 1
12. y = 2 y
−𝑘
( ) 0
−1
y(0) =
0
( )
0 1
13. y′ = 2 y
𝑘
( 0)
−1
y(0) =
0
( ) ( )
′ 0 1 0
14. y = y+
−𝑘 2 0 cos 𝜔𝑡
( )
0
y(0) =
0

0 1
[ ] [ ]
′ 𝛼
15. 𝒚 = 𝑐 𝑏 𝒚, 𝒚(0) =
− − 𝛽
𝑎 𝑎
0 1
[ ] [ ]
′ −2
16. 𝒚 = 1 2 𝒚, 𝒚(1) =
− 2 − 3
𝑡 𝑡
0 1
[ ] [ ] [ ]
0 𝛼
17. 𝒚 ′ = 𝑐 𝑏 𝒚+ , 𝒚(0) =
− − 𝐴 sin 𝜔𝑡 𝛽
𝑎 𝑎
[ ]
′ −2 sin 2𝑡 2 cos 2𝑡
18. 𝐴 (𝑡) =
−2 cos 2𝑡 −2 sin 2𝑡
132 1 Solutions

−3𝑒−3𝑡 1
[ ]
19. 𝐴′ (𝑡) =
2𝑡 2𝑒2𝑡
⎡ −𝑡
−𝑒 (1 − 𝑡)𝑒−𝑡 (2𝑡 − 𝑡2 )𝑒−𝑡

20. 𝐴′ (𝑡) = ⎣ 0 −𝑒−𝑡 (1 − 𝑡)𝑒−𝑡 ⎦
0 0 −𝑒−𝑡
⎡ ⎤
1
21. 𝒚 ′ (𝑡) = ⎣ 2𝑡 ⎦
𝑡−1
[ ]
00
22. 𝐴′ (𝑡) =
00

𝒗 ′ (𝑡) = −2𝑒−2𝑡 𝑡22𝑡


[ ]
23. +1 −3 sin 3𝑡
[ ]
0 1
24.
−1 0
[ 2
𝑒 − 𝑒−2 𝑒2 + 𝑒−2 − 1
]
1
25. 4 1 − 𝑒2 − 𝑒−2 𝑒2 − 𝑒−2
⎡ ⎤
3/2
26. ⎣ 7/3 ⎦
ln 4 − 1
[ ]
4 8
27.
12 16

28. Continuous on 𝐼1 , 𝐼4 , and 𝐼5


[1 1 ]
𝑠 𝑠2
29. 2 1
𝑠3 𝑠−2

𝑠 1
[ ]
𝑠2 +1 𝑠2 +1
30. −1 𝑠
𝑠2 +1 𝑠2 +1

3! 2𝑠 1
[ ]
𝑠4 (𝑠2 +1)2 (𝑠+1)2
31. 2−𝑠 𝑠−3 3
𝑠3 𝑠2 −6𝑠+13 𝑠
⎡ 1 ⎤
𝑠2
⎢2⎥
32. ⎣ 𝑠3 ⎦
6
𝑠4
[ ]
2 1 −1
33. 𝑠2 −1 −1 1
1 Solutions 133
⎡1 1 1 ⎤
𝑠 𝑠2+1 𝑠(𝑠2 +1)
𝑠 1
34. ⎣ 0
⎢ ⎥
𝑠2 +1 𝑠2 +1 ⎦
0 𝑠−1
+1
𝑠
𝑠2 +1

35. 1 2𝑡 3𝑡2
[ ]

[ ]
1 𝑡
36.
𝑒𝑡 +𝑒−𝑡
2 cos 𝑡
[ ]
𝑒𝑡 + 𝑒−𝑡 𝑒𝑡 − 𝑒−𝑡
37.
𝑒𝑡 − 𝑒−𝑡 𝑒𝑡 + 𝑒−𝑡

𝑒𝑡 𝑡𝑒𝑡
⎡ ⎤

38. ⎣ −4 𝑒−3𝑡
3 + + 𝑒𝑡 sin 𝑡⎦
⎢ ⎥
3
3 cos 3𝑡 𝑒3𝑡

Section 7.2
1
[ ]
0 𝑒𝑡 0
[ ]
−1 𝑠−1 −1 −1
1. (𝑠𝐼 − 𝐴) = and ℒ (𝑠𝐼 − 𝐴) =
0 1 0 𝑒2 𝑡
𝑠−2

𝑠−2 −1
[ ] [2 ]
𝑠(𝑠−3) 𝑠(𝑠−3) 3 + 31 𝑒3𝑡 1
3 − 13 𝑒3𝑡
2. (𝑠𝐼 − 𝐴)−1 = −2 𝑠−1 and ℒ−1 (𝑠𝐼 − 𝐴)−1 = 2 2 3𝑡 1
𝑠(𝑠−3) 𝑠(𝑠−3) 3 − 3𝑒 3 + 23 𝑒3𝑡
⎡1 1 𝑠+1 ⎤ 𝑡2
⎡ ⎤
𝑠 𝑠2 𝑠3 1 𝑡 𝑡+ 2
⎢ 1 1 ⎥
3. (𝑠𝐼 − 𝐴)−1 = ⎣ 0 𝑠 𝑠2 ⎦ and ℒ−1 (𝑠𝐼 − 𝐴)−1 = ⎣0 1
⎢ ⎥
𝑡 ⎦
0 0 1𝑠 00 1
𝑠 1
[ ] [ ]
−1 𝑠2 +1 𝑠2 +1 −1 −1 cos 𝑡 sin 𝑡
4. (𝑠𝐼 − 𝐴) = and ℒ (𝑠𝐼 − 𝐴) =
−1 𝑠 − sin 𝑡 cos 𝑡
𝑠2 +1 𝑠2 +1

𝑒−𝑡 0
[ ] [ −𝑡 ]
𝑒
5. ; 𝒚(𝑡) =
0 𝑒3𝑡 −2𝑒3𝑡
[ ] [ ]
cos 2𝑡 sin 2𝑡 cos 2𝑡 − sin 2𝑡
6. ;
− sin 2𝑡 cos 2𝑡 1 sin 2𝑡 + cos 2𝑡
[ 2𝑡 2𝑡 ] [ 2𝑡
−𝑒 + 2𝑡𝑒2𝑡
]
𝑒 𝑡𝑒
7. ; y(𝑡) =
0 𝑒2𝑡 2𝑒2𝑡
[ −𝑡
𝑒 cos 2𝑡 𝑒−𝑡 sin 2𝑡
] [ −𝑡 ]
𝑒 cos 2𝑡
8. ; y(𝑡) =
−𝑒−𝑡 sin 2𝑡 𝑒−𝑡 cos 2𝑡 −𝑒−𝑡 sin 2𝑡
134 1 Solutions

3𝑒𝑡 − 𝑒−𝑡 −𝑒𝑡 + 𝑒−𝑡


[ ] [ −𝑡 ]
1 𝑒
9. ; y(𝑡) =
2 3𝑒𝑡 − 3𝑒−𝑡 −𝑒𝑡 + 3𝑒−𝑡 3𝑒−𝑡
[ 𝑡
𝑒 + 2𝑡𝑒𝑡 −4𝑡𝑒𝑡
[ 𝑡
𝑒 − 2𝑡𝑒𝑡
] ]
10. ; y(𝑡) =
𝑡𝑒𝑡 𝑒𝑡 − 2𝑡𝑒𝑡 𝑒𝑡 − 𝑡𝑒𝑡
[ ] [ ]
cos 𝑡 + 2 sin 𝑡 −5 sin 𝑡 cos 𝑡 + 7 sin 𝑡
11. ; y(𝑡) =
sin 𝑡 cos 𝑡 − 2 sin 𝑡 3 sin 𝑡 − cos 𝑡
[ ] [ ]
𝑒−𝑡 cos 2𝑡 −2𝑒−𝑡 sin 2𝑡 2𝑒−𝑡 (cos 2𝑡 + sin 2𝑡)
12. 1 −𝑡
; y(t) = −𝑡
2𝑒 sin 2𝑡 𝑒−𝑡 cos 2𝑡 𝑒 (sin 2𝑡 − 2 cos 2𝑡)
[ 𝑡 3𝑡 3𝑡 𝑡
] [ 3𝑡 ]
1 𝑒 +𝑒 𝑒 −𝑒 𝑒
13. 2 𝑒3𝑡 − 𝑒𝑡 𝑒𝑡 + 𝑒3𝑡 ; y(𝑡) =
𝑒3𝑡
[ 𝑡
𝑒 + 4𝑡𝑒𝑡 2𝑡𝑒𝑡
[ 𝑡
3𝑒 + 10𝑡𝑒𝑡
] ]
14. ; y(𝑡) =
−8𝑡𝑒𝑡 𝑒𝑡 − 4𝑡𝑒𝑡 −𝑒𝑡 − 20𝑡𝑒𝑡

0 32 (𝑒𝑡 − 𝑒−𝑡 )
⎡ −𝑡
−2𝑒−𝑡 + 3𝑒𝑡
⎤ ⎡ ⎤
𝑒
15. ⎣ 0 𝑒2𝑡 0 ⎦; y(𝑡) = ⎣ 𝑒2𝑡 ⎦
0 0 𝑒 𝑡 2𝑒𝑡

cos 2𝑡 2 sin 2𝑡 0 2 cos 2𝑡 + 2 sin 2𝑡


⎡ ⎤ ⎡ ⎤

16. ⎣ − 21 sin 2𝑡 cos 2𝑡 0 ⎦; y(𝑡) = ⎣ cos 2𝑡 − sin 2𝑡 ⎦


−𝑒 + cos 2𝑡 2 sin 2𝑡 𝑒−𝑡
−𝑡
2 cos 2𝑡 + 2 sin 2𝑡
⎡ −3𝑡
+ 1 3𝑒−𝑡 − 3𝑒−3𝑡 1 − 𝑒−3𝑡 2 + 𝑒−𝑡
⎤ ⎡ ⎤
2𝑒
1⎣
17. 3 0 3𝑒−𝑡 0 ⎦; y(𝑡) = ⎣ 𝑒−𝑡 ⎦
−3𝑡 −𝑡 −3𝑡 −3𝑡
2 − 2𝑒 −3𝑒 + 3𝑒 𝑒 +2 4 − 𝑒−𝑡

−𝑡𝑒𝑡 + 𝑒𝑡 𝑡𝑒𝑡 𝑡𝑒𝑡


⎡ 𝑡
3𝑒 + 𝑡𝑒𝑡
⎡ ⎤ ⎤

18. ⎣ 𝑒2𝑡 − 𝑒𝑡 𝑒𝑡 −𝑒2𝑡 + 𝑒𝑡 ⎦; y(𝑡) = ⎣ 𝑒𝑡 ⎦


−𝑡𝑒 − 𝑒 + 𝑒 𝑡𝑒𝑡 𝑒2𝑡 + 𝑡𝑒𝑡
𝑡 2𝑡 𝑡
3𝑒𝑡 + 𝑡𝑒𝑡

𝑒 𝑡𝑒 − 21 𝑡2 𝑒3𝑡 − 𝑡𝑒3𝑡
⎡ 3𝑡
𝑒 − 𝑡𝑒3𝑡
⎡ 3𝑡 3𝑡 ⎤ ⎤

19. ⎣ 0 𝑒3𝑡 −𝑡𝑒3𝑡 ⎦; y(𝑡) = ⎣ −𝑒3𝑡 ⎦


3𝑡
0 0 𝑒 0
[ −𝑡
𝑒 + 𝑡𝑒𝑡
]
20. y(𝑡) =
3𝑒−𝑡 + 𝑡𝑒𝑡

1 + 2𝑒−𝑡 sin 2𝑡
[ ]
21. y(𝑡) =
−2 + 2𝑒−𝑡 cos 2𝑡
[ ]
1 2 cos 𝑡 + 16 sin 𝑡 + 2𝑡 cos 𝑡 − 𝑡 sin 𝑡
22. y(𝑡) = 2
−2 cos 𝑡 + 7 sin 𝑡 + 𝑡 cos 𝑡
1 Solutions 135

Вам также может понравиться